Está en la página 1de 169

UNIVERSIDAD NACIONAL DEL CALLAO

FACULTAD DE CIENCIAS NATURALES Y MATEMATICA

OPTIMIZACION CONTINUA

ERIK ALEX PAPA QUIROZ

Agosto de 2011

CALLAO - PERU
Gracias por tu paciencia.

ii
Indice

1 Preliminares 3
1.1 Notaciones . . . . . . . . . . . . . . . . . . . . . . . . . . . . . . . . . . . 3
1.2 Resultados de Analisis . . . . . . . . . . . . . . . . . . . . . . . . . . . . 4

2 El Problema de Optimizacion 11
2.1 Problema General de Optimizacion . . . . . . . . . . . . . . . . . . . . . 12
2.2 Clasicacion de Problemas de Optimizacion . . . . . . . . . . . . . . . . 14
2.3 Ejemplos de Problemas de Optimizacion . . . . . . . . . . . . . . . . . . 16

3 Existencia de Puntos de Mnimo Global 33


3.1 Optimos Globales y Locales . . . . . . . . . . . . . . . . . . . . . . . . . 33
3.2 Resultados de Existencia . . . . . . . . . . . . . . . . . . . . . . . . . . . 34
3.3 Ejemplos de Existencia de Puntos Optimos . . . . . . . . . . . . . . . . . 40
3.4 Ejercicios Resueltos . . . . . . . . . . . . . . . . . . . . . . . . . . . . . . 42
3.5 Ejercicios Propuestos . . . . . . . . . . . . . . . . . . . . . . . . . . . . . 48

4 Condiciones de Optimalidad de Problemas Diferenciables 50


4.1 Problemas sin Restricciones . . . . . . . . . . . . . . . . . . . . . . . . . 50
4.2 Restricciones Arbitrarias . . . . . . . . . . . . . . . . . . . . . . . . . . . 56
4.3 Restricciones de Igualdad . . . . . . . . . . . . . . . . . . . . . . . . . . . 59
4.4 Restricciones de Desigualdad . . . . . . . . . . . . . . . . . . . . . . . . . 74
4.5 Restricciones Mixtas . . . . . . . . . . . . . . . . . . . . . . . . . . . . . 84
4.6 Ejercicios Resueltos . . . . . . . . . . . . . . . . . . . . . . . . . . . . . . 88

iii
4.7 Ejercicios Propuestos . . . . . . . . . . . . . . . . . . . . . . . . . . . . . 107

5 Elementos de Convexidad 108


5.1 Elementos de Convexidad . . . . . . . . . . . . . . . . . . . . . . . . . . 108
5.2 El Teorema de la Proyeccion . . . . . . . . . . . . . . . . . . . . . . . . . 113
5.3 Separacion de Conjuntos Convexos . . . . . . . . . . . . . . . . . . . . . 116
5.4 Funciones convexas . . . . . . . . . . . . . . . . . . . . . . . . . . . . . . 121
5.5 Continuidad de Funciones Convexas . . . . . . . . . . . . . . . . . . . . . 129
5.6 Derivada Direccional de Funciones Convexas . . . . . . . . . . . . . . . . 132
5.7 Subdiferenciabilidad . . . . . . . . . . . . . . . . . . . . . . . . . . . . . 134
5.8 Funciones Convexas Diferenciables . . . . . . . . . . . . . . . . . . . . . 143
5.9 Ejercicios Resueltos . . . . . . . . . . . . . . . . . . . . . . . . . . . . . . 144
5.10 Ejercicios Propuestos . . . . . . . . . . . . . . . . . . . . . . . . . . . . . 148

6 Optimizacion Convexa 149


6.1 Minimizacion Convexa Irrestricta . . . . . . . . . . . . . . . . . . . . . . 151
6.2 Minimizacion Convexa Restricta . . . . . . . . . . . . . . . . . . . . . . . 152
6.3 Convexidad Generalizada . . . . . . . . . . . . . . . . . . . . . . . . . . . 154
6.4 Ejercicios Resueltos . . . . . . . . . . . . . . . . . . . . . . . . . . . . . . 155
6.5 Ejercicios Propuestos . . . . . . . . . . . . . . . . . . . . . . . . . . . . . 162

Referencias Bibliograficas 163

iv
Introduccion

El material de la presente edicion trata del estudio formal del problema de encontrar
los puntos de maximos y/o mnimos de una funcion sin restricciones o sujeta a algunas
restricciones sobre su dominio. Esta funcion, llamada funcion objetivo, puede representar
diversos fenomenos en estudio dependiendo del campo de aplicacion, por ejemplo, puede
representar la temperatura, la energa, el benecio de una empresa, el costo de produccion
de terminados productos, etc, es por eso que el tema tiene gran interes en las diversas
lneas de investigacion en ciencias e ingenieras.
El area que estudia de manera rigurosa el problema de encontrar los puntos de
maximos y/o mnimos de una funcion es llamada optimizacion matematica y esta a su
vez, dependiendo cual sea la naturaleza del problema, se divide en optimizacion continua,
combinatoria, entera, dinamica, estocastica, entre otras.
El tema que presentare en esta edicion es la optimizacion continua en espacios eu-
clidianos que es la base para el estudio de las otras subareas de la optimizacion y que
en esencia estudia problemas de optimizacion cuando las variables que denen el prob-
lema no son discretas, esto es, pueden tomar cualquier valor dentro del conjunto de las
restricciones.
Despues de presentar algunos problemas de optimizacion encontrados en las aplica-
ciones, estudiaremos las condiciones sucientes para garantizar la existencia de soluciones
optimas, para luego estudiar las condiciones necesarias y sucientes para caracterizar
optimos de funciones sucientemente regulares. Finalizaremos el trabajo con el estudio
de la convexidad de conjuntos, funciones convexas y problemas de optimizacion convexa.
Debo aclarar que no existe un aporte innovador en la teora presentada pero lo que si
encontraremos, a diferencia de otros textos, es la variedad de ejemplos, contraejemplos y

1
ejercicios resueltos para que la teora quede bien sustentada y entendida.
El material esta motivado por las lecturas de algunos libros de optimizacion que
se encuentran en las referencias bibliogracas y que a lo largo de estos anos he leido
cuidadosamente, es por eso, que algunos de los ejercicios propuestos en esos libros fueron
aqu desarrollados. Tambien, algunos ejemplos de problemas de optimizacion fueron
tomados de recientes trabajos de investigacion que he tenido la oportunidad de revisar
en algunas revistas cientcas y otros como por ejemplo los problemas de optimizacion
semidenida de algunas investigaciones que he realizado con algunos co-autores.
Este trabajo esta dividido en 6 captulos: El primer captulo da algunos preliminares
matematicos usados a lo largo del texto. El segundo captulo presenta el problema general
de optimizacion y algunos ejemplos de problemas en diversos campos de las ciencias que
pueden ser modelados como problemas de optimizacion. El tercer captulo trata sobre la
existencia de soluciones optimas de problemas denidos en espacios euclidianos. El cuarto
captulo presenta las condiciones de optimalidad necesarias y sucientes de problemas de
optimizacion para funciones diferenciables. El quinto captulo trata de la convexidad de
conjuntos, los teoremas de separacion, as como tambien, de las principales propiedades
de las funciones convexas y el sexto captulo trata de problemas de optimizacion convexa.
Para nalizar debo resaltar que este material formo parte del curso de Seminario
de Tesis I, disciplina del noveno ciclo, realizado en la Facultad de Ciencias Naturales y
Matematica de la Universidad Nacional del Callao a nivel de Pre-Grado y por ende me
gustara agradecer a las autoridades de esta institucion por la invitacion para formar
parte de la plana docente, al profesor Runo Moya Calderon por la motivacion indirecta
para continuar este trabajo cuando las fuerzas se desvanecian, al personal Administrativo
que me ha dado toda la comodidad suciente para realizar un trabajo sin contratiempos,
a los colegas por la armoniosa y amigable convivencia en todo este tiempo y agradecer
especialmente a los alumnos que con sus preguntas y sugerencias hicieron nacer la idea
de preparar este texto. A cada uno de ustedes que colaboraron directa e indirectamente
con el presente trabajo un sincero agradecimiento.

Bellavista, 3 de Agosto del 2011


Erik Alex Papa Quiroz.

2
Captulo 1

Preliminares

En este captulo presentamos las notaciones utilizadas a lo largo del texto como
tambien los resultados basicos de analisis real necesarios para el buen entendimiento
de la teora presentada.

1.1 Notaciones
En el decorrer de este libro, adoptaremos las siguientes notaciones:

Rn = {x = (x1 , x2 , . . . , xn ) : xi R, para todo i = 1, . . . , n}.


Rn++ = {x = (x1 , x2 , . . . , xn ) Rn : xi > 0, para todo i = 1, . . . , n}.
R+ : el conjunto de los numeros reales no negativos.
R++ : el conjunto de los numeros reales positivos.
P
Dados x, y Rn , xT y = ni=1 xi yi .

Dado x Rn , kxk := xT x: la norma euclidiana.
(0, 1) := {z R : 0 < z < 1}.
Dado X Rn , X denotara la clausura topologica de X.
Dado X Rn , Front(X) es la frontera de X.
Dado X Rn , int(X) es el interior de X.
x X\Y signica que x X y x
/ Y.
xk x, denota limk+ xk = x.

3
f : el gradiente de f.
B(x, ) = {x Rn : kx xk < } : la bola abierta de centro x y radio .
Dado x = (x1 , x2 , . . . , xn ) Rn , x 0 signica que xi 0, para todo i = 1, . . . , n.
2 f (x) denota la matriz hessiana de f en el punto x.
Ker(h (x)) denota el nucleo de la matriz h (x).
Im h (x)T = {v Rn : v = h (x)T }.
Dado A Rnn , AT denota la transpuesta de A.
Dado A Rnn , A 0 signica que A es denida positiva.
Dado A Rnn , A  0 signica que A es semidenida positiva.
S n = {X Rnn : X = X T }: el espacio de las matrices simetricas.
Dado X S n , tr(X) denota la traza de la matriz X.
Dado X S n , X Y := tr(XY ): produto interno en S n .
Sea un abierto de Rn , C k () = {F : U Rm : f es k veces diferenciable en U }.
C () = {F : Rm : F es una funcion innitamente diferenciable en }.
lim inf k+ f (xk ) = supn inf kn {f (xk )}.
lim supk+ f (xk ) = inf n supkn {f (xk )}.
 R
L2 () = f : C, medible en el sentido de Lebesgue y |f |2 < .
H01 () = {u L2 () : u L2 () u(x) = 0 , x Front()}.
Dado g : X Rn R, x argmax{g(y) : y X}, denota g(x) g(y), para todo
y X.
Dado g : X Rn R, z argmin{g(y) : y X}, denota g(z) g(y), para todo
y X.
ln z denota el logaritmo neperiano de z.
F c = Rn \F denota el complemento del conjunto F .

1.2 Resultados de Analisis


Consideremos el espacio Rn = {x = (x1 , . . . , xn ) : xi R} y denamos dos operaciones:

+ : Rn Rn Rn : +(x, y) := (x1 + y1 , . . . , xn + yn ),

4
: R Rn Rn : (, x) := (x1 , . . . , xn ).

Por simplicidad denotaremos +(x, y) = x + y, y (, x) = x. Observe que Rn con


estas dos operaciones forma un espacio vectorial sobre el cuerpo de los numeros reales R.
As, hemos dotado a Rn con propiedades algebraicas.
Como en todo espacio es necesario medir magnitudes, calcular errores, etc, dotaremos
a Rn de una metrica, para ello denimos:
n
X
n n
d : R R R : d(x, y) := (xi yi )2 .
i=1

Se puede vericar que d cumple las siguientes propiedades:

d1 : Para todo x, y Rn , d(x, y) 0, d(x, y) = 0 si, y solo si, x = y.

d2 : Para todo x, y Rn , d(x, y) = d(y, x).

d3 : Para todo x, y, z Rn , d(x, z) d(x, y) + d(y, z).

La funcion d se llama metrica euclidiana y el espacio vectorial Rn con esta metrica es


llamado epacio euclidiano.
En el espacio euclidiano Rn , denamos
n
X
n
k.k : R R : kxk := d(x, 0) = x2i .
i=1

Esta funcion es llamada norma euclidiana o norma 2 y satisface:

n1 : kxk 0, para todo x Rn ; ||x|| = 0 si, y solo si, x = 0.

n2 : kxk = ||||x||, para todo x Rn , para todo R.

n3 : kx + yk kxk + kyk, para todo x, y Rn .

En el espacio euclidiano tambien es posible introducir un producto interno que per-


mitira denir posteriormente el concepto de angulo entre dos puntos
n
X
n n
h , i : R R R : hx, yi := xi yi .
i=1

El producto interno hx, yi sera denotado tambien por xT y.

5
Observacion 1.1 De las definiciones dadas obtenemos las siguientes igualdades:
p
d(x, y) = ||x y|| = (x y)T (x y).

Dado x Rn y > 0 denamos

B(x, ) = {x Rn : ||x x|| < } .

Definicion 1.1 Un conjunto A Rn es abierto en Rn si para todo x A, existe x > 0


tal que B(x, x ) A.
Un conjunto F Rn es cerrado en Rn si para toda sucesion {xk } F tal que xk x se
tiene que x F .

Definicion 1.2 Dado X Rn , un punto x Rn es llamado punto adherente de X si


para todo > 0, B(x, ) X 6= .
Un punto x Rn es llamado punto de acumulacion de X si para todo > 0, B(x, )
X\{x} 6= .

Definicion 1.3 Dado X Rn , la clausura de X, denotado por X, es el conjunto de


todos los puntos adherentes.

Teorema 1.1 Sea F Rn . F es cerrado en Rn si y solo si F C = Rn \F es abierto en


Rn .

Demostracion. Ver [11], Teorema 17, pagina. 40.

Definicion 1.4 Un conjunto C Rn es llamado acotado si existe M > 0 tal que ||c||
M, para todo c C.

Teorema 1.2 Si {xk } Rn es una sucesion limitada, entonces existe una subsucesion
{xkj } de {xk } convergente.

Demostracion. Ver [11], Teorema 5, pagina. 17.

Definicion 1.5 Sea un conjunto K Rn . K es compacto si y solo si K es cerrado y


acotado.

6
Definicion 1.6 Dado una matriz A Rnn , A es llamada matriz semidefinida positiva,
denotado por A  0, si dT A d 0, para todo d Rn . A es definida positiva si dT A d > 0,
para todo d Rn \ {0} .

Definicion 1.7 Sea f : A Rm una funcion donde A Rn . Se dice que f es continua


en x A si para cada > 0, existe > 0 tal que si ||x x|| < , con x A, entonces
||f (x) f (x)|| < .
Decimos que f es continua en A si f es continua para cada punto x A.

Teorema 1.3 Sea f : A Rm una funcion donde A Rn . f es continua en x si, y solo


si, para toda sucesion {xk } tal que xk x se tiene que f (xk ) f (x).

Demostracion. Ver [11], Teorema 13, pagina. 26.

Definicion 1.8 Sea f : A R una funcion definida en el conjunto abierto A Rn ,


x A y v Rn . La derivada direccional de f en el punto x, en la direccion de v, se
define como el limite
f (x + tv) f (x)
f (x, v) = lim ,
t0 t
cuando este limite existe.

Si tomamos v = ei = (0, 0, ..., 0, 1, 0, ..., 0) (con valor de 1 en la posicion i) entonces la


derivada direccional en esta direccion, si existe el limite, es llamada derivada parcial y
f
denotaremos por xi
(x), esto es,

f f (x + tei ) f (x)
(x) = lim
xi t0 t
Definicion 1.9 Sea f : A R una funcion definida en el conjunto abierto A Rn y
x A. Decimos que la funcion f es diferenciable en x si existen las derivadas parciales
f f f
x1
(x), x 2
(x), ..., x n
(x) y ademas para todo vector v = (v1 , v2 , ..., vn ) tal que x + v A
se tiene:
Xn
f
f (x + v) = f (x) + (x)vi + o(v), (1.1)
i=1
xi
donde
o(v)
lim = 0.
v0 ||v||

7
Definicion 1.10 Sea f : A R una funcion definida en el conjunto abierto A Rn ,
con derivada direccional en el punto x A. La diferencial de f en el punto x es una
aplicacion df (x) : Rn R definida por df (x)(v) = f (x, v).

Definicion 1.11 Dada una funcion f : A R diferenciable en el conjunto abierto


A Rn , definimos el gradiente de f en el punto x A como el vector f (x) tal que

hf (x), vi = df (x)(v),

para todo v Rn .

Observemos de la denicion anterior, que si tomamos v = ei entonces tenemos:

f
hf (x), ei i = df (x)(ei ) = (x),
xi

es por eso que el gradiente queda expresado en las base canonica como
 
f
f (x) = (x), ..., f (x) .
x1 xn

Luego (1.1) queda expresado por:

f (x + v) = f (x) + hf (x), vi + o(v),

donde
o(v)
lim = 0.
v0 ||v||

Sea f : A R una funcion denida en el conjunto abierto A Rn y x A.


2f 2f
Denotamos
( f )(x)
xi xj
por xi xj
(x) y
( f )(x)
xi xi
por x2i
(x) respectivamente.

Definicion 1.12 Sea f : A R una funcion definida en el conjunto abierto A Rn y


x A. Definimos la matriz Hesiana de f en x, el cual sera denotado por 2 f (x), como

2f 2f 2
(x) (x) . . . (x)
x21 x1 x2 x1 xn
2f 2f 2f
x2 x1 (x) 2 (x) . . . x2 xn (x)
f (x) =
2

x 1

... ... ...

2f 2f 2f
xn x1
(x) xn x2 (x) . . . x2
(x)
n

8
Definicion 1.13 Sea f : A R una funcion definida en el conjunto abierto A Rn y
x A. Decimos que la funcion f es dos veces diferenciable en x si existen las derivadas
2f
parciales de segundo orden xi xj
(x) para todo i, j = 1, ..., n, y ademas para todo vector
v Rn tal que x + v A se tiene:
1
f (x + v) = f (x) + hf (x), vi + hv, 2 f (x)vi + o(v 2 ), (1.2)
2
donde
o(v 2 )
lim = 0.
v0 ||v||2

Teorema 1.4 (Teorema del Valor Medio en R)


Si g : [a, b] R es derivable en (a, b) y continua en [a, b], entonces existe c (a, b) tal
que
g(b) g(a)
= g (c).
ba
Demostracion. Ver [3], pagina 224.

Teorema 1.5 (Teorema del Valor Medio en Rn ) Sea f : Rn R diferenciable en todos


los puntos del segmento de recta (a, a + v) y continua en [a, a + v] Rn . Entonces existe
(0, 1) tal que: f (a + v) f (a) = hf (a + v), vi .

Demostracion. Ver [3], pagina 398.

Teorema 1.6 (Teorema de la funcion implcita). Supongamos dado un conjunto de


m ecuaciones y n variables, m < n: gi (x) = 0, para todo i = 1, 2, . . . , m. Sea x0 =
(x01 , . . . , x0m , x0m+1 , . . . , x0n ) Rn satisfaciendo las propiedades:

i) Existe una vecindad V0 de x0 y p 1 tal que las funciones gi C p (V0 )

ii) gi (x0 ) = 0, para todo i = 1, . . . , m.

iii) La matriz Jacobiana de orden m:



g1 g1 g1
(x0 ) (x0 ) ... (x0 )
x1 x2 xm
g2 g2 g2
(x0 ) (x0 ) ... (x0 )
Jg (x ) =
0

x1 x2 xm

... ... ...

gm gm gm
x1
(x0 ) x2
(x0 ) ... xm
(x0 )

9
es no singular. Entonces, existe una vecindad U 0 de (x0m+1 , x0m+2 , . . . , x0n ) Rnm
tal que para cada x = (xm+1 , xm+2 , . . . , xn ) U 0 existen funciones i : Ux
Rnm Rm , donde Ux es una vecindad de x tal que:

a) i C p (Ux )

b) x0i = i (x0m+1 , x0m+2 , . . . , x0n ), para todo i = 1, 2, . . . m

c) gi (1 (x), 2 (x), . . . , m (x), x) = 0, para todo i = 1, 2, . . . m

Demonstracion. Ver [11], pagina 164.

10
Captulo 2

El Problema de Optimizacion

En las diversas aplicaciones de las ciencias e ingenieras a menudo es necesario conocer


los valores maximos o mnimos que pueden alcanzar ciertos objetos en estudio, por ejem-
plo la maxima o mnima temperatura de un determinado cuerpo, el mnimo costo en la
distribucion de la energa electrica, la maxima ganancia o el mnimo costo de produccion
de un determinado producto. Todos estos problemas, como veremos a lo largo de este
texto, pueden ser modelados matematicamente y estudiados de un punto de vista formal
dentro del campo de la Optimizacion.
La Optimizacion es una de las areas de la Matematica Aplicada que estudia el prob-
lema de minimizar o maximizar una funcion sujeta generalmente a restricciones sobre
su dominio. Si bien es cierto que algunas sub-areas de la Optimizacion eran conocidas
siglos atras, por ejemplo el Calculo de Variaciones, la Optimizacion surgio entre los anos
treinta y cuarenta del siglo anterior en Inglaterra dentro del campo de la Investigacion de
Operaciones cuando un grupo de cientcos entre matematicos y fsicos fueron reclutados
para idear estrategias optimas de ataque y defensa contra las tropas alemanas. Debido al
gran exito de estas estrategias, estas fueron usadas tambien por los americanos durante
la segunda guerra mundial cumpliendo un papel decisivo en el descenlace de esta guerra.
Al conocerse que las estrategias de Investigacion de Operaciones eran aplicables
tambien a la Economia, Administracion y Finanzas, la Optimizacion tubo un gran apoyo
economico por parte de las empresas privadas para realizar investigaciones en esta area.
As, se fundaron sociedades de Investigacion de Operaciones en los principales paises

11
desarrollados como por ejemplo: Operational Research Society en Inglaterra, Oper-
ational Research American Society y The Institute of Management Sciences en los
Estados Unidos y La Societe Francaise de Recherche Operationelle en Francia, entre
otros. Paralelamente se crearon diversas Revistas Cientcas dedicadas a la publicacion
de los diversos trabajos de investigacion en Optimizacion y sus aplicaciones.
Actualmente la Optimizacion con ayuda del avance de la Computacion Cientca
conoce aplicaciones en diferentes areas, por ejemplo en Economia, Administracion, Fsica-
Matematica, Ingenieras: Qumica, Biologica, Civil, Ambiental, Electrica, Mecanica, etc,
y se ha convertido en una de las cuatro lneas de investigacion mas estudiadas en el
ambito de las ciencias e ingenieras.
Antes de denir que es un problema de optimizacion es necesario introducir los conceptos
de mnimo y maximo de una funcion.

Definicion 2.1 (Mnimo y maximo global). Sea M un conjunto arbitrario y f : X


M R una funcion definida sobre X.
Un punto x X es llamado punto de mnimo global de f sobre X si

f (x) f (x), x X.

Un punto x X es llamado punto de maximo global de f sobre X si

f (x ) f (x), x X.

2.1 Problema General de Optimizacion


Dado una funcion f : X M R, el problema general de optimizacion consiste en
resolver el siguiente modelo matematico

opt {f (x) : x X} (2.1)

que signica encontrar un punto de mnimo o maximo global de f sobre X, f es llamada


funcion objetivo y X el conjunto de restricciones del problema, ver guras 2.1 y 2.2.
Si el objetivo es encontrar un punto de mnimo global de f, el problema sera denotado

12
f (x )

f (x)

x x

Mnimo global
Maximo global

Figura 2.1: Puntos unicos de mnimo y maximo global

por min {f (x) : x X}. Analogamente, si el objetivo es encontrar un punto de maximo


global entonces denotaremos max {f (x) : x X}.
Observando el hecho que encontrar un punto de maximo de g es equivalente a encon-
trar un mnimo de f = g, ver Figura 2.3, podemos concluir que todo problema de
maximizacion puede ser transformado en uno de minimizacion. As, sin perdida de gen-
eralidad, podemos estudiar el problema

min{f (x) : x X} (2.2)

Definicion 2.2 Dos problemas de optimizacion (P1 ) : min{f (x) : x X} y (P2 ) :


min{g(x) : x Z} son equivalentes si tienen el mismo conjunto de soluciones, es decir,
x resuelve (P1 ) si, y solo si, x resuelve (P2 ).

Ejemplo 2.1 El problema min{x2 : x R} es equivalente a min{|x| : x R}.

13
Puntos de maximo Puntos de mnimo global
global global

Figura 2.2: Puntos de mnimo y maximo global

Figura 2.3: x es un punto de maximo de g y punto mnimo de f = g.

2.2 Clasificacion de Problemas de Optimizacion


Existen diversas maneras de clasicar los problemas de optimizacion, presentaremos
a seguir algunas de ellas.

a) Por sus restricciones, los problemas pueden ser restrictos o irrestrictos. Si X = M,


entonces el problema (2.2) es llamado problema de minimizacion sin restricciones
o problema irrestricto y escribiremos

min{f (x) : x M }.

Si X es un subconjunto propio de M entonces el problema (2.2) es llamado problema

14
de minimizacion con restricciones o restricto. Un ejemplo tpico de X es

X = {x M ; hi (x) = 0, gj (x) 0, para todo i = 1, . . . , m y para todo j = 1, . . . , p},

donde hi : M R y gj : M R son funciones dadas.


En este caso, el problema de minimizacion tambien sera denotado por:


min f (x)


hi (x) = 0, para i = 1, 2, ..., m



g (x) 0 para j = 1, 2, ..., p
j

o tambien,

min f (x)


h(x) = 0



g(x) 0
donde h : M Rm y g : M Rp son funciones tales que h(x) = (h1 (x), ..., hm (x))
y g(x) = (g1 (x), ..., gp (x)).

b) Por la funcion objetivo y las propiedades de sus restricciones, los problemas pueden
ser lineales o no lineales.
Si f en (2.2) es lineal y X es un conjunto denido por funciones anes ( una funcion
f : Rn R es afn si para todo x, y Rn y R se tiene que f (x + (1 )y) =
f (x)+(1)f (y),), entonces el problema se llama problema de optimizacion lineal.
La forma padron de un problema de optimizacion lineal en el espacio euclidiano es:

min{cT x : Ax = b, x 0},

donde c Rn , b Rm , A Rmn y x Rn es la incognita. Observe que en este


problema la funcion f denida por f (x) = cT x es lineal y las funciones h(x) = Axb
y g(x) = x son anes.
Si f no es lineal o alguna de las funciones que denen el conjunto X no es afn,
entonces el problema de optimizacion es no lineal.

c) Por la naturaleza del problema, los problemas pueden ser discretos, continuos o
mixtos.

15
Si X es un conjunto nito o enumerable entonces el problema es denominado
problema de optimizacion discreta y las variables que denen las restricciones son
llamadas variables discretas. Un ejemplo particular es el siguiente problema:

min{x2 : xi {0, 1}, para cada i = 1, ..., n};

aqu las variables xi son llamadas variables discretas. Mas ejemplos de estos prob-
lemas se pueden encontrar dentro de la optimizacion entera y optimizacion combi-
natoria.

Si X es innito no enumerable entonces el problema es llamado problema de op-


timizacion continua y sus variables son llamadas variables continuas. Un ejemplo
particular es:
min{x2 : xi R, para cada i = 1, ..., n};

en este caso las variables xi son llamadas variables continuas.

Si el problema de optimizacion tiene variables discretas y continuas, entonces el


problema es llamado problema de optimizacion con variables mixtas. Un ejemplo
es
min{f (x1 , x2 , t) = x21 + x22 t2 : xi {0, 1}, para i = 1, 2 y t R+ }.

2.3 Ejemplos de Problemas de Optimizacion


Con el objetivo de mostrar la versatilidad del modelo de optimizacion presentamos a
seguir algunos ejemplos de problemas de optimizacion como aplicacion en diversas areas
de las ciencias e ingenieras. Obviamente, estos problemas aun no seran estudiados pero
lo retomaremos en los siguientes captulos.

Ejemplo 2.2 (Funcionamiento de un sistema de produccion de energa electrica).


Sea un sistema de produccion de energa electrica aislada compuesta de n centrales
termicas unidas por lneas de transmision al nodo de concentracion de la carga, (ver
Figura 2.4). El problema consiste en distribuir la energa producida por las centrales de

16
Central Central
n nodo 1
Energa Energa
xn x1

Central Central
n1 2
Energa Energa
xn1 x2

Central Central
n2 3
Energa Energa
xn2 x3

Figura 2.4: Central de produccion de energa electrica.

tal manera que se minimice el costo de combustible utilizado para producir la cantidad de
energa requerida.
Designamos por: xi =la energa producida por la i-esima central, i = 1, 2, . . . , n. Para
cada i = 1, . . . , n, la energa xi varia entre i y i , lmites definidos para las cargas, esto
es,
i xi i .

Ademas, es necesario respetar las condiciones de la balanza energetica: la cantidad


total de energa producida debe ser igual a la cantidad total de energa consumida Ec ,
adicionando las perdidas totales Ep en las lneas de transmision. As tenemos
n
X
xi = Ec + Ep .
i=1

La cantidad de conbustible necesaria para producir la energa xi es dada por una


funcion Ti (xi ) definida sobre el segmento [i , i ]. Por lo tanto, obtenemos el siguiente
problema de optimizacion:
( n n
)
X X
min Ti (xi ) : xi = Ec + Ep , i xi i , i = 1, . . . , n .
i=1 i=1

El modelo anterior es el mas simple problema de minimizacion de un sistema de


produccion de energa electrica. En una situacion mas real, la carga es concentrada en
varios puntos, as, es necesario considerar m nodos.

17
Por otro lado, las perdidas de un sistema no son constantes, ellos son definidos por
la intensidad de energa transmitida y los parametros de las lneas de transmision. De
esta manera, para una aproximacion mas real del modelo se debe examinar el problema
cuando Ep es una funcion bilineal de xij , donde xij denota la energa transmitida de la
iesima central al jesimo nodo. As, obtenemos el siguiente problema de optimizacion:


X n
m X m X
X n
min Tij (xij ) : xij = Ec + Ep (xij ), ij xij ij , i = 1, . . . , n, j = 1, . . . , m .

j=1 i=1 j=1 i=1

Ejemplo 2.3 (Problema de ajuste de curvas). Una clase de problemas no lineales


es la eleccion de la mejor curva dentro de una famlia de curvas para ajustar los datos
provistos por algun experimento o algun muestreo.
Supongamos que m datos de espectro solar {y1 , y2 , . . . , ym } fueron tomados por un
satelite en tiempos {t1 , t2 , . . . , tm } respectivamente y la teora existente afirma que cualquiera
de los m puntos (t1 , y1 ), (t2 , y2 ), . . . , (tm , ym ), pueden ser ajustados por la curva:
(t+x3 )2

g(x1 , x2 , x3 , x4 , t) = x1 + x2 e x4
. (2.3)

En la practica, sin embargo, existio errores experimentales como muestra la figura 2.5.
Para establecer conclusiones sobre los datos deseamos encontrar la curva mas cercana a
y

t1 t2 t3 ... ... tm1 tm x

Figura 2.5: Datos experimentales tomandos por un satelite

los m puntos. Desde que la ecuacion general de la curva en estudio es dado por (2.3),

18
esto significa que debemos escoger x1 , x2 , x3 , x4 que minimice la discrepancia (error) entre
los datos y la curva. La discrepancia es dada por:

ri (x) := g(x1 , x2 , x3 , x4 , ti ) yi , i = 1, . . . , m.

La medida mas utilizada es la suma de cuadrados, obteniendo el siguiente problema


de optimizacion sin restricciones
( m m  2 )
X X
(ti +x3 )2
min f (x) = ri (x)2 = x1 + x2 e x4
yi : (x1 , x2 , x3 , x4 ) R4 .
i=1 i=1

Observemos que tambien pueden ser utilizados otros tipos de medida para la funcion
objetivo, por ejemplo
m
X
f1 (x) = |ri (x)| ,
j=1

f (x) = max |ri (x)| .


i=1,...,m

La ventaja de la funcion f es que esta es continuamente diferenciable y eso favorece en


la eleccion de un metodo que use derivadas parciales para resolver el problema planteado.

Ejemplo 2.4 (Problema de seleccion de carteras de inversion). El problema


de la seleccion de cartera consiste en la inversion apropiada de un capital en un grupo
de acciones de tal manera que al mismo tiempo se tenga un gran rendimiento y un
riesgo pequeno de perdida de la inversion. Debido que en la realidad estas dos metas son
contradictorias es necesario estudiar un metodo para satisfazer estos dos requerimientos.
Un rendimiento puede ser interpretado de la siguiente manera: supongamos que una
inversion de di soles se aplica al activo i y suponga que durante algun periodo de tiempo
especificado estos di soles se convierten en 1.5di . En este caso diremos que el rendimiento
durante ese periodo de tiempo es de

1.5di di
= 0.5.
di

El concepto de riesgo es medido usualmente mediante la varianza del rendimiento de


la cartera. Ya que el inversionista busca un equilibrio entre alto rendimiento y bajo riesgo,
una forma de abarcar el problema es minimizar el riesgo sujeto a un limite inferior del

19
rendimiento esperado. Tambien puede existir algunas restricciones sobre la parte de la
cartera dedicada a acciones individuales en particular.
Para obtener el modelo de optimizacion definimos:
xi : la proporcion de la cartera invertida en la accion i.
i2 : la varianza de los rendimientos anuales de la accion i.
Ri : rendimiento anual esperado de la accion i.
G : lmite inferior sobre el rendimiento anual esperado de la inversion total.
Si : lmite superior sobre la inversion en la accivn i.
ij : la covarianza de los rendimientos anuales de las acciones i y j (es un numero que
describe el grado hasta donde los rendimientos de las dos acciones i y j ascienden o
descienden juntos).
Pn
El rendimiento anual de la cartera se define por i=1 xi Ri . La varianza del rendimiento
de la cartera (o riesgo de la cartera) que es obtenido por procedimientos estadsticos es
definido por:
n
X n1 X
X n
f (x) = f (x1 , . . . , xn ) = x2i i2 + 2 xi xj ij .
i=1 i=1 j=i+1

Luego obtenemos el problema de optimizacion


( n n
)
X X
min f (x) : xi = 1, G xi Ri , 0 xi Si , i = 1, . . . , n .
i=1 i=1

Por ejemplo si n = 3 y si tenemos los siguientes valores:

12 = 0.09 22 = 0.06 32 = 0.04 12 = 0.02 13 = 0.01


23 = 0.02 G = 0.03 S1 = 0.75 S2 = 0.9 S3 = 0.8
R1 = 0.02 R2 = 0.03 R3 = 0.07;

entonces, el problema de optimizacion es:



min f (x) = 0.09x21 + 0.06x22 + 0.04x23 + 2(0.02x1 x2 + 0.01x1 x3 + 0.02x2 x3 )





0.02x1 + 0.03x2 + 0.07x3 = 1


0 x1 0.75





0 x2 0.9



0 x3 0.8

20
Para resolver el modelo general los valores de i2 , ij , y Ri deben ser conocidos (como
mostramos en el ejemplo particular para n = 3), esto es, si en los datos del problema
estos no son dados entonces deben ser estimados con datos historicos.
Por ejemplo, si se cuenta con m periodos de datos (en anos) el activo i tendra
un rendimiento historico real Rit relacionado con cada periodo t {1, 2, . . . , m}. El
rendimiento esperado del activo i se estima con:
m
1 X t
Ri = R.
m t=1 i

Los rendimientos historicos periodicos tambien son utiles patra estimar las varianzas
y las covarianzas, dadas por:
m m
1 X t 1 X t
i2 = (Ri Ri ) y ij = (R Ri )(Rjt Rj ).
m t=1 m t=1 i

Ejemplo 2.5 (Problema de la mochila). Un viajante debe visitar n ciudades dis-


tintas, comenzando y terminando su viaje en la misma ciudad. Cada ciudad debe ser
visitada una sola vez. El problema es encontrar un viaje de longitud mnima. Sea
C = {1, 2, . . . , n} el conjunto de ciudades. Supongamos que el viaje comienza y ter-
mina en la ciudad 1. Puede ser probado que todas las posibilidades para encontrar el
viaje optimo es dado por (n 1)!, luego resulta imposible verificar todas las opciones para
resolver el problema cuando n es grande. A seguir modelamos el problema de la mochila
como un problema de optimizacion. Definamos

dij : la distancia entre las ciudades i y j, (dii = 0)




1, si el viajante va de i a j
xij =

0, caso contrario.

Cada recorrido implica partir de la ciudad i, esto es,


n
X
xij = 1, para todo i = 1, 2, . . . , n.
j=1

Cada recorrido implica llegar a la ciudad j


n
X
xij = 1, para todo j = 1, 2, . . . , n.
i=1

21
La longitud del viaje es:
n X
X n
f (x11 , x12 , . . . , xij , . . . , xnn ) = dij xij .
j=1 i=1

As, el problema de optimizacion es:


( n n n n
)
XX X X
min dij xij : xij = 1, xij = 1, xij {0, 1} .
j=1 i=1 j=1 i=1

Por ejemplo, si n = 3, entonces usando el hecho que xii = 0 (no hay viaje de la ciudad i
a la ciudad i) y dii = 0 tenemos que el problema de optimizacion es:


min f (x) = d12 x12 + d13 x13 + d21 x21 + d23 x23 + d31 x31 + d32 x32





x12 + x13 = 1





x21 + x23 = 1




x +x =1
31 32

x21 + x31 = 1





x12 + x32 = 1





x13 + x23 = 1




x12 , x13 , x21 , x23 , x31 , x32 {0, 1}

n=3
2
d 21
d2
d 12 3

d3
2

d13
1 3
d31

Figura 2.6: (3-1)!= 2 viajes posibles: (d12 , d23 , d31 ) y (d13 , d32 , d21 )

22
n=4

2 d32

d 21
d2
d 12 d42 3

d13
1 d31 3

4
d3
d41

3
d4
d4
1
4

Figura 2.7: (4-1)!=6 viajes posibles

Ejemplo 2.6 (Problema de la dieta). Supongamos que un centro (Colegio, Univer-


sidad, Hospital, etc.) se plantea el problema de disenar una dieta a partir de n alimentos
que cuestan ci por unidad, i = 1, 2, . . . , n, de tal manera que resulte lo mas barato posible
y se satisfagan las necesidades nutricionales que aconsejan las organizaciones de salud.
Se precisa ingerir de m nutrientes basicos al menos bj unidades diarias de cada uno de
ellos. Por otra parte, se conoce que entre los alimentos disponibles, una unidad de ali-
mento i contiene aij unidades del nutriente j, donde i = 1, 2, . . . , n y j = 1, 2, . . . , m. El
problema es determinar las cantidades apropiadas de cada alimento que deben incluirse
en la dieta, para que su costo sea mnimo. Sean

xi = la cantidad a ingerir del alimento i-esimo,

ci = el precio (unitario) del alimento i-esimo.

Entonces el problema es:


( n n
)
X X
min c i xi : aij bj (j = 1, 2, . . . , m), xi 0 .
i=1 i=1

23
Ejemplo 2.7 (Problema de transporte). Considere el problema de transportar una
mercancia de m puntos de origen a n puntos de destino. Se considera que en los pun-
tos de origen 1, 2, . . . , m se tienen respectivamente las cantidades a1 , a2 , . . . , am de un
determinado bien. Sean los puntos de destino 1, 2, . . . , n a los que deben de llegar las
cantidades b1 , b2 , . . . , bn , ver Figura 2.8.
Supondremos que existe un equilibrio entre la oferta y la demanda, esto es,

DESTINOS
1 2 ... 3
O 1 x11 x12 ... x1n a1
R 2
x21 x22 ... x2n a2

... . .. .. ..
I
G .. . . .
E
N
E
S am
m xm1 xm2 xmn
b1 b2 ... bn

Figura 2.8: Problema de Transporte.

m
X n
X
ai = bj .
i=1 j=1

Sean

cij : el costo unitario de enviar una unidad del origen i al destino j.

xij : cantidad a enviar del origen i al destino j.

Se deben satisfacer las siguientes condiciones:

i. Para cada origen i se debe cumplir que la suma de las cantidades desde i hacia los
diferentes destinos deben coincidir con la cantidad ai que hay en el origen, esto es,
Xn
xij = ai .
j=1

24
ii. Para cada destino j, se debe verificar que la suma de las cantidades que llegan a j
es igual a la cantidad bj , esto es,
m
X
xij = bj ,
i=1

Luego el problema a resolver es:


( n n n m
)
XX X X
min cij xij : xij = ai , xij = bj , xij 0 .
j=1 i=1 j=1 i=1

Ejemplo 2.8 (Problema geometrico de Euclides). Este problema aparecio en el


primer libro de geometra de Euclides (300 A.C). Dado un triangulo ABC, se desea
inscribir un paralelogramo ADEF de tal manera que EF//AC y DE//AB tal que el
area de este paralelogramo sea maxima, vea la figura 2.9
C

D E h(altura)

A x bx B
F
b

Figura 2.9: Primer problema de optimizacion encontrado en el libro de Euclides

Puede probarse que el problema consiste en resolver


 
xh(b x)
max :0<x<b ,
b
donde h es la altura del triangulo, b es la medida de la base del triangulo y x es la base
del parelogramo.

Ejemplo 2.9 ( Calculo de variaciones). Los problemas variacionales son una famlia
particular de problemas de optimizacion. Ellos se caracterizan por estar definidos en

25
espacios de funciones de dimension infinita y por el hecho que la funcion objetivo es un
operador integral. Los problemas variacionales clasicos pueden expresarse como
 Z b 

min I(u) := L(t, u(t), u (t))dt ; u X ,
a

donde L : [a, b] R R R es una funcion dada, u : [a, b] R es la funcion incognita


y X es un subconjunto del espacio C 1 [a, b].
Las restricciones usuales son:

1. Condicion de contorno: u(a) = y u(b) = .

2. Restriccion Lagrangeana: g(t, u(t), u (t)) = 0, con g : [a, b] R R R una


funcion dada.
Rb
3. Restriccion isoperimetrica: a
g(t, u(t), u (t))dt = c, donde g : [a, b] R R R
es una funcion dada.

A modo de ilustracion mostraremos el problema clasico de la Braquistocrona (del griego


brachystos-mnimo, chronos-tiempo).
En 1696 Johann Bernoulli propuso el siguiente problema: Sean P0 y P1 dos puntos dados
sobre un plano vertical. Se debe encontrar una curva uniendo esos puntos de tal forma
que un punto partiendo de P0 que la recorra, sobre la influencia solamente de su propio
peso, alcance P1 en el menor tiempo posible. Considere ademas la velocidad inicial v0
dada. Tal problema fue formulado inicialmente por Galileo en 1638, quien crea que la
solucion del problema era un arco de circunferencia. Bernoulli dio al problema el nombre
de Braquistocrona y usando el principio de refraccion de Fermat obtuvo la siguiente
formulacion:
(Z  1/2 )
b
1 + u (t)2
min dt ; u C 1 [a, b], u(a) = , u(b) = ,
a 2gu(t) + c

donde g es la constante gravitacional y c es la energa total del cuerpo (cinetica+potencial)


en el instante inicial.

Ejemplo 2.10 (Problema de autovalores). Dado una matriz A Rnn . Decimos


que es un autovalor de A si existe un vector no nulo x Rn , llamado autovector de

26
A, tal que Ax = x. El problema de hallar los autovalores y autovectores de A pude ser
formulado como el problema de optimizacion:

min {f (x, ) = kAx xk : (x, ) Rn R, x 6= 0} ,

donde k.k es la norma euclidiana. Observemos que si el problema tiene solucion y su


valor optimo es cero entonces resolvemos el problema de hallar un autovalor de A y su
correspondiente autovector.
Para obtener el menor autovalor de A, bajo algunas condiciones sobre la matriz (ver
Ejercicio 6.3), es suficiente resolver el modelo:
 
1 T
min x Ax : kxk = 1 .
2

De forma analoga para encontrar el mayor autovalor de A es suficiente, bajo algunas


condiciones sobre la matriz, resolver:
 
1 T
max x Ax : kxk = 1 .
2

Ejemplo 2.11 (Solucion de sistemas de ecuaciones lineales). Dados A Rnn


una matriz simetrica definida positiva y b Rn . El problema de resolver un sistema de
ecuaciones lineales consiste en encontrar un punto x Rn tal que: Ax = b. Resolver el
problema dado es equivalente a resolver el problema de optimizacion (ver Ejercicio 6.1):
 
1 T T n
min (x Ax) b x : x R .
2

Este abordaje de optimizacion es utilizado principalmente cuando la matriz A es esparsa


y de dimension grande.

Ejemplo 2.12 (Teora economica de la decision). Los modelos matematicos en la


teora economica de la decision consisten en la eleccion optima de un agente economico
(una empresa, un gobierno, un estado, etc.) dentro de todas las posibles alternativas a
escoger. Hay tres elementos que caracterizan el problema de decision del agente:

El conjunto de eleccion, que nos dice cual es el universo de alternativas.

27
El criterio de valorizacion, que estudia la manera en la que el agente evalua las
diferentes alternativas que se le ofrecen. Este criterio se define mediante una
relacion binaria reflexiva y transitiva -, interpretada economicamente como es
al menos tan preferible que refleja sus preferencias sobre el conjunto de eleccion.
En economa esta relacion es llamada preferencia.

Las restricciones, que delimitan el conjunto de oportunidades sobre el cual el agente


puede efectivamente elegir.

Dado el conjunto de eleccion X, decimos que : X R es una funcion de utilidad para


- si para todo x, y X

y - x, si y solo si (y) (x) .

Las preferencias - bajo ciertas condiciones sobre X pueden representarse por una funcion
de utilidad . As, elegir la mejor alternativa se convierte en encontrar una alternativa
que maximice la funcion . Por lo tanto concluimos que resolver el problema de decision
del agente es equivalente a resolver el problema de maximizacion:

max{ (x) : x X}.

Un tipo particular de funcion de utilidad es la funcion cuasi-concava, esto es, satisface:

(x + (1 )y) min{(x), (y)}, para todo x, y X; y [0, 1].

Se puede probar que esta funcion esta ntimamente relacionada a la hipotesis de convexi-
dad de la preferencia. Recordemos que - es convexa si dados x, y X con x - y y x - z
en X y 0 1 entonces
x - y + (1 ) z.

En terminos economicos esta definicion se interpreta como la tendencia natural del


agente a diversificar su eleccion entre todas las mejores posibles, esto es, si z e y son dos
alternativas al menos tan preferibles que x, entonces es natural pensar que la combinacion
y + (1 ) z, para todo [0, 1], continua siendo al menos tan preferible que x.
La condicion de convexidad de la preferencia - y algunas otras condiciones apropiadas
permiten garantizar la existencia de una funcion de utlidad cuasi-concava y por lo tanto

28
tener un problema de optimizacion cuasi-concavo (problemas donde la funcion objetivo
es cuasi-concava y las funciones que definen las restricciones son cuasi-concavas).

Ejemplo 2.13 (El problema de la produccion). Considere que una empresa puede
producir n productos en cantidades x1 , x2 , . . . , xn , siendo sus costos de produccion (por
unidad) respectivamente igual a p1 , p2 , . . . ., pn , donde pi > 0, para todo i = 1, 2, . . . , n.
Si se tiene un capital de b soles para realizar la produccion y una funcion continua f
que representa la utilidad de produccion de los n productos, el problema de conocer una
utilidad mnima o maxima es expresado por

opt{f (x) : pT x b, xi 0}.

Ejemplo 2.14 (Teora de la decision proximal, ver [18]). Sea el estado X = Rn+ ,
interpretado economicamente como el conjunto de alternativas, ejecuciones, acciones,
decisiones, etc. El termino agente sera referido a una persona, empresa o institucion
encargada de la toma de decisiones. Un agente tiene una funcion objetivo g : X R,
que representa la ganancia por unidad de tiempo (utilidad, pago, ingreso, beneficio).
Consideremos x X algun estado inicial dado y supongamos que el supremo de este
objetivo es finito, esto es, g = sup {g(x), x X} < +. Supongamos que el agente
comienza de algun estado inicial x0 = x X, conoce el supremo de g y el valor inical
g(x) = g(x0 ). As, el conoce cuan lejos esta del supremo. Esta diferencia o llamada
tambien salto es definida por
n(x) = g g(x).

La manera mas natural de que el agente alcance su meta es encontrar el objetivo de


manera progresiva dividiendolo por periodos que detallamos a continuacion.

Fase Exploratoria El agente explora en un entorno de x, denotado por:

E(x, r(x)) X,

durante algun tiempo > 0, para descubrir y E(x, r(x)) X.

Fase Desplazamiento El agente se desplaza de x X al nuevo estado y X en un


tiempo > 0.

29
Fase Explotacion El agente debe beneficiarse al obtener un nuevo estado mejorado. El
escoge explotar su beneficio en un tiempo > 0.

Es as que del estado x X descubrimos un nuevo estado y X mejorado, tal que


g(y) > g(x), de modo que n(y) < n(x).
En cada periodo k IN , se considera la funcion ganancia proximal

Pk (xk , y) = g(y) k C(xk , y, )


1
donde xk , y E(xk , r(xk )), k = k
, C(xk , y, ) es el costo de desplazamiento satisfa-
ciendo C(xk , xk , 0) = 0.
Asumiremos por simplicidad que E(xk , r(xk )) = X, k IN . As, el agente tratara
de maximizar su ganancia proximal resolviendo para cada k

xk+1 argmax{Pk (xk , y) : y X},

el cual es equivalente a: xk+1 argmin{n(y) + k C(xk , y, ); y X}.



El objetivo de este modelo es generar una sucesion xk tal que en el lmite se obtenga
la meta propuesta.

Ejemplo 2.15 (Suma de los mayores autovalores). Recientemente Overton y Wom-


ersley [14] (Teorema 3.4, pagina 329), presentaron la siguiente caracterizacion para la
suma de los primeros k mayores autovalores de una matriz simetrica

1 (A) + . . . + k (A) = max{A X : TrX = k, 0  X  I},

donde j (A), j = 1, . . . , k, denota el j-esimo mayor autovalor de A y A X denota la


traza de AX. El problema dado pertenece a una clase especial de problemas de opti-
mizacion llamados problemas de optimizacion semidefinida (las variables son matrices
semidefinidas positivas) y cuyo formato general para el caso acotado es

min {f (X) : AX = b, 0  X  I} (2.4)


XS n

donde f : S n R es una funcion real, S n es el espacio vectorial de las matrices


simetricas, I S n es la matriz identidad, el operador A : S n Rm es definido por
AX := (Ai X)m m n n
i=1 R , con Ai S ; X S ; X  0 significa que X es semidefinida

positiva, y X  I significa que I X  0.

30
Ejemplo 2.16 (Mnimo de la suma de los mayores autovalores, ver [15]). Con-
sideremos el problema de minimizar la suma de los primeros k mayores autovalores de
una matriz simetrica:

min{1 (A(y)) + . . . + k (A(y)) : y Rm }


m
X
donde A(y) = A0 + yi Ai . Puede ser probado (ver Alizadeh [1], Teorema 4.3), que el
i=1
dual de este problema es

max{A0 X : TrX = k, Aj X = 0 para j = 1, . . . , m, 0  X  I}. (2.5)

Note que podemos expresar (2.5) como

max{A0 X ; AX = b, 0  X  I},

donde AX = (I X, A1 X, . . . , Am X)T y b = (k, 0, 0, . . . 0) Rm+1 . Por tanto, este


problema tiene la forma de (2.4).

Ejemplo 2.17 (Minimizacion de la suma ponderada de los autovalores). Con-


sidere el siguiente problema:

min{m1 1 (A) + . . . + mk k (A)}, donde m1 . . . mk > 0. (2.6)

Donath y Hoffman en [6] reformularon esta suma como sigue:

m1 1 + . . . + mk k = (m1 m2 )1 + (m2 m3 )(1 + 2 ) + . . . + (mk1 mk )(1 + . . . + k ),

donde por simplicidad usamos la notacion i = i (A). Para cada suma parcial de auto-
valores del lado derecho de la igualdad podemos usar la formulacion del ejemplo anterior,
obteniendo

min (m1 m2 )X1 A + (m2 m3 )X2 A + . . . + mk Xk A


s.a: Tr(Xi ) = i para i = 1, . . . , k
0  Xi  I.

Observe que este problema puede ser expresado en la forma (2.4), donde f (X) = CX, con
X = diag(X1 , X2 , . . . , Xn ), C = diag((m1 m2 )A1 , . . . , , (mk1 mk )Ak ), A(X) = I X,
y b = (1, 2, . . . , k)T . As, (2.6) es un caso particular de (2.4).

31
Ejemplo 2.18 (El problema de particion de un grafo). Un importante caso del
problema de particion de un grafo, ver [1], es formulado como:

min{C X; Xii = k/n, 0  X  I}. (2.7)

Podemos expresar (2.7) como (2.4) usando f (X) = CX, A1 = diag(1, 0, . . . , 0), A2 =
diag(0, 1, 0, . . . , 0), . . . , An = (0, 0, . . . , 0, 1) y b = (k/n, k/n, . . . , k/n) Rn .

32
Captulo 3

Existencia de Puntos de Mnimo


Global

El espacio euclidiano donde estudiaremos las condiciones de existencia y caracteri-


zacion de las soluciones optimas de problemas de optimizacion es un espacio bien natural
para modelar problemas de optimizacion. Incluso si el problema se encuentra en uno de
dimension innita los diversos metodos de discretizacion permiten transformar el prob-
lema original en un problema denido en un espacio euclidiano.

3.1 Optimos Globales y Locales


En Optimizacion existen dos tipos de optimos: locales y globales, que pasamos a denir.

Definicion 3.1 (Mnimo local y mnimo global) Sea f : X Rn R una funcion


definida sobre X. Un punto x X es llamado un punto de mnimo local de f sobre X
si existe > 0 tal que f (x) f (x), para todo x X B(x, ).
El punto x X es llamado punto de mnimo global de f sobre X si f (x) f (x), para todo x
X.

Observacion 3.1 La relacion inmediata de estas dos definiciones es que todo mnimo
global es un mnimo local, pero la viceversa, evidentemente, no es verdadera.

Analogamente, tenemos la siguiente denicion.

33
Definicion 3.2 (Maximo local y maximo global) Sea f : X Rn R una funcion
definida sobre X. Un punto x X es llamado un punto de maximo local de f sobre X
si existe > 0 tal que f (x) f (x), para todo x X B(x, ).
El punto x X es llamado punto de maximo global de f sobre X si f (x) f (x), para
todo x X.

Figura 3.1: Optimos locales y globales.

Observacion 3.2 Desde que todo problema de maximizacion puede ser transformado
en uno de minimizacion, a partir de ahora consideraremos para el analisis solamente
problemas de minimizacion.

3.2 Resultados de Existencia


Considere el problema
min{f (x) : x X},

donde f : X Rn R es una funcion denida sobre X. Una pregunta natural es


saber si existe un punto que resuelva el problema planteado. Veremos que para ello sera
necesario imponer algunas condiciones sobre f y X.

34
Una condicion necesaria para probar la existencia de un punto de mnimo global es que
la funcion f sea acotada inferiormente, esto es, inf{f (x) : x X} > . Observemos
que esta condicion no es suciente, considere por ejemplo la funcion exponecial f (x) = ex ,
donde x R. Esta funcion es acotada inferiormente por cero pero no existe un punto de
mnimo global como se muestra en la gura 3.2.

ex

Figura 3.2: La funcion exponencial tiene nmo pero no mnimo.

Un resultado clasico para garantizar la existencia de puntos de mnimo global es el


conocido Teorema de Bolzano-Weierstrass, aplicado a funciones continuas sobre conjun-
tos compactos. En este libro consideraremos una hipotesis mas debil, la condicion de
semicontinuidad inferior, que pasamos a denir.

Definicion 3.3 Dada una funcion f : X Rn R. f es semicontinua inferior en


x X, si para toda sucesion {xk } de X convergente en x se tiene que

lim inf f (xk ) f (x),


k+

donde lim inf k+ f (xk ) := supnN inf kn f (xk ). Si f es semicontinua inferior para todo
x X, entonces decimos que f es semicontinua inferior en X.

Resulta claro de la denicion que toda funcion continua es semicontinua inferior pero el
recproco no es verdad como muestra el siguiente ejemplo.

35
Ejemplo 3.1 La funcion f : [1, 2] R tal que


(x + 1)2 , si x (1, 2],
f (x) =

1, si x = 1,

es semicontinua inferior en x = 1 pero no es continua en x = 1 (ver figura 3.3). En


efecto, es obvio que la funcion no es continua en 1, probemos que es semicontinua inferior
en este punto. Sea {xk } [1, 2] tal que xk 1, como la funcion es creciente para n IN
tenemos que inf kn f (xk ) = 4, por lo tanto supnN inf kn f (xk ) = 4 > 1 = f (1).

Figura 3.3: La funcion f es semicontinua inferior pero no es una funcion continua.

Teorema 3.1 Dada una funcion f : X Rn R. Si f es semicontinua inferior en un


conjunto no vacio y compacto X entonces existe un punto de mnimo global.

Demostracion. Probemos inicialmente que f es acotada inferiormente. Por con-


tradiccion, supongamos que existe una sucesion {xk } X tal que

lim f (xk ) = . (3.1)


k+

Como {xk } X y X es un conjunto compacto, entonces esta sucesion contiene una


subsucesion {xkj } convergente en X, esto es, existe x X tal que limj+ xkj = x.

36
Debido que f es semicontinua inferior tenemos:

f (x) lim inf f (xkj ).


j+

Ya que {f (xkj )} es una subsucesion de {f (xk )} y por (3.1) obtenemos que f (x) ,
lo cual es una contradiccion. Por lo tanto, f es limitada inferiormente en X. As, existe
R tal que
= inf{f (x) : x X},

esto implica que existe una sucesion {xl } X tal que liml+ f (xl ) = . Por la
compacidad de X existe una subsucesion {xlj } y x X tal que limj+ xlj = x. Debido
a la semicontinuidad inferior

f (x) lim f (xlj ) = .


j+

De esta ultima desigualdad obtenemos que f (x) = y por lo tanto f (x) f (x), para
todo x X, esto es, x es un mnimo global de f en X.

Ejemplo 3.2 Considere la funcion f definida en el Ejemplo 3.1, como esta es semicon-
tinua inferior y [1, 2] es compacto, entonces usando el teorema anterior concluimos que
para f existe un mnimo global en [1, 2].

Corolario 3.1 Dada una funcion f : X Rn R. Si X es no vacio y compacto y f


es continua en X, entonces existe un punto de mnimo global.

Prueba. Si f es continua entonces es semicontinua inferior y aplicando el teorema


precedente obtenemos el resultado.

Observacion 3.3 La condicion de compacidad de X es muy fuerte para garantizar la


existencia de un mnimo global. En efecto, considere el problema

min{x2 : x R} (3.2)

Aqu X = R no es compacto pero existe el mnimo global.

Para debilitar la hipotesis de compacidad de X, denamos, para R, el conjunto


Lf () := {x X : f (x) } , llamado conjunto de nivel inferior de f (Ver gura 3.4).

37
Figura 3.4: Graco del conjunto de nivel inferior.

Corolario 3.2 Sea una funcion f : X Rn R. Si Lf () es no vacio y compacto para


algun R y f es semicontinua inferior en Lf (), entonces existe un punto de mnimo
global de f en X.

Prueba. Consideremos el siguiente problema de optimizacion

min{f (x) : x Lf ()}.

Como f es semicontinua inferior en Lf () y Lf () es no vacio y compacto, entonces


por el Teorema 3.1, existe x Lf () tal que f (x) f (x), para todo x Lf ().
Tomemos x como candidato para mnimo global de f sobre X. Sea x X (arbitrario) y
consideremos dos casos:

i). Si x Lf (), por el resultado anterior tenemos f (x) f (x).

ii). Si x
/ Lf (), entonces f (x) > f (x). As, f (x) < f (x).

De ambos casos obtenemos que x es un mnimo global de f sobre X.

Corolario 3.3 Sea una funcion f : X Rn R. Si X es cerrado y Lf () es no vacio


y acotado para algun R, con f semicontinua inferior en Lf (), entonces existe un
punto de mnimo global de f en X.

Prueba. Similar al corolario anterior.

38
Observacion 3.4 En el corolario anterior la condicion de cerradura de X es fundamen-
tal. En efecto, considere por ejemplo X = (1, 1] (conjunto no cerrado) y f (x) = (x+1)2 .

Aqu para todo R++ se tiene que Lf () = (1, 1] es no vacio, acotado y f es
continua pero no existe el mnimo global de f en X.

Para debilitar mas aun las condiciones de existencia de mnimos globales deniniremos
el concepto de funcion coerciva.

Definicion 3.4 Decimos que una sucesion {xk } en X es crtica (en relacion al conjunto
X) si: limk+ kxk k = + limk+ xk = x X\X, donde la notacion x X\X
significa que x X y x
/ X.
La funcion f : X Rn R es llamada coerciva en X si para toda sucesion crtica
{xk } se tiene:
lim sup f (xk ) = +,
k+

donde lim supk+ f (xk ) := inf nN supkn f (xk ).

Ejemplo 3.3 Dado x0 Rn fijo, f : Rn R tal que f (x) = kx x0 k, es una funcion


coerciva en Rn .

Corolario 3.4 Dada una funcion f : X Rn R. Si f es coerciva y semicontinua


inferior en un conjunto no vacio X, entonces existe un punto de mnimo global de f en
X.

Prueba. Como X 6= , existe R tal que Lf () 6= (X 6= , entonces existe x X,


as podemos tomar = f (x)). Probaremos que Lf () es acotado. Por contradiccion,
supongamos que Lf () no es acotado, entonces existe una sucesion {xk } Lf () tal que

lim kxk k = +,
k+

luego {xk } es una sucesion crtica en Lf (). Como xk Lf (), se tiene f (xk ) ,
tomando lmite superior cuando k + y usando la coercividad de f tenemos que
lim supk+ f (xk ) = + , lo que es una contradiccion. Por lo tanto Lf () es
acotado.

39
Probaremos ahora que Lf () es cerrado. Sea {xk } Lf () una sucesion tal que xk x
y supongamos que x
/ Lf (), esto es, x
/ X o f (x) > .
/ X entonces {xk } es una sucesion crtica y por la coercividad de f se tiene que:
Si x
+ = lim supk+ f (xk ) , lo que es una contradiccion.
Si f (x) > , entonces por la semicontinuidad inferior se tiene que

< f (x) lim inf f (xk ) ,


k+

lo que es una contradiccion. Por lo tanto x Lf () y as Lf () es cerrado.


Finalmente, usando el Corolario 3.2 obtenemos el resultado deseado.

3.3 Ejemplos de Existencia de Puntos Optimos


Ejemplo 3.4 (Existencia de soluciones de problemas de maximizacion). Considere el
problema de maximizacion
max{f (x) : x X} (3.3)

donde f : X Rn R y X denota el conjunto de las restricciones. Decimos que f es


semicontinua superior (s.c.s.) en x X, si para toda sucesion {xk } X tal que xk x
se tiene que lim supk+ f (xk ) f (x).
Usando los resultados del Teorema 3.1 se puede probar facilmente que si f : X Rn R
es s.c.s en X, no vacio y compacto, entonces existe un maximo global.
Consideremos ahora el conjunto de nivel superior Uf () := {x X : f (x) }. Usando
el Corolario 3.2 se puede probar que si f es s.c.s en X y Uf () es no vacio y compacto,
para algun , entonces existe un punto de maximo global de f en X.
Ademas, usando el Corolario 3.4 se tiene que si f es coerciva y s.c.s en un conjunto
no vacio X, entonces existe un punto de maximo global de f en X.

Ejemplo 3.5 Dados p = (p1 , ..., pn ), con pi > 0, i = 1, ..., n, y b R++ . El problema de
la utilidad mnima y maxima del Ejemplo 2.13 consiste en resolver

opt{f (x) : pT x b, x 0}

40
donde recordamos que opt significa minimizar o maximizar f y x 0 significa que cada
componente xi 0, para todo i = 1, ..., n.
Estudiaremos si el problema dado tiene mnimo y maximo global. Definamos

X = x Rn : pT x b, x 0 .

Afirmamos que X es compacto. En efecto, dado x X, entonces

p1 x1 + p2 x2 + .... + pn xn b.

b
Como los xi 0 y pi > 0, entonces xi pi
, para todo i = 1, ..., n.
Definamos M = min{pi , i = 1, ..., n} > 0. Elevando al cuadrado la desigualdad anterior

nb2
y tomando sumatoria tenemos: kxk2 M2
, esto es, kxk nb
M
.
Si f es semicontinua inferior, entonces el problema tiene un mnimo global y si f es semi-
continua superior el problema tiene maximo global. Ademas, si f es continua entonces
concluimos que el problema tiene mnimo y maximo global.

Ejemplo 3.6 El problema del funcionamiento de un sistema de produccion de energa


electrica consista en resolver:
( n n
)
X X
min Ti (xi ) : xi = Ec + Ep , i xi i .
i=1 i=1
 
P
n
Es facil probar que X = x : xi = Ec + Ep , i xi i es compacto y que f (x) =
i=1
P
n
Ti (xi ) es semicontinua inferior, entonces usando el Teorema 3.1, se tiene que el prob-
i=1
lema dado tiene un mnimo global.

Ejemplo 3.7 El problema de minimizacion cuadratica consiste en resolver


 
1 T T n
min f (x) = x Ax b x + c : x R ,
2

donde A Rnn , b Rn y c es un numero real. Si se cumple que A 0, entonces existe


un punto de mnimo global. En efecto, probaremos que la funcion f es coerciva en Rn .

Por contradiccion, supongamos que f no lo sea, entonces existe xk tal que
k
x +, pero f (xk ) M, para todo k IN ,

41
1 kT
para algun M R. Tenemos por lo tanto que: 2
x Axk bT xk + c M. Para k
suficientemente grande se tiene que xk 6= 0 y
T
!    k 
1 xk xk T x c M
A b 2 + 2 .
2 kx kk k
kx k kxk k kxk k kxk k2
Tomando una subsucesion si es necesario, existe z 6= 0 tal que
xk
zk = z.
kxk k
Pasando al lmite en la ultima desigualdad y multiplicando por 2 se obtiene z T Az 0, lo
que contradice la hipotesis de que A es definida positiva. As, f (x) = 21 xT Ax bT x + c es
continua y coerciva y por lo tanto, usando el Corolario 3.4, se tiene que existe un mnimo
global de f.

Ejemplo 3.8 (El problema de la mnima distancia). Dado un subconjunto X de Rn . El


problema de la mnima distancia de y a X es

min {f (x) = kx yk : x X} .

Si X es cerrado, entonces existe el mnimo global del problema. En efecto, analisemos


dos casos

i.) Si X es acotado, entonces X es compacto y como f es continua, el mnimo global


existe.

ii.) Si X no es acotado, entonces existe xk tal que limk+ kxk k = +. Como X

es cerrado, entonces xk es una sucesion crtica, y como f (xk ) +, se tiene
que f es coerciva en X.
Como f es continua y coerciva entonces existe el mnimo global.

3.4 Ejercicios Resueltos


Ejercicio 3.1 Probar que el problema

min f (x, y) = x2 + y + 1
x+y
sujeto a : x + y > 0

tiene una solucion global.

42
Solucion. Consideremos el conjunto X = {(x, y) R2 : x + y > 0}, que no es cerrado
ni acotado, la idea entonces es probar que el conjunto de nivel
 
2 1
Lf (c) = (x, y) X : x + y + c
x+y
es compacto para algun c R.
Probemos que Lf (c) es acotado.

Supongamos que Lf (c) no es acotado, entonces existe (xk , y k ) Lf (c) tal que k(xk , y k )k
+, entonces:
(xk )2 + (y k )2 + (3.4)

Desde que (xk , y k ) Lf (c), tenemos tambien que:
 2
k 1 1 1
x < (xk )2 + (y k ) < (xk )2 + (y k ) + k c (3.5)
2 4 x + yk

Entonces xk es acotado y por la ecuacion (3.4), (y k )2 +, cuando k +, luego
se tiene que y k +, cuando k + (si y k , de la condicion xk + y k > 0 y de
la limitacion de xk se tendra que 0).
De la ecuacion (3.5) se tiene que + c, lo que es una contradiccion, entonces Lf (c) es
acotado.
Probemos que Lf (c) es cerrado.
Por contradiccion, supongamos que Lf (c) no es cerrado, entonces existe

(xk , y k ) Lf (c) : (xk , y k ) (x, y) X\X.
1
As x + y = 0. Usando nuevamente la ecuacion (3.5) se tiene (xk )2 + (y k ) + xk +y k
c.
Tomando lmite tenemos que + c, lo que es una contradiccion.
De ambos resultados concluimos que Lf (c) es compacto y como f es continua en X
entonces existe un mnimo global.

Ejercicio 3.2 Diga si la funcion f (x1 , x2 ) = x41 + x42 32x22 es o no es coerciva.

Solucion. La funcion f puede ser expresada como f (x1 , x2 ) = x41 + (x22 16)2 162 .
Probaremos que f es coerciva. Para ello sea {(xk1 , xk2 )} una sucesion tal que

lim ||(xk1 , xk2 )|| = +.


k

De aqu se tiene que limk |xk1 | = + o limk |xk2 | = +.

43
Si limk |xk1 | = + y como f (x1 , x2 ) x41 162 , se tiene que limk f (xk1 , xk2 ) = +

Si limk |xk2 | = + y como f (x1 , x2 ) (x22 16)2 162 , se tiene que limk f (xk1 , xk2 ) =
+.

As queda probado que la funcion f es coerciva.

Ejercicio 3.3 Verificar si las siguientes funciones son o no coercivas:

a) f (x1 , x2 , x3 ) = x61 + 5x42 + x3 3x1 x2 x23 .


2 2 2
b) f (x1 , x2 , x3 ) = ex1 + ex2 + ex3 x100 100 100
1 x2 x3 .

Solucion.

a) La funcion f (x1 , x2 , x3 ) = x61 + 5x42 + x3 3x1 x2 x23 , no es coerciva.


En efecto, tome la sucesion {xk }, donde xk = (0, 0, k), entonces tenemos que
||xk || + pero limk+ f (xk ) = limk+ k = .
2 2 2
b) La funcion f (x1 , x2 , x3 ) = ex1 + ex2 + ex3 x100 100 100
1 x2 x3 es coerciva desde que
2
limr+ er r100 = +.

Ejercicio 3.4 Sean X1 y X2 subconjuntos de Rn tales que X2 X1 y f : Rn R una


funcion dada, pruebe que inf xX1 f (x) inf xX2 f (x). Ademas, si x es un minimizador
de f sobre X1 tal que x X2 entonces x es un minimizador de X2 .

Solucion. Si f no es acotada inferiormente en X1 entonces inf f (x) = inf f (x)


xX1 xX2
y el ejercicio queda probado.
Sea f acotada inferiormente en X1 entonces existe el nmo de f en concescuencia
inf f (x) := m f (x), para todo x X1 . Como X1 X2 , en particular tenemos:
xX1
m f (x), para todo x X2 , de aqu, f es acotada inferiormente en X2 y as existe el
nmo de f en X2 y m inf f (x). Sustituyendo el valor de m se obtiene
xX2

inf f (x) inf f (x).


xX1 xX2

Si x es minimizador de f sobre X1 , entonces f (x) f (x) para todo x X1 . En particular


se tiene f (x) f (x), para todo x X2 , luego, como por hipotesis x X2 se tiene que
este punto es un minimizador de f sobre X2 .

44
Ejercicio 3.5 Supongamos que el problema de minimizacion irrestricta de una funcion
continua en Rn tiene un mnimo global. Responder si eso implica que f tiene un mini-
mizador global sobre cualquier conjunto cerrado de Rn . Si no lo fuera dar un contraejem-
plo.

Solucion. No implica. En efecto, considere la funcion:



ex , x (, 0];
f (x) =
(1 x)2 , x (0, +)

f es continua en R y tiene un minimizador global en x = 1. Considerando el conjunto


cerrado (, 0] (pues el complemento (0, +) es abierto) se tiene que f no tiene mnimo
(, 0].

Ejercicio 3.6 Sea f : Rn R continua. Pruebe que x es un minimizador global de f


en Rn si y solo si todo x Rn tal que f (x) = f (x) es un minimizador local.

Solucion.
Si x es un minimizador global de f entonces todo x tal que f (x) = f (x) es un minimizador
global y por lo tanto local.
Supongamos ahora que todo x Rn tal que f (x) = f (x) es un minimizador local.
Probaremos que x es un minimizador global. Por contradiccion, supongamos que existe
Rn tal que f () < f (x) y 0 < f (x) f () tomando = (f (x) f ())/2 y usando la
denicion de continuidad de f en , existe > 0 tal que, para todo x B(, ) se tiene:
f (x) f ()
|f (x) f ()| < .
2
f (x)f ()
Tomando x B(, ) tal que f (x) = f (x) entonces tenemos |f (x) f ()| < 2
, lo
que es una contradiccion.

Ejercicio 3.7 Sea f una funcion continua y limitada inferiormente. Seja u un min-
imizador de f , esto es, f (u) inf xRn f (x) + Dado > 0 considere g(x) := f (x) +

kx uk. Muestre que la funcion g tiene un minimizador irrestrito y que si v es un


minimizador irrestricto entonces:


f (v) f (u)


kv uk



f (v) f (x) + kx uk, para todo x Rn .

45
Solucion.

i. Probaremos que g tiene un mnimo global. Para ello, debido a la continuidad de g,


es suciente probar que g es coerciva (ver Corolario 3.4). Sabemos que:

inf f (x) + kxk kuk infn f (x) + kx uk f (x) + kx uk
xRn xR
Como kx uk es coerciva tenemos que, si kxk + entonces:

infn f (x) + (kxk kuk) +
xR
As, g(x) +. Luego g es coerciva.

ii. Si v es un minimizador de g entonces g(v) g(x), para todo x Rn . En particular


tomando x = u resulta f (v) + kv uk f (u) y as f (v) f (u).

iii. Como f (v) + kv uk f (u) y por hipotesis f (u) infn f (x) + , se tiene que:
xR


f (v) + kv uk f (u) infn f (x) + f (v) + ,
xR

lo que implica que kv uk .

iv. Como v es un minimizador irrestricto de g, entonces:



f (v) + kv uk f (x) + kx uk


f (x) + kx vk + kx vk.

De la desigualdad anterior obtenemos que f (v) f (x) + kx uk, para todo
x Rn .

Ejercicio 3.8 Sea f : Rn R una funcion dada. Pruebe que f es semicontinua inferior
en Rn si y solo si, Lf () = {x Rn : f (x) } es cerrado para todo R.

Solucion. Probemos inicialmente que si f es semicontinua inferior en Rn , entonces


Lf () = {x R : f (x) } es cerrado para todo R. Sea x Lf (), entonces existe
una sucesion {xk } Lf () tal que xk x, k + y como f es s.c.i entonces:

f (x) lim inf f (xk ). (3.6)


k

46
Ya que xk Lf () entonces f (xk ) , luego tomando limite inferior tenemos

lim inf f (xk ) lim inf = ,


k k

esto es,
lim inf f (xk ) . (3.7)
k

Ahora de (3.6) y (3.7) concluimos que f (x) y as, x Lf (). Por lo tanto, Lf () es
cerrado.
Reciprocamente, si Lf () = {x Rn : f (x) } es cerrado para todo R, probaremos
que f es semicontinua inferior en Rn .
Por contradiccion, supongamos que existe una sucesion {xk } que satisface xk x y
f (x) > lim inf k f (xk ), entonces existe R tal que:

f (x) > > lim inf f (xk ). (3.8)


k


De aqu se deduce que > lim inf k f (xk ) := supnIN inf kn f (xk ) , pero esto im-
plica que > inf kn f (xk ), para todo n IN , y en consecuencia > inf kn f (xk ), para
todo n IN .
As existe una subsucesion {xkl } de {xk } tal que f (xkl ) . De aqu,

xkl Lf () y xkl x.

Ademas, como Lf () es cerrado se tiene que x Lf () y as f (x) . De lo anterior


y de (3.8) tenemos que f (x) > f (x), lo que es una contradiccion. Por lo tanto f es
semicontinua inferior.

Ejercicio 3.9 Estudie la coercividad de la funcion f (x) = aT x, donde a Rn .

Solucion. estudiaremos dos casos:

i) Si a = 0, entonces la funcion es f (x) = 0, para todo x Rn y as esta no es coerciva.

ii) Si a 6= 0, entonces existe i0 {1, 2, ..., n} tal que ai0 6= 0. Sin perdida de generalidad,
supongamos que ai0 > 0, entonces podemos denir la sucesion {xk }, tal que xk =

47
(0, 0, ..., k, 0, ..., 0), donde el valor de k corresponde a la componente i0 , y obtener
que ||xk || +. Luego, se tiene que

lim f (xk ) = lim aT xk = lim ai0 k = .


k+ k+ k+

De ambos casos, concluimos que la funcion f no es coerciva.

Ejercicio 3.10 Estudie la coercividad de f (x) = aT x + ||x||2 , donde > 0 y x Rn

Solucion. Por la desigualdad de Cauchy-Schwartz se tiene que f (x) (||x|| ||a||)||x||.


Sea {xk } una sucesion tal que limk ||xk || = +. De aqu y de la desigualdad anterior
se tiene que limk f (xk ) = +.

3.5 Ejercicios Propuestos


1. El problema de la cartera de inversion del Ejemplo 2.4 consiste en resolver
( n n
)
X X
min f (x) : xi = 1, G xi Ri , 0 x i Si ,
i=1 i=1
Pn Pn1 Pn
donde f (x) = f (x1 , ..., xn ) = i=1 x2i i2 + 2 i=1 j=i+1 xi xj ij . Vericar si el
problema tiene solucion.

2. El problema de transporte del Ejemplo 2.7 consiste en resolver:


( n n n m
)
XX X X
min cij xij : xij = ai , xij = bj , xij 0 .
j=1 i=1 j=1 i=1

Vericar si el problema tiene solucion.

3. Considere el problema:

opt xT Ax : kxk = 1 .

Vericar si el problema tiene maximos y mnimos.

4. Estudie la existencia de soluciones del problema:



min f (x1 , x2 ) = ln(x21 + x22 ) : (x1 , x2 ) R2 .

48
5. Una empresa produce dos bienes en competencia perfecta, siendo p1 y p2 sus precios
respectivamente. La funcion de costo de la empresa biene dado por:

c(x1 , x2 ) = 2x21 + x1 x2 + 2x22 ,

donde x1 , x2 son las cantidades producidas por el bien 1 y 2 respectivamente. El


objetivo es maximizar la funcion benecio:

b(x1 , x2 ) = Ingresos Costos = (p1 x1 + p2 x2 ) (2x21 + x1 x2 + 2x22 ).

As el problema de optimizacion es

max{f (x1 , x2 ) = (p1 x1 + p2 x2 ) (2x21 + x1 x2 + 2x22 ) ; x1 0, x2 0}.

Analizar si el problema tiene solucion.

6. Se quiere construir una cisterna metalica abierta para agua, con un triangulo
rectangulo como base y lados verticales. Si el volumen de la cisterna debe ser
de 2m3 , obtener el problema de optimizacion tal que la cisterna tenga la menor
area. El problema tiene solucion?

49
Captulo 4

Condiciones de Optimalidad de
Problemas Diferenciables

4.1 Problemas sin Restricciones


Apesar de que en la practica la gran mayoria de problemas de optimizacion son restric-
tos, el estudio de tecnicas para optimizacion irrestricta es importante por varias razones,
una de ellas es que la mayora de algoritmos resuelven problemas restrictos transfor-
mandolos en una sucesion de problemas irrestrictos, la segunda es que las tecnicas de
optimizacion irrestricta pueden ser extendidas de manera natural para resolver proble-
mas con restricciones.
En esta seccion consideraremos el problema irrestricto de optimizacion

min{f (x) : x Rn } (4.1)

donde f es una funcion denida en el espacio euclidiano Rn .

Teorema 4.1 (Condicion Necesaria de Primer Orden) Sea f : Rn R una funcion


diferenciable en x . Si x es un punto de mnimo local del problema (4.1), entonces

f (x ) = 0.

Demostracion. Supongamos que d = f (x ) 6= 0. Denamos h(t) = f (x(t)), donde

50
x(t) = x td. Ya que f es diferenciable en x se tiene que h es diferenciable en 0 y

h(t) = h(0) + th (0) + o(t),

o(t)
donde limt0 t
= 0. Como x es un mnimo local de f , entonces t = 0 es mnimo
local de h, as, existe > 0 tal que h(t) h(0) 0, para todo t (, ). Usando
o(t)
este resultado en la ecuacion anterior, obtenemos h (0) + t
0, para todo t (0, ).
Tomando lmite cuando t converge para cero se obtiene que h (0) 0. Usando la regla de
la cadena concluimos que, 0 > kf (x )k2 = f (x )T d 0, implicando que 0 > 0,
lo que es una contradiccion. Por lo tanto obtenemos el resultado deseado.

Ejemplo 4.1 Sea f : R2 R tal que f (x1 , x2 ) = x21 + x22 , el punto x = (0, 0) es un
mnimo global y cumple f (x ) = 0.

Observacion 4.1 La condicion f (x) = 0 es necesaria pero no suficiente para carac-


terizar mnimos de f pues existen funciones donde el gradiente de la funcion se anula en
algun punto sin embargo tal punto no es un mnimo global ni local. En efecto, considere
la funcion f (x1 , x2 ) = x21 x22 , el punto x = (0, 0) cumple f (x) = 0 pero este no es
mnimo global ni local ya que en cualquier vecindad de x la funcion f toma valores may-
ores y menores que 0 = f (0, 0) (ver Figura 4.1). Para otro ejemplo, considere la funcion
f (x1 , x2 ) = x21 x22 , el punto x = (0, 0) satisface la condicion f (x ) = 0 pero x no
es un mnimo (es maximo global).

Definicion 4.1 Dado f : Rn R, un punto x Rn es llamado punto crtico o esta-


cionario si f (x) = 0.

Observacion 4.2 Del Teorema 4.1 concluimos que los candidatos a mnimos del pro-
blema irrestricto se obtienen resolviendo el sistema f (x) = 0.

Teorema 4.2 (Condicion Necesaria de Segundo Orden) Sea f : Rn R una


funcion dos veces diferenciable en x . Si x es un punto de mnimo local del problema
(4.1), entonces f (x ) = 0 y 2 f (x ) es semidefinida positiva.

51
x3
f (x1 , x2 ) = x21 x21

x2
x1

Figura 4.1: El punto x = (0, 0) satisface f (x) = 0 pero no es mnimo ni maximo.

Demostracion. Que f (x ) = 0 es consecuencia del Teorema 4.1, por eso solo probare-
mos que para todo d Rn :
dT 2 f (x )d 0.

La desigualdad es obvia si d = 0. Por eso, sea d 6= 0 y denamos h(t) = f (x(t)), donde


x(t) = x td. Ya que f es dos veces diferenciable en x , tenemos que h es dos veces
diferenciable en 0 y por la aproximacion de Taylor de segundo orden tenemos
t2
h(t) = h(0) + th (0) + h (0) + o(t2 ),
2
o(t2 )
donde limt0 t2
= 0. Como x es un mnimo local, t = 0 es mnimo local de h, as,
existe > 0 tal que h(t) h(0) 0, para todo t (, ). Usando este resultado en la
ecuacion anterior resulta
t2
th (0) + h (0) + o(t2 ) 0, para todo t (, ).
2
Como h (0) = f (x )T d = 0 y h (0) = dT 2 f (x )d, tenemos que
o(t2 )
dT 2 f (x )d + 2 0, para todo t (0, ).
t2
Tomando lmite cuando t converge para cero tenemos el resultado deseado.

Ejemplo 4.2 Apliquemos el resultado del teorema anterior a la funcion f (x1 , x2 ) =


x21 x22 . El punto x = (0, 0) cumple f (x) = 0 sin embargo la hessiana 2 f (x ) no es

52
semidefinida positiva, por lo tanto usando el Teorema 4.2 tenemos que x no es un punto
de mnimo local.

Observacion 4.3 La condicion dada en el Teorema 4.2 es solo necesaria y no suficiente.


En efecto, considere la funcion f (x) = x3 , el punto x = 0 satisface f (x) = 0 y
f (x) = 0 0 pero no es mnimo local y por lo tanto tampoco es mnimo global.

Para dar una condicion suciente de mnimo local necesitamos la siguiente denicion.

Definicion 4.2 (Mnimo local estricto) Sea f : Rn R una funcion. Un punto


x Rn es un punto de mnimo local estricto de f si existe > 0 tal que f (x) < f (x),
para todo x B(x, ), x 6= x.

Teorema 4.3 (Condicion Suficiente de Segundo Orden) Sea f : Rn R una


funcion dos veces diferenciable en x . Si f (x ) = 0 y 2 f (x ) es definida positiva
entonces, x es un punto de mnimo local estricto del problema (4.1).

Demostracion. Por contradiccion, supongamos que x no es un punto de mnimo local


estricto, entonces existe una susecion {xk }, xk 6= x , tal que xk x y f (xk ) f (x ),
para todo k IN . De la aproximacion de Taylor de segundo orden y de la desigualdad
anterior tenemos
1
0 f (xk ) f (x ) = f (x )T (xk x ) + (xk x )T 2 f (x )(xk x ) + o(kxk x k2 ),
2
o(kxk x k2 )
donde limxk x kxk x k2
= 0. Usando la primera hipotesis del teorema (f (x ) = 0) y
dividiendo por kxk x k2 tenemos
 k T  k 
1 x x 2 x x o(kxk x k2 )
0 f (x ) + . (4.2)
2 kxk x k kxk x k kxk x k2
Observemos que la sucesion  
k xk x
{d } =
kxk x k
es acotada, de aqu existe una subsucesion {dkj } que converge a un punto z 6= 0. Tomando
en particular k = kj en (4.12) y haciendo tender j + tenemos

0 z T 2 f (x )z,

lo que contradice la segunda hipotesis del teorema.

53
Ejemplo 4.3 Consideremos el problema
3
min{f (x1 , x2 ) = x21 + x1 x2 + x22 2x1 x2 + 2 : (x1 , x2 ) R2 }.
2
Para hallar los puntos de mnimo usaremos las condiciones de primer y segundo orden.
Usando la condicion de primer orden tenemos

2x1 + x2 = 2
x1 + 3x2 = 1.

Puede verificarse facilmente que x1 = 1 y x2 = 0 resuelven el sistema lineal, as x = (1, 0)


es el unico candidato a solucion del problema. La matriz hessiana de f es

2 1
2 f (x1 , x2 ) =
1 3

la cual es simetrica y definida positiva para todo (x1 , x2 ) R2 , en particular para


x = (1, 0), luego usando el Teorema 4.3 obtenemos que x es un mnimo local estricto.

Por otro lado, podemos garantizar tambien que x es un mnimo global (unico). En
efecto, la funcion objetivo f puede ser expresado como
1
f (x1 , x2 ) = xT Ax bT x + c,
2
donde
2 1 2
A= , b = , c = 2.
1 3 1
Visto que la matriz A es simetrica y definida positiva, usando el Ejemplo 3.7, obtenemos
que x es un mnimo global.

Ejemplo 4.4 Considere el problema


x31 x21 x22
min{f (x1 , x2 ) = + + : (x1 , x2 ) R2 }.
3 2 2
Usando la condicion necesaria de primer orden tenemos
f
x1
= x21 + x1 = 0
f
x1
= x2 = 0.

54
Luego los candidatos a solucion son (0, 0) y (1, 0). Usaremos ahora la condicion de
segundo orden. La matriz hessiana de f es

2x1 + 1 0
2 f (x1 , x2 ) = .
0 1

Evaluando en los puntos candidatos a solucion tenemos:



1 0
2 f (0, 0) = 0, entonces por el Teorema 4.3, x = (0, 0) es un mnimo
0 1
local estricto.

1 0
2 f (1, 0) =  0 entonces por el Teorema 4.2, x = (1, 0) no es
0 1
un mnimo local y por lo tanto no es mnimo global.

Observacion 4.4 El recproco del teorema anterior es falso. En efecto, considere por
ejemplo f (x) = x4 y x = 0. El punto x es un mnimo global estricto de f pero f (x) = 0.

Observacion 4.5 Consideremos ahora el problema de maximizacion

max{f (x) : x Rn }.

Como vimos anteriormente este problema es equivalente a

min{f (x) : x Rn }.

Usando los resultados obtenidos para el problema de minimizacion tenemos que las condi-
ciones para el problema de maximizacion son:

1. Condicion necesaria de primer orden: si f : Rn R es diferenciable en un punto


de maximo local x , entonces f (x ) = 0.

2. Condicion necesaria de segundo orden: si f : Rn R es dos veces diferenciable en


un punto de maximo local x , entonces 2 f (x )  0.

3. Condicion suficiente de segundo orden: si el punto x satiface f (x) = 0 y tambien


2 f (x) 0, entonces, x es un punto de maximo local estricto de f .

55
4.2 Restricciones Arbitrarias
Consideremos el problema de optimizacion

min{f (x) : x X} (4.3)

donde f : X Rn R. Dado una funcion : R Rn denotamos C k [a, b], si


existe un intervalo abierto U de R tal que [a, b] U y C k (U ).

Definicion 4.3 Dado x X, llamamos una curva en X de clase C k partiendo de x a


una funcion : [0, ] X, tal que (0) = x y C k [0, ].

Teorema 4.4 (Condicion Necesaria de Primer Orden) Si x X es un minimizador


local de (4.3), : [0, ] Rn es una curva en X de clase C 1 partiendo de x y f es
diferenciable en x , entonces f (x )T (0) 0.

Demostracion. Denamos la funcion : [0, ] R tal que (t) = f ((t)). Como x


es un minimizador local, existe > 0 tal que

f (x ) f (x), para todo x B(x , ) X. (4.4)

Por la conexidad de la bola, existe 1 (0, ) tal que (t) B(x , ) X, para todo
t [0, 1 ]. As, de (4.4) tenemos que (0) (t), para todo t [0, 1 ). En particular,
(t)(0)
(0) (t), para todo t (0, 1 ). Luego, t
0, para todo t (0, 1 ). Como es
derivable en cero tomando lmite cuando t va para cero y usando la regla de la cadena
tenemos f (x )T (0) = (0) 0.

Corolario 4.1 Si x es un punto interior de X tal que x es un minimizador local de


(4.3) y f es diferenciable en x , entonces f (x ) = 0.

Prueba. Por contradiccion, supongamos que f (x ) 6= 0. Como x es un punto interior,


existe > 0 tal que B(x , ) X. Sea 1 = /(2 kf (x )k) y deniendo : [0, 1 ] Rn
tal que (t) = x tf (x ), tenemos que (0) = x , (0) = f (x ) y (t) B(x , )
X. As, es una curva en X de clase C 1 pasando por x y como x es un mnimo local
entonces por el Teorema 4.4 tenemos f (x )T f (x ) 0. Esto implica que

0 f (x )T f (x ) = kf (x )k2 < 0,

56
lo que es una contradiccion. Por lo tanto obtenemos el resultado deseado.

Ejemplo 4.5 Considere el problema min{f (x) : x > 0} donde f : Rn R es diferencia-


ble en Rn . Por la caracterstica del problema el punto de mnimo local o global, si existe,
es un punto interior de Rn++ , entonces por Corolario 4.1 la condicion de primer orden es
f (x ) = 0.

Observacion 4.6 Si el punto de mnimo local no es un punto interior, el resultado del


Corolario 4.1 puede ser falso. En efecto, considere por ejemplo la funcion f (x) = (x+2)2
y X = [0, 1], el punto x = 0 es un mnimo global (no es un punto interior de X) y
f (0) = 4 6= 0.

Observacion 4.7 La condicion del Corolario 4.1 no es suficiente. En efecto, considere


la funcion f (x) = x3 y X = [1, 1]. El punto x = 0 pertenece al interior de X y
satisface f (0) = 0 pero no es un mnimo local.

Teorema 4.5 (Condicion Necesaria de Segundo Orden) Sea x un minimizador


local de (4.3) y f dos veces diferenciable en x .

1. Para cada curva : [0, ] X de clase C 1 partiendo de x se obtiene

f (x )T (0) = (0) 0,

donde (t) = f ((t)).

2. Si : [0, ] X es una curva en X de clase C 2 partiendo de x y (0) = 0,


entonces (0) 0.

Demostracion. El item 1 es dado por el Teorema 4.4.


Probaremos el item 2. Cuando (0) = 0 tenemos por el desarrollo de Taylor
1
(t) = (0) + (0)t2 + o(t2 ),
2
o(t2 )
donde limt0 t2
= 0. Como x es un minimizador local existe 1 (0, ) tal que
1
(t) (0) = (0)t2 + o(t2 ) 0, para todo t (0, 1 ).
2
Dividiendo por t2 y tomando lmite cuando t va para cero tenemos nuestro resultado.

57
Definicion 4.4 Dado x X, diremos que es una curva en X de clase C k pasando
por x si : [, ] X, > 0, (0) = x y C k [, ].

Lema 4.1 Si x X es un minimizador local de (4.3), es una curva en X de clase


C 1 pasando por x y f diferenciable en x , entonces f (x )T (0) = 0.

Prueba. Denamos las siguientes curvas 1 : [0, ] X y 2 : [0, ] X dadas por


1 (t) = (t) y 2 (t) = (t). As obtenemos que 1 y 2 parten de x y como x es un
mnimo local, usando el Teorema 4.4 tenemos

f (x )T (0) = f (x )T 1 (0) 0,

f (x )T ( (0)) = f (x )T 2 (0) 0.

De ambas desigualdades obtenemos el resultado deseado.

Corolario 4.2 Si f es dos veces diferenciable en x , x es un punto interior de X y un


minimizador local de (4.3), entonces f (x ) = 0 y 2 f (x ) es semidefinida positiva.

Prueba. La primera parte fue probado en el Corolario 4.1, probaremos la segunda


parte. Como x es un punto interior de X, existe > 0 tal que B(x , ) X. Sea
h i

d Rn (arbitrario) diferente de cero y denamos la curva : 2kdk , 2kdk Rn tal que
(t) = x + td. Tenemos que es una curva de clase C 1 pasando por x y que (t) X.
Por el lema 4.1 tenemos que (0) = f (x )T d = 0, donde (t) = f ((t)). Usando ahora
el Teorema 4.5 tenemos que dT 2 f (x )d = (0)2 f (x ) (0) = (0) 0, esto es lo que
deseabamos probar.

Observacion 4.8 El recproco del Corolario 4.2 es falso. En efecto, considere la funcion
f (x) = x3 y X = (1, 1), el punto x = 0 satisface f (0) = 0 y f (0) = 0 0, pero no
es un punto de mnimo local.

Definicion 4.5 (Mnimo local estricto) Sea f : X Rn R una funcion. Un


punto x Rn es un mnimo local estricto de f sobre X si existe > 0 tal que f (x) < f (x),
para todo x B(x, ) X, x 6= x.

58
Teorema 4.6 (Condicion Suficiente de Segundo Orden) Si f : X R es una
funcion dos veces diferenciable en un punto interior x de X, tal que f (x ) = 0 y
2 f (x ) es definida positiva, entonces x es un minimizador local estricto del problema
(4.3).

Demostracion. Analogo al caso irrestricto.

Observacion 4.9 El recproco del teorema anterior es falso. En efecto, considere la


funcion f (x) = x4 y X = (2, 2), el punto x = 0 es un punto interior de mnimo global
estricto sin embargo se tiene que f (0) = 0 no es positivo.

Observacion 4.10 Consideremos el siguiente problema

min{f (x) : x U },

donde U es un conjunto abierto de Rn . Usando los corolarios anteriores tenemos las


siguientes condiciones de optimalidad para problemas restrictos a conjuntos abiertos:

1. Condicion necesaria de primer orden sobre abiertos: si x es un mnimo local de f


en U y f es diferenciable en x entonces f (x ) = 0.

2. Condicion necesaria de segundo orden sobre abiertos: si x es un mnimo local de


f en U y f es dos veces diferenciable en x , entonces 2 f (x )  0.

3. Condicion suficiente de segundo orden sobre abiertos: si el punto x U satiface


f (x) = 0 y 2 f (x) 0, entonces, x es un punto de mnimo local estricto de f
sobre U .

Concluimos que las condiciones de optimalidad de problemas restrictos sobre conjuntos


abiertos son identicas a las condiciones de optimalidad de problemas sin restricciones.

4.3 Restricciones de Igualdad


Consideremos el problema de optimizacion con restricciones de igualdad

(p) min{f (x) : h(x) = 0}

59
donde h : Rn Rm es una funcion tal que h(x) = (h1 (x), h2 (x), . . . , hm (x)), con hi :
Rn R. Poniendo en el formato general tenemos que X = {x Rn : h(x) = 0}.

Definicion 4.6 Sea x X, llamamos conjunto tangente a X en el punto x (denotado


por Tx X) al conjunto de los vectores tangentes a las curvas en X pasando por x, es decir,

Tx X = {v Rn : existe : [, ] X, C ([, ]) tal que (0) = x y (0) = v}

Tx X
(0) = v
x = (0)

(t)

Figura 4.2: Graco del conjunto tangente de X.

Usaremos la siguiente notacion



h1
(x) h 1
(x) ... h1
(x) h1 (x) T
x1 x2 xn
h2
x1 (x) h 2
(x) h2
. . . x (x) h2 (x)T

h (x) = x2 n = ,
.. .. ... .. ..
. . . .

hm
x1
(x) h m
x2
(x) . . . hm
xn
(x) hm (x)T

donde hi (x)T denota la transpuesta del vector columna hi (x).

Podemos relacionar Tx X con el nucleo del Jacobiano de h, denotado por Ker(h (x)).

Lema 4.2 Para todo x X, Tx (X) Ker(h (x)).

Prueba. Sea v Tx X, entonces existe : [, ] X tal que (0) = x y (0) = v.


Como (t) X, para todo t [, ], entonces, h((t)) = 0. Derivando usando la regla
de la cadena, tenemos que h ((t)) (t) = 0. Evaluando en t = 0 y usando (0) = v
tenemos que h (x)v = 0. As, v Ker(h (x)).

60
Observacion 4.11 El recproco del lema anterior no es verdadero. En efecto, consider-
emos por ejemplo h(x1 , x2 ) = x1 x2 , entonces X = {(x1 , x2 ) R2 : h(x1 , x2 ) = x1 x2 = 0}.
Tomando x = (0, 0) se tiene T(0,0) X = {(v1 , v2 ) R2 : v1 v2 = 0}. En efecto, sea v
T(0,0) X entonces existe : [, ] X tal que (0) = (0, 0) y (0) = v. De aqu se
tiene que 1 (t)2 (t) = 0, para todo [, ]. Lo anterior implica 1 (t) = 0 o 2 (t) = 0,
para todo t (, ).
Tomando en particular para t = 0 obtenemos 1 (0)2 (0) = v1 v2 = 0, es decir que
1 (0)2 (0) = 0. Recprocamente, sea v = (v1 , v2 ) R2 tal que v1 v2 = 0. Dado > 0 defi-
namos : [, ] R2 : (t) = (0, 0) + t(v1 , v2 ), entonces (0) = (0, 0), (0) = (v1 , v2 )
y 1 (t)2 (t) = (tv1 )(tv2 ) = 0. As, (t) X y por lo tanto v = (v1 , v2 ) T(0,0) X.
Por otro lado, Ker(h ((0, 0))) = {(v1 , v2 ) R2 : 0v1 + 0v2 = 0} = R2 . As obtenemos
que Ker(h ((0, 0))) 6 T(0,0) X.

Para tener la igualdad necesitamos introducir algunas hipotesis sobre h.

Definicion 4.7 Decimos que x X = {x Rn : h(x) = 0} es un punto regular, si el


rango de h (x) es igual a m ({h1 (x), . . . , hm (x)} es linealmente independiente).

Lema 4.3 Sea A Rmn , m < n, con rango m entonces, AAT es invertible.

Prueba. Sea v = (v1 , v2 , . . . vn ) Rm tal que (AAT )v = 0, probaremos que v =


0. Como (AAT )v = 0, entonces AT v Ker(A). Usando el resultado (de Algebra

Lineal) Ker(A) = (Im(AT )) , tenemos AT v Im(AT ) . Luego, obtenemos que
(AT v)T (AT v) = 0. Denotando A como sigue

AT1


AT2
A=


.. ,
.

T
Am

tenemos que AT v = v1 A1 + v2 A2 + . . . + Am vm = 0. Como A1 , . . . , Am son linealmente


independientes se tiene que v = 0.

Teorema 4.7 Sean X = {x Rn : h(x) = 0}, h C k (Rn ), x X un punto regular,


entonces Tx X = Ker(h (x)).

61
Demostracion. La inclusion Tx X Ker(h (x)) (sin la hipotesis de punto regular)
fue probado en el lema 4.2. Probaremos ahora la otra inclusion. Sea v Ker(h (x)),
entonces
h (x)v = 0. (4.5)

Consideremos el siguiente sistema de ecuaciones (en general no lineal)

(t, u) = h(x + tv + h (x)T u) = 0

Para x y v jos, este sistema es de m ecuaciones y m + 1 incognitas y cumplen las


siguientes condiciones

i) i C k (R Rm ), i = 1, . . . , m, donde i (t, u) = hi (x + tv + h (x)T u).

ii) Tomando (t, u) = (0, ), donde denota el vector nulo, tenemos que

(0, ) = h(x) = 0

iii) La matriz Jacobiana de (con respecto a u) evaluado en el punto (0, ) es

(0, ) = h (x)h (x)T ,

el cual es no singular ya que h (x) tiene rango m (ver Lema 4.3).

Luego por el teorema de la funcion implcita (ver Teorema 1.6), existe C k denido

en [, ] tal que (0) = , y (t) = (t, (t)) = h x + tv + h (x)T (t) = 0, para todo
t [, ].
Derivando y evaluando en t = 0, tenemos (0) = h (x)(v + h (x)T (0)) = 0. Usando
(4.5), la ecuacion anterior se reduce a h (0) = (h (x)h (x)T ) (0) = 0. Como h (x)h (x)T
es invertible se tiene que (0) = . Ahora denamos : [, ] Rn tal que (t) =
x + tv + h (x)T (t). De la denicion anterior tenemos que (0) = x, (t) X y (0) = v.
As, v Tx X y de esta manera Ker(h (x)) Tx X, por lo tanto se obtiene el resultado.

Teorema 4.8 Si x es un minimizador local regular del problema

min{f (x) : h(x) = 0},

entonces f (x )Ker(h (x )).

62
Demostracion. Sea v Ker(h (x )) (arbitrario). Como x es un punto regular, por
Teorema 4.7 tenemos que v Tx X y as existe : [, ] X tal que (0) =
x y (0) = v. Aplicando el Lema 4.1 tenemos f (x )T v = 0. As obtenemos que
f (x )Ker(h (x )).

Definicion 4.8 Considere el problema con restricciones de igualdad

(p) min{f (x) : h(x) = 0}.

Definimos la funcion de Lagrange o funcion Lagrangeana, con respecto al problema (p),


P
como L : Rn Rm R dado por L(x, ) = f (x) + m i=1 i hi (x), donde 1 , . . . , m R

son llamados multiplicadores de Lagrange.

Denotaremos
m
X
x L(x, ) = f (x) + i hi (x)
i=1
m
X
2xx L(x, ) 2
= f (x) + i 2 hi (x)
i=1

Teorema 4.9 (Condicion Necesaria de Lagrange) Si x Rn es un mnimo local


regular del problema (p), entonces existen unicos 1 , 2 , . . . , m R tal que x L(x , ) =
0.

Demostracion. Probemos la existencia de los multiplicadores de Lagrange. Como x


es un mnimo local regular, por el Teorema 4.8 obtenemos f (x )Ker(h (x )). Del re-
sultado de Algebra Lineal, Ker(h (x )) = (Im h (x )T ) , tenemos f (x ) Im h (x )T .
Luego existen 1 , 2 , . . . , m

R tal que
m
X
T
f (x ) = h (x ) = hi (x )i . (4.6)
i=1

Deniendo i = i , para todo i = 1, 2 . . . , m, obtenemos x L(x , ) = f (x ) +


Pm
i=1 i hi (x ) = 0. Ahora probaremos la unicidad. Supongamos que existe otro vector

6= tal que
f (x ) = h (x )T . (4.7)

63
Pm
De (4.6) y (4.7) tenemos h (x )T ( ) = 0, esto es, i=1 (i i )hi (x ) = 0. Como
{hi (x )} son linealmente independientes obtenemos que i = i para todo i = 1, . . . , n,
lo que es una contradiccion. Por lo tanto es unico.

Observacion 4.12 Sin la hipotesis de regularidad del punto de mnimo local tambien es
posible probar la existencia de los multiplicadores de Lagrange para una clase especial de
problemas de optimizacion como veremos en el siguiente resultado.

Teorema 4.10 Si x es un mnimo local del problema

min{f (x) : Ax = b},

entonces, existen 1 , 2 , . . . , m R tal que x L(x , ) = f (x ) + AT = 0.

Demostracion. Sean v Ker(A) y > 0. Denamos la curva (t) = x + tv, con


t [, ]. Tenemos que A((t)) = A(x + tv) = Ax + tAv = b, esto es, (t) X =
{x Rn : Ax = b}. Como (0) = x y (0) = v, podemos decir que es una curva que
pasa por x de clase C 1 . As por lema 4.1, tenemos f (x )T v = 0. Como v es arbitrario
entonces f (x )Ker(A). Debido que Ker(A) = (Im AT ) tenemos f (x ) Im AT .
Luego existen 1 , 2 , . . . , m R tal que f (x ) = AT . Deniendo i = i , para todo
los i = 1, 2 . . . , m, obtenemos x L(x , ) = f (x ) + AT .

Observacion 4.13 Los resultados de los teoremas 4.10 y 4.9, nos dice que para obtener
candidatos a solucion del problema de minimizacion con restricciones de igualdad debemos
resolver el siguiente sistema (m + n) (m + n)

P
m
x L(x, ) = f (x) + i hi (x) = 0
i=1

h(x) = 0.

Los puntos que resuelven esta ecuacion son llamados puntos crticos o estacionarios o
candidatos a solucion del problema (p).

Ejemplo 4.6 Considere el problema min{f (x1 , x2 ) = x21 + 21 x22 : x21 +x22 = 1}. La funcion
objetivo f es continua y el conjunto que define las restricciones del problema es compacto,

64
as el problema tiene un mnimo global. Usando la observacion anterior hallaremos los
candidatos a solucion. La funcion Lagrangeana es L(x1 , x2 , ) = (x21 + 12 x22 )+(x21 +x22 1)
y su gradiente con respecto a las variables (x1 , x2 ) es

2x1 + 2x1
x L(x1 , x2 , ) =
x2 + 2x2

As, para obtener los candidatos a solucion debemos resolver el siguiente sistema




x1 + x1 = 0


x2 + 2x2 = 0




x21 + x22 = 1.

Si x1 6= 0 y x2 6= 0 entonces de la primera y segunda ecuacion, respectivamente, = 1


y = 12 , lo que no puede suceder, as x1 = 0 x2 = 0. Si x1 = 0, entonces de la tercera
ecuacion x2 = 1, lo que implica que = 12 . Si x2 = 0, entonces de la tercera ecuacion
x1 = 1, lo que implica que = 1, por lo tanto los puntos candidatos a solucion son

{(0, 1), (0, 1), (1, 0), (1, 0)},

donde los dos primeros puntos es con respecto al multiplicador = 12 y los dos ultimos
con respecto a = 1, ver Figura 4.3. Observemos tambien que estos puntos son regu-
lares, as el mnimo global es uno de estos puntos. Debido que los puntos candidatos
forman un conjunto finito podemos inmediatamente encontrar el mnimo global sim-
plemente evaluando la funcion objetivo f y comparando sus valores. En efecto, como
f (0, 1) = 21 , f (0, 1) = 21 , f (1, 0) = 1 y f (1, 0) = 1, se tiene que los puntos de mnimo
global son (0, 1) y (0, 1), ver Figura 4.4.

Teorema 4.11 Supongamos que f : Rn R y h : Rn Rm son funciones dos veces


diferenciables en un minimizador local regular x del problema (p) y 1 , 2 , . . . , m R
son los multiplicadores de Lagrange dado en el Teorema 4.9, entonces v T 2xx L(x , )v
0, para todo v Ker(h (x )).

65
(0, 1)

(1, 0)
(1, 0)

(0, 1)

Figura 4.3: Puntos candidatos a mnimo.

Demostracion. Del Teorema 4.9 tenemos que f (x ) + h (x )T = 0. Tomemos un


v Ker(h (x )), por el Teorema 4.7 existe una curva en X de clase C 2 pasando por
x ((0) = x ) tal que (0) = v, as tenemos (0) Ker(h (x )). Denamos ahora
(t) = f ((t)). Por el Lema 4.1 se tiene (0) = f (x )T (0) = 0, luego usando el
Teorema 4.5
(0) = (0)T 2 f (x ) (0) + f (x )T (0) 0. (4.8)

Como (t) X se tiene i (t) := i hi ((t)) = 0. Tenemos as para todo t (, +)

i (t) = i hi ((t))T (t) = 0, y

i (t) = i (t)T 2 hi ((t)) (t) + i hi ((t))T (t) = 0

Sumando de i = 1 hasta m y evaluando en t = 0 tenemos


m
X m
X
i (0)
= (0) T
i 2 hi (x ) (0) + T h (x ) (0) = 0 (4.9)
i=1 i=1

Sumando (4.8) y (4.9) se sigue


m
!
X 
(0)T 2 f (x ) + i 2 hi (x ) (0) + f (x ) + h (x )T (0) 0
i=1

Finalmente, usando la condicion que f (x ) + h (x )T = 0 tenemos el resultado.

66
(0, 1)

(0, 1)

Figura 4.4: Puntos de mnimo global.

Teorema 4.12 Supongamos que f : Rn R y h : Rn Rm son funciones dos veces


diferenciables en x . Si x X = {x Rn : h(x) = 0} entonces

i. x L(x , ) = 0, para algunos 1 , 2 , . . . , m R.

ii. v T 2xx L(x , )v > 0, para todo v Ker(h (x )), v 6= 0.

entonces x es un mnimo local estricto de f en X.

Demostracion. Por contradiccion, supongamos que x no es un punto de mnimo local


estricto de f en X, entonces existe una susecion {xk } X, xk 6= x , tal que xk x y
f (xk ) f (x ), para todo k IN .
De la aproximacion de Taylor de segundo orden y de la desigualdad anterior tenemos

1
0 f (x )T (xk x ) + (xk x )T 2 f (x )(xk x ) + o(kxk x k2 ), (4.10)
2
o(kxk x k2 )
donde limxk x kxk x k2
= 0.
De forma analoga para cada hi , i = 1, ..., m, se tiene

1
0 = hi (xk ) hi (x ) = hi (x )T (xk x ) + (xk x )T 2 hi (x )(xk x ) + (kxk x k2 ),
2

67
(kxk x k2 )
donde limxk x kxk x k2
= 0. Multiplicando por i la igualdad anterior y sumando de
i = 1 hasta m la ecuacion anterior obtenemos
m
X m
X
T k 1
0= i hi (x ) (x x )+ (xk x )T i 2 hi (x )(xk x )+o(kxk x k2 ) (4.11)
i=1 i=1
2

Sumando (4.10) y (4.11), usando la hipotesis i del teorema y dividiendo por kxk x k2
tenemos
 T  
1 xk x xk x O(kxk x k2 )
0 2xx L(x , ) + . (4.12)
2 kxk x k kxk x k kxk x k2
Observemos que la sucesion  
k xk x
{d } =
kxk x k
es acotada, de aqu existe una subsucesion {dkj } que converge a un punto d 6= 0. Tomando
en particular k = kj en (4.12) y haciendo tender j + tenemos

dT 2xx L(x , )d 0 (4.13)

Por otro lado, como xk , x X, xk 6= x y por la aproximacion de primer orden se tiene


o(kxk x k)
0 = hi (x )T dk + , i = 1, ..., m
kxk x k
Tomando k = kj y j tenemos que hi (x )T d = 0, i = 1, 2, ..., m; esto es, d
Ker(h (x )).
Finalmente, de lo anterior y de (4.13) llegamos a una contradiccion con la hipotesis ii
del teorema.

Ejemplo 4.7 Vimos en el Ejemplo 4.6 que los puntos candidatos a solucion del problema
dado eran {(0, 1), (0, 1), (1, 0), (1, 0)}, donde los dos primeros puntos es con respecto al
multiplicador = 21 y los dos ultimos con respecto a = 1. Como la unica restriccion
es h(x1 , x2 ) = x21 + x22 1 tenemos que h (x1 , x2 ) = 2(x1 x2 ). Entonces podemos verificar
que Ker(h (x1 , x2 )) = {(v1 , v2 ) R2 : x1 v1 + x2 v2 = 0} y la matriz hessiana de la funcion
de Lagrange esta dado por

2 + 2 0
2xx L(x1 , x2 , ) =
0 1 + 2

Analizaremos en cada punto candidato a solucion.

68
i. Para x = (0, 1) con multiplicador = 12 tenemos

Ker(h (0, 1)) = {(v1 , v2 ) R2 : v2 = 0},



1 0
2xx L(0, 1, ) =
0 0

Luego, para todo v = (v1 , v2 ) Ker(h (0, 1)), v 6= (0, 0), tenemos que v1 6= 0 y as

v T 2xx L(0, 1, )v = v12 > 0,

y aplicando el Teorema 4.12 concluimos que el punto (0, 1) es un mnimo local


estricto.

ii. Para x = (0, 1) con multiplicador = 12 tenemos en forma analoga al caso


anterior que (0, 1) es un mnimo local estricto.

iii. Para x = (1, 0) con multiplicador = 1 tenemos

Ker(h (1, 0)) = {(v1 , v2 ) R2 : v1 = 0},



0 0
2xx L(1, 0, ) =
0 1

As, para todo v = (v1 , v2 ) Ker(h (1, 0)), v 6= (0, 0), tenemos que v2 6= 0 y se
tiene
v T 2xx L(0, 1, )v = v22 < 0,

luego aplicando el Teorema 4.11 y siendo (1, 0) un punto regular concluimos que
(1, 0) no es un mnimo local.

iv. Similar al caso anterior.

Observacion 4.14 Consideremos el problema de maximizacion:

(P ) max{f (x) : h(x) = 0}

Como vimos anteriormente este problema es equivalente a

(P ) min{f (x) : h(x) = 0}

69
La funcion de Lagrange y sus derivadas son

L(x, ) = f (x) + T h(x)


m
X
x L(x, ) = f (x) + i hi (x)
i=1
m
X
2xx L(x, ) 2
= f (x) + i 2 hi (x)
i=1
Usando los teoremas 4.9, 4.11, 4.12, se tiene que las condiciones de optimalidad para
el problema (P ) son:

1. Condicion necesaria de primer orden: Si x es un maximo local regular de (P )


P
m
entonces existen unicos 1 , 2 , . . . , m R tal que f (x) + i hi (x) = 0.
i=1
En efecto, como x es un maximo local regular, entonces x es un mnimo local
regular de (P ). De aqu, usando el Teorema 4.9, existen 1 , 2 , . . . , m R tal
P
m
que f (x) + i hi (x) = 0. Definiendo i = i , tenemos el resultado.
i=1

2. Condicion necesaria de segundo orden: Si x es un maximo local regular de (P )


P
m
entonces existen unicos 1 , 2 , . . . , m R tal que la matriz 2 f (x) + i 2 hi (x)
i=1
es semidefinida negativa sobre Ker(h (x)). En efecto, si x es un maximo local
regular de (P ) entonces x es un mnimo local regular de (P ) entonces, por Teorema
P
m
4.11 aplicado al problema (P ), la matriz 2 f (x) + i 2 hi (x), es semidefinida
i=1
2
P
m
positiva sobre Ker(h (x)), y por tanto f (x) + i 2 hi (x), es semidefinida
i=1
negativa sobre Ker(h (x)). Definiendo i = i , para i = 1, . . . , m se tiene el
resultado.

3. Condicion suficiente de segundo orden: Si x X = {x Rn : h(x) = 0} satisface


P
m
(a) estacionaridad del Lagrangiano x L(x, ) = f (x) + i hi (x) = 0.
i=1
P
m
(b) la Hessiana de la funcion de Lagrange 2xx L(x, ) = 2 f (x) + i 2 hi (x),
i=1
es definida negativa sobre el Ker(h (x))\{0}.

entonces, x es un maximo local estricto de f sobre X. En efecto, de (a) y (b)


P
m P
m
tenemos que f (x)+ (i )hi (x) = 0, y la matriz 2 f (x)+ (i )2 hi (x)
i=1 i=1

70
es definida positiva sobre Ker(h (x))\{0}, luego aplicando el Teorema 4.12, tenemos
que x es un mnimo local estricto de f , lo que es equivalente a x es un maximo
local estricto de f en X.

Observacion 4.15 Basados en la observacion anterior y la Observacion 4.13, con-


cluimos que para obtener candidatos de mnimo y maximo de un modelo de optimizacion
con restricciones de igualdad debemos resolver el siguiente sistema (m + n) (m + n)


Pm
L(x, ) = f (x) +
x h (x) = 0
i i
i=1


h(x) = 0.

Ejemplo 4.8 Como una aplicacion de los resultados previos, considere el problema

1
opt{f (x1 , x2 ) = x21 + x22 : x21 + x22 = 1}.
2

Por el Ejemplo 4.6 y la observacion anterior obtenemos que los candidatos a mnimo
y maximo son {(0, 1), (0, 1), (1, 0), (1, 0)}, donde los dos primeros puntos estan aso-
ciados al multiplicador = 12 y los dos ultimos a = 1. Ya vimos en el ejemplo
4.7 que (0, 1) y (0, 1) son los unicos mnimos locales estrictos y como tienen el mismo
valor entonces, estos son mnimos globales. Ademas podemos probar, usando las condi-
ciones suficientes de segundo orden para maximizacion, que los puntos (1, 0) y (1, 0)
son maximos locales estrictos y como tienen el mismo valor objetivo, entonces ellos son
maximos globales.

Ejemplo 4.9 Considere el problema de optimizacion

1 1
min{f (x1 , x2 , x3 ) = x21 + x22 + x23 : x21 + x22 + x23 = 1, x1 x2 = 0}
2 3

El problema tiene solucion pues f es continua y el conjunto que define las restricciones
es compacto. Hallaremos la solucion usando las condiciones de optimalidad de primer y
segundo orden. La funcion de Lagrange es

1 1
L(x1 , x2 , x3 , 1 , 2 ) = x21 + x22 + x23 + 1 (x21 + x22 + x23 1) + 2 x1 x2
2 3

71
Las ecuaciones de primer orden que debe satisfacer el punto de mnimo son



2x1 + 21 x1 + 2 x2 = 0






x2 + 21 x2 + 2 x1 = 0



(2/3)x3 + 21 x3 = 0





x21 + x22 + x23 = 1.





x1 x2 = 0.

Si x2 6= 0, entonces x1 = 0 y las ecuaciones se reducen a




2 x2 = 0






1 + 21 = 0



(2/3)x3 + 21 x3 = 0




x2 + x2 = 1.
2 3

de donde obtenemos los puntos (0, 1, 0) y (0, 1, 0) con 1 = 12 y 2 = 0. Si x1 6= 0,


entonces x2 = 0 y las ecuaciones se reducen a


1 + 1 = 0






2 x1 = 0



(1/3)x3 + 1 x3 = 0




x2 + x2 = 1.
1 3

Resolviendo el sistema, obtenemos los puntos (1, 0, 0) y (1, 0, 0) con 1 = 1 y 2 = 0.


Si x1 = x2 = 0, entonces x3 = 1 x3 = 1, con 1 = 13 y 2 R. Luego los puntos
candidatos a solucion para este caso son (0, 0, 1) y (0, 0, 1) con 1 = 13 y 2 R. De
los tres casos analizamos concluimos que los puntos candidatos a mnimo del problema
son

i) (0, 1, 0) y (0, 1, 0) con 1 = 21 y 2 = 0.

ii) (1, 0, 0) y (1, 0, 0) con 1 = 1 y 2 = 0.

iii) (0, 0, 1) y (0, 0, 1) con 1 = 13 y 2 R.

72
Como segunda etapa, analizaremos la condicion de segundo orden. La matriz hessiana
de la funcion de Lagrange es

2 + 21 2 0

2
xx L(x1 , x2 , x3 , 1 , 2 ) = 2 1 + 21 0 y

2
0 0 3
+ 21

x1 v 1 + x2 v 2 + x3 v 3 = 0
Ker(h (x)) = (v1 , v2 , v3 ) :
x2 v 1 + x1 v 2 = 0
Analizaremos todos los puntos candidatos a solucion.

a. x = (0, 1, 0) con 1 = 12 y 2 = 0. En este caso tenemos



1 0 0


2xx L(x, 1 , 2 ) = 0 0 0 ;

1
0 0 3

y Ker(h (0, 1, 0)) = {(0, 0, v3 ) : v3 R}. Sea v = (0, 0, v3 ) con v3 6= 0, entonces


v T 2xx L(x, 1 , 2 )v = 13 v32 < 0, luego por el Teorema 4.11, (0, 1, 0) y (0, 1, 0) no
son mnimos del problema.

b. x = (1, 0, 0) con 1 = 1 y 2 = 0. En este caso tenemos



0 0 0


2xx L(x, 1 , 2 ) = 0 1 0 ;

4
0 0 3

y Ker(h (1, 0, 0)) = {(0, 0, v3 ) : v3 R}. Sea v = (0, 0, v3 ) con v3 6= 0, entonces


v T 2xx L(x, 1 , 2 )v = 43 v32 < 0, luego por la condicion necesaria de segundo orden,
Teorema 4.11, (1, 0, 0) y (1, 0, 0) no son mnimos del problema.

c. x = (0, 0, 1) con 1 = 13 y 2 cualquier numero real, en este caso tenemos:



4
2 0
3

2xx L(x , 1 , 2 ) = 2 13 0 y

0 0 0

73
Ker(h (0, 0, 1)) = {(v1 , v2 , 0) : v1 , v2 R} = R2

1
Tomando en particular 2 = 3
tenemos que la matriz 2xx L(x , 1 , 2 ) es definida
positiva sobre R2 , por lo tanto por la condicion suficiente de segundo orden, Teorema
4.12, los puntos (0, 0, 1) y (0, 0, 1) son mnimos locales estrictos del problema
(observe que estos puntos no son puntos regulares es por eso que 2 no es unico).

4.4 Restricciones de Desigualdad


Consideremos el problema de optimizacion con restricciones de desigualdad

(pd) min{f (x) : g(x) 0}

donde f : Rn R es una funcion con valor real, g : Rn Rp es una funcion tal que
g(x) = (g1 (x), g2 (x), . . . , gp (x)), con gi : Rn R y la notacion g(x) 0 signica que
gi (x) 0, para todo i = 1, 2, . . . , p.

Definicion 4.9 Para cada x X = {x Rn : g(x) 0}, gi es llamada restriccion activa


en x si gi (x) = 0. Analogamente llamamos a gi restriccion inactiva en x si gi (x) < 0.

Dado x denotaremos
I(x) = {i {1, 2, . . . , p} : gi (x) = 0} .

Ejemplo 4.10 Consideremos el problema

min{x1 2x2 : 2x1 + x2 6; x1 + x2 4; x1 0; x2 0}.

La region viable es dada por X = {(x1 , x2 ) : 2x1 + x2 6; x1 + x2 4; x1 0; x2 0},


cuya grafica se muestra en la Figura 4.5. En este problema g1 (x1 , x2 ) = 2x1 + x2 6,
g2 (x1 , x2 ) = x1 + x2 4, g3 (x1 , x2 ) = x1 y g4 (x1 , x2 ) = x2 . Dado x = (3, 0), tenemos
que g1 (3, 0) = 0, g2 (3, 0) = 1, g3 (3, 0) = 3, g4 (3, 0) = 0. As g1 y g4 son restricciones
activas para (3, 0) y I(3, 0) = {1, 4}. Si tomamos x = (0, 4) entonces se puede verificar
facilmente que I(0, 4) = {2, 3}.

74
6

(0, 4)

X
(2, 2)

(0, 0) (3, 0) (4, 0)

Figura 4.5: Region viable.

Definicion 4.10 Un punto x X = {x Rn : g(x) 0} es llamado punto regular del


problema (pd) si {gi (x)}iI(x) existen y son linealmente independientes.

Lema 4.4 Si x es un minimizador local del problema (pd), entonces x es un mini-


mizador local del problema

min{f (x) : gi (x) = 0, i I(x )}.

Prueba. Inmediato de la denicion de mnimo local.

Lema 4.5 Si x es un minimizador local regular de (pd) entonces para cada i I(x ),
P
existen unicos i R tal que f (x ) + iI(x ) i gi (x ) = 0.

Prueba. Analogo al Teorema 4.9.

Observacion 4.16 El Lema anterior nos dice que en un punto de mnimo local regular el
f (x ) es combinacion lineal de los gradientes de las restricciones activas. El siguiente
resultado muestra que la combinacion dada es una combinacion conica (combinacion
lineal donde los escalares son no negativos).

Teorema 4.13 (Condicion de Karush-Kuhn-Tucker) Si x es un mnimo local reg-


ular del problema (pd), f y gi , i I(x ), son diferenciables en x y gi , i
/ I(x ), son

75
continuas en x , entonces existen unicos i R, para todo i I(x ), tal que

P
f (x ) +
iI(x ) i gi (x ) = 0

0, para todo i I(x ).
i

Demostracion. Por el lema anterior existen unicos i R tal que


X
f (x ) + i gi (x ) = 0. (4.14)
iI(x )

Probaremos que i 0, para todo i I(x ). Por contradiccion, supongamos que existe
k I(x ) tal que k < 0. Denamos XI(x ) = {x Rn : gi (x) = 0, i I(x )}, y

Xk = {x Rn : gi (x) = 0, i I(x ), i 6= k}

y sean Tx XI(x ) y Tx Xk sus respectivos conjuntos tangentes. Por la regularidad de


x , gk (x ) no es combinacion lineal de los otros gradientes de restricciones activas en
x . Por lo tanto existe y Tx (Xk ) tal que gk (x )T y < 0. Ahora, sea (t) una
curva en Xk pasando por x con (0) = y. Para t sucientemente pequeno se tiene,
(t) {x Rn : g(x) 0}. Sea (t) = f ((t)) entonces (0) = f (x )T y . Luego de
(4.14) tenemos
X
T f (x )y = i gi (x )T y
iI(x )
X
= k gk (x )T y i gi (x )T y
iI(x )

= k gk (x )T y.

Entonces (0) < 0, lo que es una contradiccion.

Ejemplo 4.11 Consideremos el problema del Ejemplo 4.10 y estudiemos la condicion


necesaria del teorema anterior. Para x = (3, 0), tenemos que los gradientes son f (3, 0) =
(1, 2), g1 (3, 0) = (2, 1), g4 (3, 0) = (0, 1), respectivamente podemos verificar
facilmente (como se ve en la grafica de la Figura 4.7) que f (3, 0) no puede ser expre-
sado como una combinacion conica de g1 (3, 0) y de g4 (3, 0). Luego, x = (3, 0) no es
un punto de mnimo.

76
g4

f (x)

g3

g3

g2
g1
g4

Figura 4.6: Interpretacion geometrica de las condiciones de KKT.

Ahora consideremos x = (0, 4), entonces resulta que los gradientes g2 (0, 4) = (1, 1),
g3 (0, 4) = (1, 0), f (0, 4) = (1, 2). Es facil probar, como se observa en la grafica
de la Figura 4.8, que el punto x = (0, 4) satisface las condiciones necesarias de KKT y
as este es un punto candidato para resolver el problema dado.

Ejemplo 4.12 Considere el problema

min{(x1 5)2 + (x2 5)2 : x21 x2 6, x1 + 3x2 12, x1 0, x2 0}

La grafica de la region viable se muestra en la Figura 4.9. Veremos que en x = (3, 3)


se cumplen las condiciones de primer orden. En efecto, en nuestro problema f (x1 , x2 ) =
(x1 5)2 +(x2 5)2 y las restricciones son g1 (x1 , x2 ) = x21 x2 6; g2 (x1 , x2 ) = x1 +3x2 12;
g3 (x1 , x2 ) = x1 ; g4 (x1 , x2 ) = x2 . As tenemos que I(3, 3) = {1, 2}. El gradiente de la
funcion objetivo y de las restricciones activas en (3, 3) son, respectivamente, f (3, 3) =
(4, 4), g1 (3, 3) = (6, 1), g2 (3, 3) = (1, 3). Se puede verificar facilmente que

f (3, 3) = 1 g1 (3, 3) + 2 g2 (3, 3),

donde 1 = 8/19 y 2 = 28/19 (ver el grafico de la Figura 4.10). As, x = (3, 3)


satisface las condiciones de primer orden del Teorema. Ademas, como el problema tiene

77
2 f (3, 0) = (1, 2)

1 g1 (3, 0) = (2, 1)

1 2

g4 (3, 4) = (0, 1)

Figura 4.7: Graca de las condiciones de KKT en el punto x = (3, 0).

solucion (la funcion objetivo es continua y el conjunto de restricciones forma un conjunto


compacto y aplicando el Corolario 3.1) podemos concluir que (3, 3) es un mnimo global
del problema.

Observacion 4.17 Las condiciones del Teorema 4.13 solo son necesarias y no sufi-
cientes, i.e, existen soluciones viables que verifican las condiciones de KKT pero, sin
embargo, no son soluciones optimas. Considere, por ejemplo, el problema

min{f (x1 , x2 ) : x21 x2 6, x1 + 3x2 12, x1 0, x2 0},

donde f (x1 , x2 ) = (x1 2)2 (x2 5)2 . El punto x = (2, 0) satisface las condiciones
de primer orden pero no es mnimo local. En efecto, dado > 0 y definiendo el punto

(x1 , x2 ) = (2 + , 0) donde (0, 6 2) (0, ) tenemos que (x1 , x2 ) B(x, ) y ademas

f (2 + , 0) = (2 + 2)2 (x2 5)2 < f (x)

Corolario 4.3 Con las hipotesis del teorema anterior y si ademas gi son diferenciables
/ I(x ), entonces existen unicos i 0, para todo i = 1, 2, . . . , m tales que
i
p
P

f (x
) + i gi (x ) = 0


i=1

i gi (x ) = 0, i = 1, 2 . . . , p




0, i = 1, 2 . . . , p.
i

78
2 f( 0, 4) = (2, 1)

1 g2 (0, 4) = (1, 1)

g3 (0, 4) = (1, 0) 1

Figura 4.8: Graca de las condiciones de KKT en el punto x = (0, 4).

Prueba. Inmediato del teorema previo.

Observacion 4.18 El resultado anterior nos ofrece un metodo para obtener los can-
didatos a solucion mediante la solucion del sistema que denominaremos condiciones de
optimalidad de KKT (Karush-Kuhn-Tucker)

P
f (x) + pi=1 i gi (x) = 0






gi (x) 0, i = 1, 2 . . . , p



i gi (x) = 0, i = 1, 2 . . . , p




0, i = 1, 2 . . . , p.
i

Los puntos x que resuelven el sistema son llamados puntos de KKT.

Ejemplo 4.13 Consideremos el problema dado en el Ejemplo 4.10, esto es,

min{x1 2x2 : 2x1 + x2 6; x1 + x2 4; x1 0; x2 0}

Usaremos el metodo para encontrar los candidatos a solucion. La funcion de Lagrange


es

L(x1 , x2 , 1 , 2 , 3 , 4 ) = x1 2x2 + 1 (2x1 + x2 6) + 2 (x1 + x2 4) 3 x1 4 x2

El gradiente de la funcion de Lagrange es

x L(x1 , x2 , 1 , 2 , 3 , 4 ) = (1 + 21 + 2 3 , 2 + 1 + 2 4 )

79
X
4
3 (3, 3)

(0, 0) 3

Figura 4.9: Graca de la region viable del Ejemplo 4.12.

Luego el sistema KKT queda establecido por




1 + 21 + 2 3 = 0





2 + 1 + 2 4 = 0





1 (2x1 + x2 6) = 0





2 (x1 + x2 4) = 0




x = 0
3 1

4 x2 = 0





2x1 + x2 6





x1 + x2 4





x1 , x2 0




1 , 2 , 3 , 4 0
De la tercera ecuacion tenemos que 1 = 0 2x1 + x2 6 = 0. Si 2x1 + x2 6 = 0,
llegamos a una contradiccion con el sistema dado, por lo tanto debemos tener que 1 = 0.
De aqu, la primera y segunda ecuacion se reducen, respectivamente, a

2 3 = 1 (4.15)

2 4 = 2 (4.16)

80
f (3, 3)
g2 (3, 3) = (1, 3)

g1 (3, 3) = (6, 1)

Figura 4.10: Graca de la combinacion conica.

De ambas ecuaciones se tiene que 2 6= 0 (Si 2 = 0 entonces 3 < 0 el cual contradice


la condicion de no negatividad de la variable). Luego de la cuarta ecuacion del sistema
se tiene que
x1 + x2 = 4 (4.17)

Tambien se tiene que 3 6= 0 (pues de lo contrario de (4.15) tendriamos 2 = 1 y de


(4.16), 4 = 1 < 0 lo que es una contradiccion). As de la quinta ecuacion del sistema
se obtiene que x1 = 0 y de (4.17), x2 = 4. Usando este resultado en la sexta ecuacion
del sistema obtenemos 4 = 0 y substituyendo en (4.16) se tiene 2 = 2. Finalmente de
(4.15), obtenemos 3 = 1. Concluimos as que el unico candidato a solucion del problema
es (0, 4) con los multiplicadores 1 = 4 = 0, 2 = 2 y 3 = 1.

Teorema 4.14 Si f , gi , i I(x ), son dos veces diferenciables en x , gi , i


/ I(x ), son
continuas en x y x es un punto de mnimo local regular del problema (pd), entonces
existen unicos puntos i 0, i = 1, ..., m, tal que
P
1. f (x ) + iI(x ) i gi (x ) = 0.

2. v T 2xx L(x , )v 0, para todo v Rn : gi (x )T v = 0, y todo i I(x ),

81
Pp
donde L(x, ) = f (x) + i=1 i gi (x).

Demostracion. La parte 1 es una consecuencia del Teorema 4.13. Probemos la parte


2. Del Lema 4.4 y del Teorema 4.11 tenemos que

X
v T 2 f (x ) + i 2 gi (x ) v 0, v Rn : gi (x )T v = 0, i I(x ).
iI(x )

Deniendo = 0, i
/ I(x ) obtenemos que

v T 2xx L(x , )v 0, v Rn : gi (x )T v = 0, i I(x ).

Teorema 4.15 Sean f, gi , i I(x ), dos veces diferenciables en x . Si x X =


{x Rn : g(x) 0} satisface:
P
i. f (x ) + iI(x ) i gi (x ) = 0, para algunos i 0, i I(x )
 P 
ii. v T 2 f (x ) + iI(x ) i 2 gi (x ) v > 0, para todo v Rn , v 6= 0 : gi (x )T v =
0, y todo i I(x ).

Entonces x es un mnimo local estricto de f en X.

Demostracion. Por contradiccion, supongamos que x no es un punto de mnimo local


estricto de f en X, entonces existe una susecion {xk } X, xk 6= x , tal que xk x y
f (xk ) f (x ), para todo k IN .
De la aproximacion de Taylor de segundo orden y de la desigualdad anterior tenemos
1
0 f (x )T (xk x ) + (xk x )T 2 f (x )(xk x ) + o(kxk x k2 ), (4.18)
2
o(kxk x k2 )
donde limxk x kxk x k2
= 0.
De forma analoga para cada gi , i I(x ), se tiene
1
0 gi (xk ) gi (x ) = gi (x )T (xk x ) + (xk x )T 2 gi (x )(xk x ) + (kxk x k2 ),
2
(kxk x k2 )
donde limxk x kxk x k2
= 0. Multiplicando por i la desigualdad anterior y sumando
para todo i I(x ) obtenemos
X 1 X k
0 i gi (x )T (xk x ) + (x x )T i 2 gi (x )(xk x ) + o(kxk x k2 )
2
iI(x )
iI(x )
(4.19)

82
Sumando (4.18) y (4.19), usando la hipotesis i del teorema y dividiendo por kxk x k2
tenemos

 k
T X  k 
1 x x 2 2 x x O(kxk x k2 )
0 f (x ) + i g i (x ) + .
2 kxk x k kxk x k kxk x k2
iI(x )
(4.20)
Observemos que la sucesion  
k xk x
{d } =
kxk x k
es acotada, de aqu existe una subsucesion {dkj } que converge a un punto d 6= 0. Tomando
en particular k = kj en (4.20) y haciendo tender j + tenemos

X
dT 2 f (x ) + i 2 gi (x ) d 0 (4.21)
iI(x )

Por otro lado, como xk 6= x y por la aproximacion de primer orden se tiene


o(kxk x k)
0 gi (xk ) = gi (x )T dk + k
, i I(x )
kx x k
Tomando k = kj y j tenemos que

gi (x )T d 0, i I(x ) (4.22)

Tambien de la aproximacion de primer orden para f tenemos

0 f (xk ) f (x ) = f (x )T (xk x ) + +o(kxk x k)

Dividiendo por ||xk x ||, tomando k = kj y tendiendo j obtenemos:


0 f (x )T d.

De la hipotesis i se tiene que


X
f (x )T d =
i gi (x )T d,
iI(x )

de aqu
gi (x )T d 0, i I(x ) (4.23)

De (4.22) y (4.23) se tiene que gi (x )T d = 0, i I(x ). Finalmente, de (4.21) llegamos


a una contradiccion con la hipotesis ii del teorema.

83
Ejemplo 4.14 Considere el problema

min{(x1 5)2 + (x2 5)2 : x21 x2 6, x1 + 3x2 12, x1 0, x2 0}

Vimos en el ejemplo 4.12 que el punto x = (3, 3) satisface la condicion i del Teorema
4.15. Como I(x ) = {1, 2}, entonces tenemos que

X 5+ 21 0
2 f (x ) + i 2 gi (x ) =
iI(x ) 0 5
donde 1 = 8/19, el cual es una matriz definida positiva en todo R2 en particular en el
conjunto {v R2 : v 6= 0, gi (x )T v = 0, i = 1, 2}. As se cumple tambien la condicion
ii del Teorema 4.15 y por lo tanto x = (3, 3) es un mnimo local estricto.

4.5 Restricciones Mixtas


Finalmente consideremos el problema de optimizacion no lineal




min f (x)


(P N L) h(x) = 0





g(x) 0

donde h : Rn Rm y g : Rn Rp son funciones dadas.


Podemos establecer condiciones analogas al Teorema 4.13 para el problema (P N L).
De manera similar a los casos anteriores, denimos punto regular del conjunto viable como
un punto donde los gradientes de las restricciones activas son linealmente independientes.
Las demostraciones son simples extensiones de los casos vistos anteriormente es por eso
se deja como ejercicio para el lector.

Teorema 4.16 Si x es un minimizador local de (P N L) y ademas es un punto regular,


esto es,
{h1 (x ), . . . , hm (x )} {gi (x ) , i I(x )}

es un conjunto linealmente independiente donde I(x ) = {i {1, . . . , p} : gi (x ) = 0} ,


entonces existen unicos 1 , . . . , m R y i 0 para todo i I(x ) tal que
Xm X
f (x ) + i hi (x ) + v i gi (x ) = 0.
i=1 iI

84
Corolario 4.4 Asuma las condiciones del teorema anterior y que gi son funciones difer-
enciables en x, i I(x), entonces existen unicos 1 , . . . , m R y i 0 para todo
i = 1, . . . , p tal que
m p
X X
f (x ) + i hi (x ) + i gi (x ) = 0.
i=1 i=1

Observacion 4.19 El corolario anterior nos da un metodo para encontrar los puntos
candidatos a solucion del problema (PNL). Un punto candidato a solucion es un punto
x tal que para algunos escalares (, ) se satisface
m p
X X
f (x) + i hi (x) + i gi (x) = 0 (4.24)
i=1 i=1
h(x) = 0 (4.25)

i gi (x) = 0, i = 1, . . . , m (4.26)

i 0, i = 1, . . . , m

gi (x) 0, j = 1, . . . , q

Las n + m + p ecuaciones (4.24) - (4.26) forman un sistema no lineal en las incognitas


x Rn , Rm y Rp . Los puntos que resuelven este sistema son llamados puntos
estacionarios de (P N L).

Teorema 4.17 Si x es un punto regular y minimizador local de (P N L), A es la matriz


cuyas filas son los gradientes de las restricciones activas en x excluyendo los gradientes
de aquellas restricciones de desigualdad cuyo multiplicador es cero, entonces

y T 2xx l(x , , )y 0 para todo y N (A),

donde y son los vectores de multiplicadores de Lagrange dados en el Teorema 4.16 y


m p
X X
l(x, , ) = f (x) + i hi (x) + i gi (x).
i=1 i=1

Teorema 4.18 Si existen (x , , ) satisfaciendo la condicion necesaria de primer or-


den para (P N L) y ademas
y T 2xx l(x , , )y > 0,

85
para todo y N (A), y 6= 0, donde A es la matriz cuyas filas son los gradientes de las
restricciones activas en x excluyendo los gradientes de aquellas restricciones de desigual-
dad cuyo multiplicador es cero, entonces x es minimizador local estricto del problema
(P N L).

Ejemplo 4.15 Considere el problema

1 1
min{f (x1 , x2 , x3 ) = x21 + x22 + x23 : x21 + x22 + x23 1, x1 + x2 + x3 = 1/2}
2 3

El problema tiene solucion pues f es continua y el conjunto que define las restricciones
es compacto. Hallaremos la solucion usando las condiciones de optimalidad de primer y
segundo orden. La funcion de Lagrange es

1 1
l(x1 , x2 , x3 , 1 , 2 ) = x21 + x22 + x23 + 1 (x1 + x2 + x3 1/2) + 1 (x21 + x22 + x23 1)
2 3

El sistema de primer orden que debe satisfacer el punto de mnimo son






2x1 (1 + 1 ) + 1 = 0





x2 (1 + 21 ) + 1 = 0







x3 ((2/3) + 21 ) + 1 = 0


x1 + x2 + x3 = 1/2



2

x1 + x22 + x23 1.







1 (x21 + x22 + x23 1) = 0




0.
1

Resolviendo el sistema obtenemos que el unico punto candidato a solucion es x =


(1/2, 1/6, 1/4) con 1 = 1/6 y 1 = 0. Analizaremos ahora la condicion de segundo
orden.

De las dos restricciones, tenemos que la unica restriccion activa para el punto crtico
es h(x) = x1 + x2 + x3 1/2 y por lo tanto A = h(x)T = (1 1 1). y as

N (A) = {v R3 : v1 + v2 + v3 = 0}

86
La matriz hessiana de la funcion de Lagrange es

2 + 21 0 0


2xx l(x1 , x2 , x3 , 1 , 1 ) = 0 1 + 21 0

2
0 0 3
+ 21

Evaluando en el punto x = (1/2, 1/6, 1/4) con 1 = 1/6 y 1 = 0, tenemos



2 0 0


2xx l(x, 1 , 1 ) = 0 1 0 ,

2
0 0 3

que es una matriz definida positiva en todo R3 y as en particular sobre N (A). Por lo
tanto x == (1/2, 1/6, 1/4) es un mnimo local estricto.

Ejemplo 4.16 Considere el problema

min{f (x1 , x2 ) = 5x21 2x1 x2 + 5x22 : x1 + x2 = 1, x1 0, x2 0}

Hallaremos los puntos de mnimo. La funcion de Lagrange es

l(x1 , x2 , 1 , 1 , 2 ) = 5x21 2x1 x2 + 5x22 + 1 (x1 + x2 1) 1 x1 2 x2 ,

y las condiciones de KKT son




10x1 2x2 + 1 1 = 0







2x1 + 10x2 + 1 2 = 0







x1 + x2 = 1





x 1 0



x2 0





1 x1 = 0







2 x2 = 0





1 0





0
2

87
Resolviendo el sistema tenemos que el unico punto candidato a solucion es x =
(1/2, 1/2) con 1 = 4, 1 = 0 y 2 = 0. Analicemos la condicion de segundo orden. La
matriz hessiana de la funcion de Lagrange es

10 2
2xx l(x, 1 , 1 , 2 ) =
2 10

Observamos que la unica restriccion activa es h(x) = x1 + x2 1 entonces tenemos


que la matriz A = h(x)T = (1 1), luego N (A) = {v R2 : v1 + v2 = 0}. Ahora, sea
v N (A), v 6= 0, de aqu tenemos que

  10 2 v
v T 2xx lv = v1 v2 1 = 24v12 > 0,
2 10 v2

por lo tanto x = (1/2, 1/2) es un punto mnimo global estricto.

4.6 Ejercicios Resueltos


Ejercicio 4.1 Considere el problema
 
x22 x23 2
min f (x1 , x2 , x3 ) = x1 + + + : x1 , x2 , x3 > 0
4x1 x2 x3

Probar que f es coerciva en el conjunto de restricciones y encontrar las soluciones


globales del problema.

Solucion. Sea X = {(x1 , x2 , x3 ) : xi > 0, para todo i = 1, 2, 3}, entonces el problema es




min f (x)


s.a:



xX

Probemos que f es coerciva. Por contradiccion, supongamos que f no es coerciva,


entonces existe una sucecion crtica {xk } X tal que limk f (xk ) 6= +, esto es,
existe c > 0 siempre que f (xk ) c, para todo k IN . Usando la denicion de f tenemos

(xk2 )2 (xk3 )2 2
xk1 + k
+ k + k c. (4.27)
4x1 x2 x3

88
Entonces, {xk } es limitada (ya que cada 0 < xki c) y como {xk } es crtica, entonces
existe x FrontX tal que xk x = (x1 , x2 , x3 ). Luego, existe i {1, 2, 3} tal que
xi = 0, lo que implica, xki 0, y de (4.27) tenemos que + c, que contradice la
suposicion, por lo tanto f es coerciva en X.
Como f es coerciva y continua en X entonces por Corolario 3.4 existe un mnimo global.
Hallemos los puntos candidatos a solucion (f (x) = 0). Derivando con respecto a las
variables x1 , x2 , x3 tenemos

f x2
= 1 22 = 0,
x1 4x1
f x2 x3
= = 0,
x2 2x1 x2
f 2x3 2
= 2 = 0.
x3 x2 x3

De la primera, segunda y tercera ecuacion obtenemos respectivamente


 2
x2
= 4. (4.28)
x1
 2
x2 x3
= (4.29)
2x1 x2
x3 1
= 2 (4.30)
x2 x3

De (4.28) y considerando que x1 , x2 > 0, tenemos

x2
=2 (4.31)
x1
 2
x3
Sustituyendo la ecuacion (4.31) en (4.29) da 1 = x2
, lo que implica que

x3 = x2 . (4.32)

Sustituyendo (4.32) en (4.30) tenemos x23 = 1 y as x3 = 1. Sustituyendo en (4.32) y


(4.31) obtenemos que el unico punto candidato es x = ( 21 , 1, 1) y como existe un mnimo
global este es el unico mnimo global del problema.

Ejercicio 4.2 Demostrar que la funcion f (x1 , x2 ) = (x2 x21 )2 + x51 tiene un unico punto
estacionario que no es maximizador ni minimizador de f en Rn .

89
Solucion.

f
i) Hallemos los puntos estacionarios. Las derivadas parciales son x1
= 2(x2
f
x21 )(2x1 ) + 4x41 = 4x1 (x2 x21 ) + 4x41 y x2
= 2(x2 x21 )(1) = 2(x2 x21 ).
Para hallar los puntos crticos formamos la ecuacion

4x (x x2 ) + 4x4 = 0
1 2 1 1
(x x2 ) = 0
2 1

Se verica facilmente que el unico punto que resuelve las ecuaciones es x = (0, 0).
As, la funcion f (x1 , x2 ) = (x2 x21 )2 + x51 tiene un unico punto estacionario.

ii) Veamos si x es un punto de mnimo, maximo o punto de silla. Las derivadas de


2f 2f 2f
segundo orden son x21
= 4(x2 x21 )+(4x1 )(2x1 )+16x31 , x22
= 2, x2 x1
= 4x1
2f
y x1 x2
= 4x1 . De aqu tenemos que

0 0
2 f (0, 0) =
0 2

Vemos que la matriz no es semidenida negativa concluyendo que x = (0, 0) no es


punto de maximo ni local ni global. Tambien, como esta matriz es solo semidenida
positiva el criterio de segundo orden no nos dice nada del punto obtenido.
Por otro lado, observemos que en cualquier vecindad de x = (0, 0) existen puntos donde
los valores de la funcion son negativos, tome por ejemplo x1 < 0 y x2 = x21 . Por lo tanto
el punto x = (0, 0) no es tampoco mnimo local ni global. Concluimos por tanto que
x = (0, 0) no es maximizador ni minimizador de f en Rn .

Ejercicio 4.3 Estudie la existencia de soluciones del problema


opt f (x1 , x2 ) = x21 x22 : (x1 , x2 ) R2

Solucion. Es facil ver que el punto crtico es x = (0, 0). Ademas, en cualquier vecindad
de x podemos tomar x1 = 0 y x2 6= 0 obteniendo f (0, x2 ) < 0, as como tambien, x1 6= 0
y x2 = 0 obteniendo f (x1 , 0) > 0. Por lo tanto x = (0, 0) no tiene mnimo ni maximo.

90
Ejercicio 4.4 Encontrar todos los valores del parametro R para los cuales el problema
de minimizacion irrestricta de la funcion f (x1 , x2 ) = x21 + 4x1 x2 + 4x22 + x1 + x2 , tiene
solucion local.

Solucion.

i) Hallemos los puntos crticos usando f (x) = 0.



f = 2x + 4x + 1 = 0
x1 1 2
f = 4x + 8x + 1 = 0
x2 1 2

1 1
Sumando las dos ecuaciones tenemos (2 2)x1 + 2
= 0 y de aqu, x1 = 4(1) .
2
Sustituyendo en la primera ecuacion tenemos que x2 = 8
. Luego los puntos
 
1
crticos son (x1 , x2 ) = 4(1) , 2
8
, 6= 1.

2f 2
ii) Calculemos la hessiana de f . Derivando parcialmente obtenemos x21
= 2, x1 x
f
2
=
2 2
4, x2 x
f
1
= 4, xf2 = 8. Luego
2


2 4
2 f (x) =
4 8

2 f (x) 0 > 0 16 16 > 0

>01>0

Luego, los valores para los cuales el parametro hace que la funcion tenga un
mnimo es (1, +).

Ejercicio 4.5 Sea f : (a, b) R, a, b R, continua en (a, b). Pruebe que, si x es un


mnimo local de f entonces f (x) = 0 o f (x) no existe.

Solucion. Si f derivable en x entonces por la condicion necesaria de primer orden dada


en el Teorema 4.1 se tiene que f (x) = 0. Si f no es derivable entonces no existe f (x).

P
n1
Ejercicio 4.6 Sea n 2 y f (x1 , x2 ) = (1 + xn )3 x2i + x2n . Muestre que
i=1

i) x = 0 es el unico punto estacionario de f en Rn .

91
ii) x = 0 es un mnimo local estricto pero no es mnimo global.

Solucion.

i) Hallemos los puntos crticos


f
= 2(1 + xn )3 xi = 0, para todo i = 1, . . . (n 1). (4.33)
xi
X n1
f
= 3(1 + xn )2 x2i + 2xn = 0. (4.34)
xn i=1

De (4.33) tenemos (1 + xn ) = 0 xi = 0, para todo i = 1, . . . , n 1, esto es,


xn = 1 xi = 0, para todo i = 1, . . . , n 1. Si xn = 1, entonces de (4.34)
tenemos que xn = 1 = 0, lo que es una contradiccion. Por lo tanto, xi = 0,
para todo i = 1, . . . , n 1. Sustituyendo en (4.34) resulta que xn = 0, luego
x = (0, 0, . . . , 0) es un punto crtico.

ii) Probemos que x es un mnimo local estricto. Como


n1
X
2f 3 2f 2 2f
= 2(1 + x n ) , = 6(1 + x n ) x i , = 6(1 + x n ) x2i + 2,
x2i xi xn x2n i=1

tenemos que
2 0 0 0


0 2 0 0
2 f (x) =
...
0.



0 0 0 2
Usando la condicion suciente de segundo orden dado por el Teorema 4.2 tenemos
que x = 0 es mnimo local estricto con valor objetivo f (0) = 0. Ahora probemos que
x no es mnimo global. Consideremos el caso n = 2, entonces tenemos f (x1 , x2 ) =
(1 + x2 )3 x21 + x22 , aqu tomamos los puntos x1 = 4, y x2 = 2, as tenemos que
f (4, 2) = 16 + 4 = 12 < 0 = f (0), as x = 0 no es mnimo global.

Ejercicio 4.7 Sea A = AT y x un punto crtico del problema






min 12 hA x, xi hb, xi + c


s.a.




x Rn

92
Pruebe que:

1. Si A  0 entonces x es un mnimo global de f .

2. Si A 0 entonces x es el unico mnimo global de f .

Solucion. Como x es un punto crtico entonces por el Teorema 4.1

f (x ) = A x b = 0

luego, para cada x Rn

1
f (x) = hAx, xi hb, xi + c
2
1
= hA(x x ), x x i + f (x )
2

As tenemos para cada x Rn

1
f (x) = f (x ) + hA (x x ), x x i
2

1. Si A  0 entonces f (x) f (x ), pata todo x Rn .

2. Si A 0 entonces f (x) > f (x ), para todo x 6= x . Por lo tanto, x es el unico


mnimo global.

Ejercicio 4.8 En el problema anterior, muestre que si x es un mnimo local entonces


Ax = b y A  0.

Solucion. Solo usar las condiciones de optimalidad dado por los teoremas 4.1 y 4.2.

Ejercicio 4.9 Sea f : Rn R una funcion diferenciable en x. Muestre que si f (x) 6= 0


entonces x no es mnimo ni maximo local de f sobre Rn .

Solucion. Usar la negacion del Teorema 4.1.

Ejercicio 4.10 Demostrar que la funcion f (x, y) = (1+ex ) cos xyey tiene una infinidad
de maximos pero no tiene mnimos.

93
Solucion. Hallemos los puntos estacionarios de f

f
= (1 + ey )senx = 0 (4.35)
x
f
= ey cosx ey yey = ey (cosx 1 y) = 0 (4.36)
y
De (4.35) tenemos que sin x = 0, y del resultado anterior cumple para todos los x tal
que x = k, para todo k ZZ (conjunto de numeros enteros). Reemplazando en (4.36)
tenemos que para todo k ZZ : y = cosk 1, esto implica que


0, si k es par o k = 0
y=

2, si k es impar

Entonces los puntos estacionarios son




(k, 0), si k = 0 o k es par
(x, y) =

(k, 2), si k es impar

Hallemos la matriz 2 f

2f 2f
a). x2
= (1 + ey )cosx c). yx
= sin xey

2f 2f
b). xy
= sin xey d). y 2
= ey (cosx 2 y)

Evaluando en los puntos estacionarios tenemos, si k=0 o k es par

2f
x2
(k, 0) = (1 + e0 ) = 2

2f
yx
(k, 0) = senke0 = 0

2f
xy
(k, 0) = senke0 = 0

2f
y 2
(k, 0) = e0 (cosk 2 0) = 1

As, para el caso donde k = 0 o k es par la matriz hessiana es



2 0
2 f (x, y) =
0 1

94
Si k es impar

2f 1
x2
(k, 2) = (1 + e2 )cosk = (1 + e2 ) = 1 + e2

2f
yx
(k, 2) = senke2 = 0

2f
xy
(k, 2) = senke2 = 0

2f 3 y
y 2
(k, 2) = e2 (cosk 2 y) = e2 (3 y) = e2
e2
.

Entonces, si k es impar tenemos



1
1 e2
0
2 f (k, 0) =
3 y
0 e2
e2

En ambos casos para k tenemos que la matriz obtenida es denida negativa (pues sus
autovalores que son las diagonales son negativas), por lo tanto todos los puntos crticos


(k, 0), si k = 0 o k es par
(x, y) =

(k, 2), si k es impar

son puntos de maximo y como k puede tomar cualquier numero entero entonces tenemos
una innidad de puntos de maximo. Ahora supongamos que existe un punto (x0 , y0 ) de
mnimo local o global, entonces por la condicion de segundo orden, Teorema 4.2, se debe
tener que la matriz hessiana en este punto es semidenida positiva, lo que es imposible
ya que todas las matrices Hesianas en los puntos crticos son denidas negativas.

Ejercicio 4.11 Encontrar los mnimos y maximos de




x31

opt + x2

3
s.a.




(x1 , x2 ) X = {(x1 , x2 ) : x21 + x22 = 2

Solucion.

x31
i) Probemos la existencia, f (x1 , x2 ) = 3
+ x2 es continua en Rn , en particular en X,
y X es compacto entonces, existe puntos de mnimo y maximo global.

95
ii) f, h C (Rn ), donde h(x1 , x2 ) = x21 + x22 2; ademas

2x
1
h(x1 , x2 ) =
2x
2

es l.i para todo (x1 , x2 ) X. Por tanto todo punto de mnimo o maximo es un
punto crtico.

iii) Hallemos los puntos crticos. Siguiendo la observacion 4.13 formamos el siguiente
sistema

x L(x1 , x2 , ) = 0

x2 + x2 2 = 0.
1 2

x31
donde L(x1 , x2 , ) = 3
+ x2 + (x21 + x22 2). Hallando el gradiente de L tenemos
que el sistema anterior queda establecido por

x21 + 2x1 = 0 (4.37)

1 + 2x2 = 0 (4.38)

x21 + x22 2 = 0 (4.39)

De (4.37) tenemos que x1 (x1 + 2) = 0 si y solo si x1 = 0 x1 = 2. Ahora, si



x1 = 0, de (4.39) tenemos x2 = 2 y sustituyendo en (4.37) resulta

1 1 1 2
= = = =
2x2 2( 2) 2 2 4

2
=

2
Luego x = (0, 2), con = 4
y x = (0, 2), con 4
. Por otro lado,
1
si x1 = 2, de (4.38) tenemos x2 = 2 . Sustituyendo en (4.39) obtenemos
= 12 . Luego x1 = 2( 12 ) = 1 y tambien x2 = 2
1
= 12 ( 11 ) = 1. As
2

e = , x
1 e e
e = . As tenemos que los
1
tenemos, x e = (1, 1), con 2
e = (1, 1) y 2

puntos crticos son



(a) x = (0, 2) con = 22 , entonces f (x) = 2 = 1.414.

(b) x = (0, 2) con = 22 , entonces f (x) = 2 = 1.414.
e = 1 , entonces f (e
e = (1, 1) con
(c) x x) = 31 1 = 34 = 1.333 . . ..
2

96
e e
e = (1, 1) con e
e = 1 , entonces f (x 1 4
(d) 2
e) = 3
+1= 3
= 1.333 . . ..

Comparando los valores objetivos, y sabiendo por i) que los valores optimos globales

existen, tenemos que x = (0, 2) es un maximo global y x = (0, 2) es un mnimo
global.

iv) Evaluaremos las condiciones de 2do orden para saber si los demas puntos son
optimos locales.
2x1 + 2 0
2xx L(x, ) =
0 2
Ker(h (x)) = {(d1 , d2 )/h (x)d = 0}

d1
= (d1 , d2 ) : 2(x 1, x2 ) =0
d2

= {(d1 , d2 ) : x1 d1 + x2 d2 = 0}

Para x
e = (1, 1) tenemos

Ker(h (1, 1)) = {(d1 , d2 ) : d1 d2 = 0} = {(d1 , d2 ) : d2 = d1 }

Luego, para todo d Ker(h (1, 1)), tenemos



  2(1) + 2(1/2) 0 d
h2xx L(e e di = d d
x, )d, 1 1
1
0 2(1/2) d1

  1 0 d1
= d1 d1 = d21 + d21 = 0
0 1 d1
Luego x
e solo satisface la condicion necesaria de segundo orden. As el criterio
no nos dice nada si es un mnimo o maximo local.
e
Para x
e = (1, 1) tenemos

  2 + 2(1/2) 0 d1
e e
e di = =0
h2xx L(x
e, )d, d1 d1
0 2(1/2) d2

e
Luego x
e solo satisface la condicion necesaria de segun orden, por lo tanto el
criterio no nos da mas informacion.

97
Ejercicio 4.12 Sea p = (p1 , p2 , p3 ) R3 y L = {a + td/ t R} una recta en R3 donde
a = (a1 , a2 , a3 ), (d1 , d2 , d3 ) R3 . Si p 6 L, entonces el problema de la mnima distancia
de p a L puede ser formulado con




min kx pk2


s.a.




x L

Responda la siguiente pregunta, existe la solucion del problema? Si no existe justifique.


Caso contrario, hallar el punto de mnimo.

Solucion. El problema tiene solucion pues L es un conjunto cerrado y as podemos usar


el resultado dado en el ejemplo 3.8. Hallemos el punto de mnimo. x = (x1 , x2 , x3 ) L
x = a + td, para algun t R, as podemos expresar




x1 = a1 + td1


x2 = a2 + td2




x3 = a3 + td3

Luego, el problema puede ser formulado por






min (x1 p1 )2 + (x2 p2 )2 + (x3 p3 )2





s.a.



x1 = a1 + td1





x2 = a2 + td2





x3 = a3 + td3

Sustituyendo el valor de x




min (a1 + td1 p1 )2 + (a2 + td2 p2 )2 + (a3 + td3 p3 )2


s.a.




tR

98
As obtenemos un problema irrestricto de una sola variable, donde

f (t) = (a1 + td1 p1 )2 + (a2 + td2 p2 )2 + (a3 + td3 p3 )2

Hallemos los puntos crticos

f (t) = 2(a1 + td1 p1 )d1 + 2(a2 + td2 p2 )d2 + 2(a3 + td3 p3 )d3 = 0

Aplicando la propiedad distributiva y asosiando convenientemente tenemos

a1 d1 + td21 p1 d1 + a2 d2 + td22 p2 d2 + a3 d3 + td23 p3 d3 = 0

t(d21 + d22 + d23 ) + (a1 d1 + a2 d2 + a3 d3 ) (p1 d1 + p2 d2 + p3 d3 ) = 0

tkdk2 + ha p, di = 0
ha p, di
t=
kdk2
Luego el unico punto de mnimo del problema es
 
hp a, di
x = a + d
kdk2

Ejercicio 4.13 Sea A Rnn una matriz simetrica. Pruebe que los puntos crticos del
problema



opt 12 hAx, xi


s.a :




kxk = 1

son los autovectores de la matriz A. Cuales son los autovalores?

Solucion.

i) f (x) = 12 hAx, xi es continua y X = {x Rn : kxk = 1} es compacto, entonces


existen puntos de mnimo y maximo globales.
p
ii) f y h(x) = kxk 1 = hx, xi 1 son diferenciables en Rn . Ademas,
1 2x x
h(x) = p =p = x, x X
2 hx, xi hx, xi
luego todo punto de mnimo y maximo es punto de silla.

99
iii) Hallemos los puntos crticos, L(x, ) = hAx, xi + (kxk 1). Los puntos crticos
son obtenidos resolviendo

x L(x, ) = 0

h(x) = 0

esto es, Ax + x = 0 y kxk = 1. Entonces se tiene Ax = x. Denotando


hx,xi
= , obtenemos que los puntos crticos satisfacen Ax = x. As, los puntos
crticos son los autovectores de la matriz A y los autovalores son el inverso aditivo
de los multiplicadores de Lagrange.

Ejercicio 4.14 Considerando que la funcion de utilidad de un comerciante es


1
f (x, y) = 18 x21 x22
2
donde x1 e x2 representa las cantidades de carne y verduras que vende (en kilos). Supong-
amos que el capital del comerciante es de 12 soles (el cual esta obligado a inertirlo total-
mente en los dos productos) y los precios de x1 y x2 son 2 y 3 soles respectivamente.

a) Formular el problema de optimizacion.

b) Resolver el problema.

Solucion.

a) La formulacion del modelo es


1
max{18 x21 x22 : 2x1 + 3x2 = 12, x1 > 0, x2 > 0}
2

b) El problema dado es equivalente a


1
min{18 + x21 + x22 : 2x1 + 3x2 = 12, x1 > 0, x2 > 0}
2
i) Hallemos el lagrangiano,
x21
L(x1 , x2 , s, p, q) = 18 + + x22 + s(2x1 + 3x2 12) px1 qx2
2
de donde
x1 + 2s p
x L = .
2x2 + 3s q

100
El sistema queda establecido como




x1 + 2s p = 0,





2x2 + 3s q = 0,







2x1 + 3x2 = 12,





x1 > 0,



x2 > 0,





px1 = 0,







qx2 = 0,





p, q 0

Como x e y son positivos, entonces p = 0 y q = 0 luego el sistema se reduce a






x1 + 2s = 0,






2x2 + 3s = 0,


2x1 + 3x2 = 12,





x1 > 0,





x2 > 0,

De la primera y segunda ecuacion tenemos que x2 = (3/4)x1 . Sustituyendo en


48 36
la tercera ecuacion obtenemos x1 = 17
y x2 = 17
. Reemplazando en la primera
24 48 36
ecuacion da s = 17 . Luego el unico punto crtico es (x1 , x2 ) = ( 17 , 17 ) con
s = 24
17
.

ii) Hallemos la matriz Hessiana de la funcion de Lagrange. Derivando parcial-


mente L obtenemos
1 0
2xx L =
0 2
Esta matriz es denida positiva en todo R2 , en particular en el Ker(h (x, y)),
luego usando el Teorema 4.12 obtenemos que el punto es mnimo local estricto.
Ademas como la funcion objetivo es convexa, entonces tal mnimo es global y
por lo tanto solucion del problema de optimizacion.

101
Ejercicio 4.15 Si f (x) = 12 xT Ax bT x + c, donde A Rnn , A = AT y A  0. Pruebe
o de un contraejemplo de la siguiente proposicion f tiene un mnimo globbal sobre Rn

Solucion. La proposicion es falsa. En efecto, considere



1 1
A=
1 1
y
2
b=
3
Supongamos que existe un mnimo global x = (x1 , x2 ) R2 , por la condicion necesaria
de primer orden se tiene que Ax = b, lo cual equivale a

1 1 x 2
1 =
1 1 x2 3

de aqu se deduce que x1 + x2 = 2 y x1 + x2 = 3 pero esto es, 2 = 3, lo que es una


contradiccion.

Ejercicio 4.16 Considere el problema

min{|| Ax b ||2 : x Rn }

donde A Rmn , m n, b Rm , rango de A = n.

a) Pruebe que el problema tiene solucion global.

b) Escribe la condicion necesaria de primer orden de optimalidad.Es una condicion


suficiente?.

c) Encuentre la solucion optima.

Solucion.

a) La funcion f se puede expresar como:

f (x) = ||Ax b||2 = hAx b, Ax bi





= AT Ax, x 2 AT b, x + ||b||2 .

102
As f puede ser expresada como f (x) = xT (AT A)x 2(AT b)T x + ||b||2 .
Sea B = AT A y b = AT b entonces, f (x) = xT Bx 2(b )T x + ||b||2 . Como
B T = (AT A)T = AT A = B (B es simetrica) y para todo z 6= 0 z T Bz = z T (AT A)z
= z T (AT A)1/2 (AT A)1/2 z = ((AT A)1/2 z)((AT A)1/2 z) > 0 (B es denida positiva),
concluimos que este problema es un caso particular del Ejemplo 3.7. Por lo tanto,
f es coerciva y as el problema tiene solucion global.

Ejercicio 4.17 Considere el siguiente problema

1
max{18 x2 y 2 : 2x + 3y 12, x 0, y 0}
2

i) El problema tiene solucion?

ii) Hallar las condiciones de KKT.

iii) Si existe la solucion del problema, hallarla.

Solucion.

i) El problema es equivalente a

1
min{ x2 + y 2 18 : 2x + 3y 12, x 0, y 0}
2

La funcion f (x) = 12 x2 + y 2 18 es continua y

X = {(x, y) : 2x + 3y 12, x 0, y 0}

es compacto, entonces el problema tiene solucion.

ii) L(x, y, s1 , s2 , s3 ) = 21 x2 + y 2 18 + s1 (2x + 3y 12) s2 x s3 y, as



x + 2s1 s2
(x,y) L(x, y, s1 , s2 , s3 ) =
2y + 3s1 s3

103
Las condiciones de KKT son


x + 2s1 s2 = 0,





2y + 3s1 s3 = 0,





2x + 3y 12,





L(x, s) = 0
x 0,

x

viabilidad y0






s0
s1 (2x + 3y 12) = 0,





s2 x = 0,





s3 y = 0,



s1 , s2 , s3 0.

iii) Debemos resolver



x + 2s1 s2 = 0





2y + 3s1 s3 = 0





2x + 3y 12,


s2 x = 0,





s3 y = 0,





s1 (2x + 3y 12) = 0,



x, y, s , s , s 0.
1 2 3

de donde, s2 x = 0 si y solo si s2 = 0 x = 0.

Si s2 = 0 entonces x + 2s1 = 0, como x 0 y s1 0

x = 0 s1 = 0.

2y = s3 .

y4

2y.y = 0

y=0

Por tanto, el punto (0, 0, 0, 0, 0) satisface las condiciones de KKT.

Si x = 0 entonces 2s1 = s2

s3 y = 0 s3 = 0 y = 0

104
Si y = 0 3s1 = s3

s1 = 0

Luego, el punto es(0, 0, 0, 0, 0).

Si s3 = 0 2y + 3s1 = 0

y = 0 s1 = 0

s1 = 0

(0, 0, 0, 0, 0) satisface las condiciones de KKT.

Luego, el unico punto que satisface las condiciones de KKT es (0, 0), y usando el
tem i, concluimos que (0, 0) es el optimo del problema.

Ejercicio 4.18 Hallar los puntos crticos del problema

1
min{cT x + xT Bx : Ax = b},
2

donde c Rn , B Rnn simetrica, invertible y A Rmn de rango m < n.

Solucion.

i) La funcion de Lagrange es: L(x, ) = cT x + 21 xT Bx T (Ax b), y como B es



simetrica entonces x 21 xT Bx = Bx, luego tenemos que x L(x, ) = c + Bx
AT .

ii) Las condiciones de KKT son




x L(x, ) = 0,

Ax = b.

de donde se deduce

AT Bx = c (4.40)

Ax = b, (4.41)

105
Multiplicando (4.40) por B 1 y por A, usando (4.41) resulta AB 1 AT b = AB 1 c.
Como A es de rango m entonces AB 1 AT es invertible, as se tiene

= (AB 1 AT )1 (b + AB 1 c). (4.42)

De (4.40) tenemos que x = B 1 AT B 1 c, y de la ecuacion (4.42), tenemos


x = B 1 AT (AB 1 AT )1 b + BAT (AB 1 AT )1 AB 1 c B 1 c. Por lo tanto, el unico
punto crtico es x = B 1 AT (AB 1 AT )1 (b + AB 1 c) B 1 c.

Ejercicio 4.19 hallar las condiciones de optimalidad de KKT del siguiente problema
m
X
min{f (x) ln(gi (x)) : gi (x) 0, hj (x) = 0}
i=1

donde f, gi , i = 1, . . . , m y hj , j = 1, . . . , p son funciones diferenciables sobre Rn y > 0


es un parametro fijo.

Solucion. Por la naturaleza del problema, este es equivalente a


m
X
min{f (x) ln(gi (x)) : hj (x) = 0}
i=1
Pm Pp
Formamos la funcion lagrangiana, L(x, ) = f (x) i=1 ln(gi (x)) j=1 j hj (x).

i) Hallamos el gradiente de la funcion de Lagrange


m
!
X
x L(x, ) = f (x) x ln(gi (x)) T h(x). (4.43)
i=1
Pm
Sea (x) = i=1 ln(gi (x)) = ln g1 (x) + ln g2 (x) + + ln gn (x), entonces
g1 (x) g2 (x) gm (x)
(x)
= xi + xi + + xi .
xi g1 (x) g2 (x) gm (x)

As,
m
! m
X g1 (x) g2 (x) gm (x) X gi (x)
x ln gi (x) = (x) = + + ... + =
i=1
g1 (x) g2 (x) gm (x) i=1
gi (x)

Sustituyendo esta expresion en (4.43) tenemos


m
X gi (x)
x L(x, ) = f (x) T h(x).
i=1
gi (x)

106
ii) Luego, las condiciones de KKT son



P

x L(x, ) = 0
f (x) mi=1
gi (x)
T h(x) = 0


gi (x)

h(x) = 0 h(x) = 0







gi (x) > 0
gi (x) > 0.

4.7 Ejercicios Propuestos


1. Considere el problema

1
min{(x1 )2 + (x2 5)2 : 2x1 + x2 6; x1 + x2 4; x1 0; x2 0}
2

. Obtenga los puntos candidatos a solucion.

107
Captulo 5

Elementos de Convexidad

En este captulo presentaremos los elementos basicos de convexidad que nos permitiran
abordar posteriormente los problemas de optimizacion convexa que tienen un gran campo
de aplicaciones en diferentes areas de las ciencias e ingenieras.

5.1 Elementos de Convexidad


Definicion 5.1 Un conjunto C Rn es llamado convexo si C = o para todo x, y C
y todo [0, 1] se tiene x + (1 )y C. Un conjunto A Rn es llamado afin, si
para todo x, y A y todo t R, tx + (1 t)y A. Un subconjunto K Rn es un cono
si, para todo d K y t 0, td K.

y
x

Figura 5.1: Graca de un conjunto convexo.

Ejemplo 5.1 Ejemplos de conjuntos convexos son:

108
C
x

Figura 5.2: Graca de un conjunto que no es convexo.

El espacio euclidiano Rn .

La bola abierta B(x, ), donde x Rn y > 0.

El ortante no negativo Rn+ .

El semiespacio {x Rn : Ax b}, donde A Rmn , b Rm .

El hiperplano {x Rn : ha, xi = }, donde a Rn y R.

Ejemplos de conjuntos que no son convexos son:

Rn \{0}.

La esfera unitaria S(0, 1) = {x Rn : ||x|| = 1}.

Ejemplo 5.2 Las rectas, los planos son espacios afines.

Ejemplo 5.3 Rn+ , S+n son conos convexos.

Definicion 5.2 Dado X Rn , la cerradura conica de X, denotado por cono(X) es el


menor cono que contiene a X.

Proposicion 5.1 Si X es convexo entonces cono(X) = {x : x X; 0}.

Prueba.

Definicion 5.3 Un punto x Rn es una combinacion convexa de puntos {x1 , . . . , xp } si


P P
existen i 0, i = 1, . . . , p, tal que pi=1 i = 1 y se satisface x = pi=1 i xi .

109
R2+

Figura 5.3: R2+ es conjunto convexo.

Figura 5.4: R2+ R2+ no es un conjunto convexo.

Teorema 5.1 Un subconjunto C Rn es convexo si y solamente si para cualquier


P P
numero p IN , xi C, i 0, i = 1, 2, . . . , p, con pi=1 i = 1, se tiene pi=1 i xi C.

Demostracion. Sea C convexo, probaremos por induccion que la combinacion convexa


esta contenida en C. Si p = 1, el resultado es inmediato. Supongamos que la hipotesis
es verdadera para p = n (hipotesis inductiva), probaremos que es valida para p = n + 1.
Pn+1
Veamos; sea x1 , x2 , . . . , xn , xn+1 C y 1 , 2 , . . . , n , n+1 0, tal que i=1 i = 1,
Pn+1 i
P n i n+1
entonces i=1 i x = i=1 i x + n+1 x . Si n+1 = 1 entonces el resultado es ver-
dadero (similar al caso p = 1).

Sea n+1 6= 1, esto es, 1 n+1 6= 0, entonces


n+1
X n 
X 
i i
i x = (1 n+1 ) xi + n+1 xn+1 (5.1)
i=1 i=1
1 n+1

110
Pn+1 Pn Pn  i

Como i=1 i = 1, se tiene, i=1 i = 1 n+1 , esto es, i=1 1n+1
= 1, donde
i
la relacion 1n+1
0, para todo i = 1, 2, . . . , n.

Pn  i 
Como x1 , x2 , . . . , xn C y i=1 1n+1 = 1. Aplicando hipotesis inductiva,
Pn  
i i
i=1 1n+1 x C. Usando este resultado en la ecuacion (5.1) y por la hipotesis
Pn+1 i
general (convexidad de C) tenemos que i=1 i x C. El recproco es inmediato

tomando en particular p = 2.

Teorema 5.2 (de Caratheodory) Si x Rn es una combinacion convexa de puntos


de X Rn , entonces existen xi X y i R+ , i = 1, 2, . . . , n + 1, satisfaciendo
Pn+1 Pn+1 i
i=1 i = 1 y x = i=1 i x .

Demostracion. Si x es una combinacion convexa de X, entonces existe p IN tal que


p p
X X
i
X= i x , i = 1.
i=1 i=1

Mostraremos que si p > n + 1, entonces podemos eliminar un punto de la combi-


nacion convexa. Si existe i0 {1 , . . . , p }, tal que i0 = 0, entonces podemos elim-
inar el punto xi0 obteniendo el objetivo deseado. Sea i > 0, para todo i = 1, . . . , p.
Como p > n + 1 entonces p 1 > n, por un resultado de algebra lineal, el conjunto
{x1 xp , x2 xp , . . . , xp1 xp } es linealmente dependiente, entonces existen {1 , . . . , p1 }
Pp1 i p
no todos nulos tal que i=1 i (x x ) = 0.

Pp1 Pp
Sea p = i=1 i , entonces i=1 i = 0. Ademas,
p p1
X X
i
i x = i (xi xp ) = 0. (5.2)
i=1 i=1
o n
j0 k
Sea j0 {1, 2, . . . , p} tal que = max1kp
j0
. Como existe i 6= 0 entonces,
k
j0 Pn
si i > 0, tenemos j > 0, caso contrario, si i < 0, de la igualdad i=1 i = 0, existe
0
j0
j > 0 y as j0
> 0. De ambos casos tenemos

j0
> 0.
j0

111
j0
Denamos qi = i ,
j0 i
para todo i = 1, . . . , p. De aqu tenemos que qi 0 y
qj0 = 0, ademas,
p p p
X X j0 X
qi = i i = 1.
i=1 i=1
j0 i=1
Luego,
p p p p
X X j0 X X
i i i0
x= i x = qi x + i x = qi xi ,
i=1 i=1
j0 i=1 i=1

donde la ultima igualdad se obtiene de (5.2). Como qj0 = 0, entonces concluimos que
Pp i
x = i=1 qi x . As, x es una combinacion convexa de p 1 puntos. Repitiendo el
i6=j0
proceso p (n + 1) veces obtenemos la combinacion convexa de n + 1 puntos.

Definicion 5.4 Sea X Rn un conjunto arbitrario. La envoltura o cerradura convexa


de X, denotado por Conv(X) es definido como la interseccion de todos los conjuntos
convexos que contienen a X, esto es,
\
Conv(X) = C .
XC
C convexo

Observacion 5.1 Puede probarse que la envoltura convexa Conv(X) es un conjunto con-
vexo y que si X es convexo, entonces Conv(X) = X.

Ejemplo 5.4 Conv ({x Rn : kxk = 1}) = {x Rn : kxk 1}.

Ejemplo 5.5 Dados x, y Rn , entonces

Conv ({x, y}) = {z : z = x + (1 )y, [0, 1]} .

Proposicion 5.2 Sea X un conjunto arbitrario de Rn . La envoltura convexa Conv(X)


es el conjunto de todas las conbinaciones convexas de puntos de X, esto es,
( p p
)
X X
Conv(X) = y Rn : y = i xi , xi X, i 0, i = 1, donde p IN .
i=1 i=1

Prueba. Sea
p p
X X
i i
C = {x = i x donde x X, i 0, i = 1, . . . , p y i = 1, p IN }.
i=1 i=1

112
Conv(X) es convexo, entonces por el Teorema 5.1, contiene todas las combinaciones
convexas de puntos de Conv(X) y en particular de X, esto es, C Conv(X).
Por otro lado, si X C entonces Conv(X) Conv(C). Si probamos que C es convexo
entonces Conv(C) = C y por tanto tendramos Conv(X) C. Sean x, y C, entonces
p1
! p2
!
X X
x + (1 )y = i xi + (1 ) j y j
i=1 j=1

p1 p2
X X
i
= (i )x + [(1 )j ] y j .
i=1 j=1

Como
p1 p2 p1 p2
X X X X
i + (1 )j = i + (1 ) j
i=1 j=1 i=1 j=1

= (1) + (1 )(1) = 1,

se tiene que, x + (1 )y es una combinacion convexa de puntos {xi }pi=1
1
{xj }pj=1
2
.
Por lo tanto, C es convexo.

Corolario 5.1 Sea X un conjunto arbitrario de Rn . La envoltura convexa Conv(X) es


el conjunto de todas las combinaciones convexas de n + 1 puntos de X, esto es,
( n+1 n+1
)
X X
Conv(X) = y Rn : y = i xi , xi X, i 0, i = 1 .
i=1 i=1

Prueba. Usando el teorema anterior y el Teorema 5.2 (teorema de Caratheodory) obten-


emos el resultado.

5.2 El Teorema de la Proyeccion


El teorema de la proyeccion da condiciones sucientes para garantizar la existencia
de una mnima distancia de un punto a un conjunto dado.

Definicion 5.5 Sea C un subconjunto de Rn y y C, decimos que PC (y) C es la


proyeccion de y sobre C si PC (y) resuelve el problema de min{||y x|| : x C}. Esta
ultima condicion tambien puede ser denotada por:

PC (y) arg min {||y x|| : x C}

113
y

PC (y)

Teorema 5.3 Sea C Rn un conjunto convexo no vacio. Dado y


/ C, las siguientes
afirmaciones son equivalentes:

a) PC (y) C y PC (y) es la proyeccion de y sobre C.

b) PC (y) C verifica hy PC (y), x PC (y)i 0, para todo x C.


y

PC (y)
90

Demostracion. Mostremos que a) implica b). Sea x C, (0, 1], entonces de la


convexidad de C, tenemos z() = PC (y) + (x PC (y)) C. Usando la hipotesis a)
tenemos
||y PC (y)||2 ||y z()||2

= ||y PC (y) (x PC (y))||2

= ||y PC (y)||2 + 2 ||x PC (y)||2 2hy PC (y), x PC (y)i

entonces
2hy PC (y), x PC (y)i 2 ||PC (y) x||2

Diviendo por se tiene

hy PC (y), x PC (y)i (/2)||PC (y) x||2 .

114
Tomando el nmo para los valores de , tenemos
 
hy PC (y), x PC (y)i inf ||PC (y) x||2
(0,1]

entonces,
hy P (y), x P (y)i 0.

Ahora probemos que b) implica a). Sea x C, luego

||y x||2 = ||(y PC (y)) + (PC (y) x)||2

= ||y PC (y)||2 + ||PC (y) x||2 2hy PC (y), x PC (y)i

Por hipotesis, hy PC (y), x PC (y)i 0, entonces

||y x||2 ||y PC (y)||2 + ||PC (y) x||2 ||y PC (y)||2 .

De aqu obtenemos que ||y x|| ||y PC (y)|| y as PC (y) es la proyeccion de y sobre
C.

Teorema 5.4 (De la proyeccion) Sea C Rn un conjunto convexo no vacio y cer-


rado. Sea y
/ C, entonces existe una unica proyeccion PC (y) de y sobre C.

y
C
x

Demostracion. Como la funcion k.k es coerciva y C es cerrado, entonces existe PC (y)


C, tal que minxC ||y x|| = ||y PC (y)||. Para probar la unicidad, supongamos que
existe otra proyeccion PC (y) de y sobre C.
Del Teorema 5.3 tenemos

hy PC (y), PC (y) PC (y)i 0 y (5.3)

115
hPC (y) y, PC (y) PC (y)i 0. (5.4)

Sumando las desigualdades (5.3) y (5.4) tenemos

hPC (y) PC (y), PC (y) PC (y)i 0,

esto es, ||PC (y) PC (y)||2 0. Por propiedad de norma concluimos que PC (y) = PC (y).

5.3 Separacion de Conjuntos Convexos


Definicion 5.6 Un hiperplano H es un subconjunto de Rn de la forma

H = {x Rn : hp, xi = } ,

donde p 6= 0 y es un numero real.

Observacion 5.2 Un hiperplano H define dos semiespacios (cerrados o abiertos o semi-


abiertos) de la forma:

H + = {x Rn ; hp, xi } y H = {x Rn : hp, xi }.

H ++ = {x Rn ; hp, xi > } y H = {x Rn : hp, xi < }.

H ++ = {x Rn ; hp, xi > } y H = {x Rn : hp, xi }.

H + = {x Rn ; hp, xi } y H = {x Rn : hp, xi < }.

Definicion 5.7 (Hiperplano separador de dos conjuntos.) Sean X1 y X2 dos con-


juntos no vacios de Rn . Sea H un hiperplano dado por H = {x Rn : hp, xi = }, donde
p 6= 0 y es un numero real.

1. Se dice que H separa X1 y X2 si hp, x1 i hp, x2 i, para todo x1 X1 y todo


x2 X2 .

2. Se dice que H separa X1 y X2 estrictamente si hp, x1 i < < hp, x2 i, para todo
x1 X1 y todo x2 X2 .

116
3. Se dice que H separa X1 y X2 fuertemente si existe > 0 tal que hp, x1 i <
+ hp, x2 i, para todo x1 X1 y todo x2 X2 .

Teorema 5.5 (Separacion de un punto a un conjunto convexo.) Sea C Rn un


/ C entonces existe p 6= 0, p Rn , y R
conjunto convexo no vacio y cerrado. Si y
tal que hp, xi < hp, yi, para todo x C.

Demostracion. Como C es convexo y cerrado, entonces por el Teorema 5.4 (teorema


de la proyeccion) y del Teorema 5.3, existe PC (y) C tal que

hx PC (y), y PC (y)i 0,

para todo x C. Denamos p = y PC (y). Como y


/ C se tiene que p 6= 0. De la
desigualdad anterior hp, xi hp, PC (y)i, para todo x C.
Deniendo = hp, PC (y)i R, tenemos

hp, xi , para todo x C. (5.5)

Ademas,
hp, yi = hp, p + PC (y)i = ||p||2 + .

Como p 6= 0, entonces ||p|| > 0 de aqu

hp, yi > . (5.6)

De (5.5) y (5.6) tenemos hp, xi < hp, yi, para todo x C.

Corolario 5.2 Sea C Rn un conjunto convexo no vacio y cerrado, entonces C es la


interseccion de todos los semiespacios que contienen a C, esto es,
\
C= H ,
CH

donde H es un semiespacio para cada I (conjunto arbitrario de parametros).

Prueba.
T
i) La inclusion C H , es inmediata.
CH

117
T T
ii) Probaremos que H C. Sea y H y supongamos que y
/ C. Como
CH CH
/ C, entonces por el Teorema 5.5, existe p 6= 0, R
C es convexo, cerrado e y
tal que
hp, xi < hp, yi, para todo x C, (5.7)

esto es, existe un hiperplano H = {z Rn : hp, zi = } que separa y y C. Por


la primera desigualdad de (5.7), C H , y por la segunda desigualdad y
/ H .
/ H y C H , lo que es una contradiccion con
Juntando ambos tenemos que y
la hipotesis. As el resultado es obtenido.

Corolario 5.3 Sea X Rn un conjunto arbitrario y y


/ Cl(Conv(X)), entonces existe
un hiperplano que separa fuertemente y de X.

Prueba. Como Conv(X) es convexo entonces Cl(Conv)(X)) es convexo y cerrado.


/ Cl(conv(X)) entonces por Teorema 5.5 existen p 6= 0 y R
Ademas, como y
tal que
hp, xi < hp, yi, para todo x Cl(Conv(X)). (5.8)
hp,yi
Deniendo = 2
> 0, tenemos

+ hp, yi hp, yi + hp, yi


+ = <
2 2

De aqu se tiene:
+ < hp, yi (5.9)

De (5.8) tenemos en particular hp, xi bar < hp, yi, para todo x X. De (5.9),
< + < hp, yi. As, obtenemos hp, xi < + hp, yi, para todo x X.

Lema 5.1 (Lema de Farkas) Sea A Rmn y c Rn entonces solamente uno de los
dos sistemas tiene solucion:

Sistema 1 Existe x Rn tal que Ax 0 y cT x > 0.

Sistema 2 Existe y Rm satisfaciendo AT y = c, con y 0.

118
Prueba. Supongamos que el Sistema 2 tiene solucion, entonces existe y Rm tal que
At y = c, y y 0. Sea x Rn arbitrario, tal que Ax 0, entonces

cT x = (AT y)T x = y T Ax 0,

por lo tanto el Sistema 1 no tiene solucion.


Supongamos ahora que el sistema 2 no tiene solucion. Denamos el conjunto

S = {x Rn /x = At y, y 0}.

Es facil probar que S es un conjunto no vacio, convexo y cerrado, ademas c


/ S. Por el
Teorema 5.5 (teorema de separacion), existe p 6= 0, R tal que hp, xi < hp, ci,
para todo x S. Esto es,

pT x < cT p, para todo x S. (5.10)

Como y 0, se puede tomar y = 0 y as 0 S. Aplicando x = 0 en (5.10), se tiene


0 < cT p, y de aqu
cT p > 0. (5.11)

Tambien, de (5.10) se tiene

pT x = (Ap)T y para todo y 0 (5.12)

De aqu se obtiene que


(Ap) 0 (5.13)

En efecto, si no se cumple entonces existe i0 {1, 2, ..., m} tal que (Ap)i0 > 0. Tomando
y = t(Ap)i0 ei0 , donde t > 0 y ei0 es el vector canonico de todos los elementos iguales a
cero excepto la componente i0 , en (5.12) obtenemos que t(Ap)i0 y tomando t
llegamos a +, lo que es una contradiccion. Por lo tanto, Ap 0. Finalmente de
(5.11) y (5.13) se concluye que el sistema 1 tiene solucion.

Definicion 5.8 (Soporte de conjuntos) Sea X un subconjunto no vacio de Rn , p


Rn con p 6= 0 y y Front(X). El hiperplano

H = {z Rn : hp, z yi = 0}

es llamado plano soporte de X en el punto y si satisface una de las siguientes inclusiones:

119
X {z Rn : hp, z yi 0}.

X {z Rn : hp, z yi 0}.

Observacion 5.3 De la definicion anterior podemos concluir que

1. Un hiperplano soporte de X en el punto y no es necesariamente unico.

2. Un hiperplano puede ser soporte para mas de un punto en la frontera.

3. Un hiperplano soporte puede contener a todo X.

Teorema 5.6 (Soporte a un conjunto convexo) Sea C 6= un conjunto convexo en


Rn y y Front(C), entonces existe un hiperplano que soporta a C en y.

Demostracion. Como y Front(C), entonces existe una sucesion de puntos {y k }, y k


/
Cl(C), tal que limk y k = y. Por el Teorema 5.5 (teorema de separacion), para cada
k IN , existe pk 6= 0 tal que hpk , xi < hpk , yi, para todo x Cl(C).
D k E D k E
Como pk 6= 0, entonces ||pk || > 0 y de aqu tenemos ||ppk || , x < ||ppk || , y , para todo
n k o
x Cl(C). La sucesion ||ppk || es limitada, entonces existe p Rn y una subsucesion
n k o kj
p j
||pkj ||
tal que limj+ ||ppkj || = p 6= 0.

D E D E
pkj pkj
Considerando k = kj en la desigualdad anterior, tenemos ||pkj ||
,x < ||pkj ||
, y , para
todo x Cl(C). Tomando j +, obtenemos hp, xi hp, yi, esto es, hp, x yi 0,
para todo x Cl(C).

Corolario 5.4 (Separacion de un conjunto convexo no cerrado) Sea C 6= un


conjunto convexo en Rn y y
/ int(C), entonces existe p Rn , p 6= 0, tal que hp, xyi 0,
para todo x Cl(C)

Prueba. Como y
/ int(C), puede suceder 2 casos:

/ Cl(C) entonces por el Teorema 5.5 (teorema de separacion), existe p Rn ,


i) Si y
p 6= 0, tal que hp, xi < hp, yi, para todo x Cl(C). Entonces hp, x yi < 0, para
todo x Cl(C).

120
ii) Si y Cl(C) entonces y Front(C), ya que y
/ int(C), de donde, por el Teorema
5.6, existe p Rn , p 6= 0, tal que hp, x yi 0, para todo x Cl(C).

Corolario 5.5 Sea X 6= un conjunto de Rn y y


/ int(Conv(X)), entonces existe un
hiperplano que separa X y y.

Prueba. Usando el corolario anterior para C = Conv(X) obtenemos que existe p 6= 0


tal que hp, x yi 0, para todo x Cl(conv(X)). Como X Cl(conv(X)) entonces
hp, x yi 0, para todo x X.
T
Corolario 5.6 Sea X 6= un conjunto de Rn y y Front(X) Front(Conv(X)) en-
tonces existe un hiperplano que separa X y y.

Prueba. Inmediato.

Teorema 5.7 (Separacion de dos conjuntos convexos) Sean C1 6= y C2 6= dos


conjuntos convexos en Rn tal que C1 C2 = . Entonces, existe un hiperplano que separa
C1 y C2 , esto es, existe un vector p 6= 0 tal que sup {hp, xi, x C1 } inf {hp, yi, y C2 }.

Demostracion. Denamos C = C1 C2 = {x = x1 x2 tal que x1 C1 , x2 C2 }. C


es convexo. Ademas 0
/ C entonces 0
/ int(C), luego por corolario 5.4 existe un vector
p Rn , p 6= 0, tal que hp, x 0i 0, para todo x Cl(C). En particular, hp, x 0i 0,
para todo x C.
De la denicion de C tenemos que hp, x1 i hp, x2 i, para todo x1 C1 y todo x2 C2 .
Tomando supremo e nmo obtenemos el resultado.

5.4 Funciones convexas


Definicion 5.9 Sea C un conjunto convexo no vacio y f : C Rn R una funcion.
La funcion f es llamada convexa si satisface

f (x1 + (1 )x2 ) f (x1 ) + (1 )f (x2 ),

para todo x1 , x2 C y todo [0, 1]. f es estrictamente convexa en C si la desigualdad


anterior es estricta para todo x1 , x2 C, x1 6= x2 y (0, 1).

121
f

f (y)

y)
f(
)
1 )
) +( )
y
(x
f (1
+
x
f(
f (x)

x x +(1 )y y

Figura 5.5: Graca de una funcion convexa

Definicion 5.10 Sea C un conjunto convexo no vacio y f : C Rn R una funcion.


La funcion f es llamada fuertemente convexa con modulo > 0 si

f (x + (1 )y) f (x) + (1 )f (y) (1 ) kx yk2 ,

para todo x, y C y [0, 1].

Observacion 5.4 Observemos que de las definiciones anteriores tenemos que si f es


fuertemente convexa entonces f es estrictamente convexa y esto implica que f es convexa,
pero los recprocos no son necesariamente verdaderos como mostramos en el siguiente
ejemplo.

Ejemplo 5.6 A seguir presentamos algunos ejemplos de funciones convexas, estricta-


mente convexas y fuertemente convexas.

1. f (x) = x2 es fuertemente convexa en R.

2. f (x) = ex es estrictamente convexa en R (pero no es fuertemente convexa).

3. f (x) = x es convexa en R (pero no es estrictamente convexa).

122
f (a) + (1 z)f (b)

f (a) + (1 z)f (b)

a b

Figura 5.6: Grafica de una funcion que no es estrictamente convexa

Definicion 5.11 Sea C un conjunto convexo no vacio y f : C Rn R una funcion.


La funcion f es llamada concava (estrictamente concava) si f es convexa (estrictamente
convexa).

Definicion 5.12 Sea C un conjunto convexo no vacio y f : C Rn R una funcion.


La funcion f es llamada fuertemente concava si f es fuertemente convexa en C.

Definicion 5.13 Una funcion f : Rn R es afn si para todo x, y Rn y R se


tiene que
f (x + (1 )y) = f (x) + (1 )f (y),

Teorema 5.8 Sea C 6= un conjunto convexo. f : C R, es una funcion convexa si y


solo si el conjunto Epi(f ) := {(x, r) C R : f (x) r} es convexo en Rn R.

Demostracion. Sea f convexa, probaremos que Epi(f ) es convexo. Sean (x, ) y


(y, ) Epi(f ) entonces f (x) y f (y) . Por la convexidad de f se tiene que para
todo [0, 1] se cumple

f (x + (1 )y) f (x) + (1 )f (y) + (1 ),

esto implica que (x + (1 )y, + (1 )) Epi(f ), para todo [0, 1], esto es

(x, ) + (1 )(y, ) Epi(f ), para todo [0, 1].

123
f (a) + (1 z)f (b)

f (a) + (1 z)f (b)

a b

Figura 5.7: Grafica de una funcion Convexa

As Epi(f ) es convexo.
Recprocamente, probaremos que si Epi(f ) es convexo, entonces f es convexa en C. Sea
x, y C, esto implica que (x, f (x)), (y, f (y)) Epi(f ). Como Epi(f ) es convexo, se tiene
que (x, f (x)) + (1 )(y, f (y)) Epi(f ), para todo [0, 1], esto es,

f (x + (1 )y) f (x) + (1 )f (y), para todo [0, 1],

As obtenemos que f es convexa.

Teorema 5.9 Sea C 6= un conjunto convexo, y f : C R, una funcion convexa.


Entonces el conjunto de nivel Lf () := {x C : f (x) } es convexo en Rn para todo
R.

Demostracion. Sean x, y Lf (), entonces f (x) y f (y) . Como f es convexa


se tiene que

f (x + (1 )y) f (x) + (1 )f (y) + (1 ) = ,

para todo [0, 1]. Esto implica que x + (1 )y Lf (), para todo [0, 1]. De
aqu, Lf () es convexo para todo R.

Observacion 5.5 El recproco del Teorema 5.9 no es verdadero, considere por ejemplo
la funcion f (x) = ln x, la cual no es convexa pero el conjunto de nivel Lf () es convexo
para todo R.

124
b)
)f (
1 z
+(
(a)
f

a b

Figura 5.8: Grafica de una funcion Convexa

Proposicion 5.3 Sea f : Rn R. Entonces f es convexa si, y solamente si, para todo
x Rn , d 6= 0, g(t) = f (x + td) es convexa en R.

Prueba. Sea f convexa y sean t1 , t2 R, entonces para todo [0, 1]

g(t1 + (1 )t2 ) = f (x + (t1 + (1 )t2 )d)

= f ((x + t1 d) + (1 )(x + t2 d))

f (x + t1 d) + (1 )f (x + t2 d)

= g(t1 ) + (1 )g(t2 ),

donde la primera igualdad es por la denicion de g, la segunda es un reacomodo de x, la


primera desigualdad es obtenida por la convexidad de f y la ultima igualdad nuevamente
por la denicion de g.
Recprocamente, asumamos que para todo x Rn y todo d Rn , d 6= 0, se tiene que
g(t) = f (x + td) es convexa. Probaremos que f es convexa. Sean x, y Rn , entonces
para todo [0, 1]
f (x + (1 )y) = f (y + (x y))

125
a
b

Figura 5.9: Grafica de una funcion no Convexa

= g()

= g(.1 + (1 )0)

g(1) + (1 )g(0)

= f (x) + (1 )f (y),

donde la desigualdad es dada por la convexidad de g. As, f es convexa.

Lema 5.2 (Desigualdad de Jensen) Sea f : C R una funcion convexa. Para cada
P
x1 , x2 , . . . , xm C y 1 , 2 , . . . , m , numeros no negativos tal que m
i=1 i = 1, se tiene

m
! m
X X
i
f i x i f (xi ).
i=1 i=1

Prueba. Si m = 1 o 2, entonces el resultado es inmediato de la denicion de convexidad


de f . Supongamos que el resultado es valido para m = n. Probaremos que es valido para
P
m = n + 1. Sean x1 , x2 , . . . , xn , xn+1 C y 1 , 2 , . . . , n , n+1 0 tal que n+1
i=1 i = 1.

La combinacion convexa de los puntos lo podemos expresar como


n+1
X n
X
i
i x = i xi + n+1 xn+1 .
i=1 i=1

126
f
Epi(f )

Figura 5.10: El Epi(f ) de una funcion convexa es un conjunto convexo

Pn+1 i n+1
Si n+1 = 1 entonces 1 = 2 = = n = 0, esto es, i=1 i x = x . As,
m
! m
X X
i n+1 1 2 n+1
f i x = f (x ) = 0f (x ) + 0f (x ) + . . . + n+1 f (x ) = i f (xi ).
i=1 i=1

Si n+1 6= 1, tenemos que


n+1
X n
X
i i
i x = (1 n+1 ) xi + n+1 xn+1 C.
i=1 i=1
(1 n+1 )

f es convexa luego
n+1
! n   !
X X i
i i
f i x (1 n+1 )f x + n+1 f (xn+1 ).
i=1 i=1
1 n+1

Como
n
X Xn
i 1 1 n+1
= i = = 1;
i=1
(1 n+1 ) 1 n+1 i=1 1 n+1
y usando la hipotesis inductiva tenemos de la desigualdad anterior
n+1
! n
X X i
i
f i x (1 n+1 ) f (xi ) + n+1 f (xn+1 )
i=1 i=1
1 n+1

n+1
X
= i f (xi ).
i=1

127
Corolario 5.7 Sea f : C R una funcion convexa y sea x C una combinacion
convexa de puntos x1 , x2 , . . . , xm C entonces f (x) maxi=1,...,m {f (xi )}.

Pm i
Prueba. Como x es una combinacion convexa de m puntos entonces, x = i=1 i x , y
P
i 0 mi=1 i = 1. Tomando f y usando el Lema 5.2 tenemos

m
! m m
X X X
i i
f (x) = f i x i f (x ) i max f (xi ) = max f (xi )
1xm 1xm
i=1 i=1 i=1

Corolario 5.8 Sea f : Rn R una funcion convexa y C = Conv({x1 , x2 , . . . , xm }),


entonces f (x) maxi=1,...,m {f (xi )}.

Prueba. Inmediato del Corolario anterior.

Teorema 5.10 Sea C un conjunto convexo, entonces f : C R es una funcion convexa


si y solamente si para todo x, y C y 0 tal que y (x y) C se tiene

f (y (x y)) f (y) (f (x) f (y)).

Demostracion. Supongamos que f es convexa y sea x, y C, 0 tal que y (x


1
y) C. Denamos z = y (x y), as y = 1+
z + 1+
x. De la convexidad de f
tenemos,
1
f (y) f (z) + f (x).
1+ 1+
Multiplicando por 1 + ,
(1 + )f (y) f (z) + f (x),

entonces
f (z) (1 + )f (y) f (x)

= f (y) (f (x) f (y))

Recprocamente, supongamos que para todo x, y C, 0 tal que y (x y) C se


tiene
f (y (x y)) f (y) (f (x) f (y)).

128
Sea x, y C y (0, 1]. Denamos

1
= 0 y = x + (1 )y

de donde,
1
x= ( (1 )y) = + ( y).

De la hipotesis tenemos f (x) = f ( (y )) f () (f (y) f ()). Usando la

denicion dada para , f (x) (1 + )f () f (y) = 1 f () 1

f (y). Multiplicando
por se tiene,
f (x) f () (1 )f (y).

De aqu,
f () f (x) + (1 )f (y),

esto es,
f (x + (1 )y) f (x) + (1 )f (y).

5.5 Continuidad de Funciones Convexas


Lema 5.3 Sea f : C R es una funcion convexa y x0 int(C), entonces f es acotada
superiormente en una vecindad de x0 .

Prueba. Sea x0 int(C) y > 0 tal que x0 ei int(C), donde ei = (0, . . . , 1, 0, . . . , 0)


con 1 en la posicion iesima. Denotemos por

X = conv{x0 ei , i = 1, . . . , n}.

  
0
Probaremos que B x0 , n X. Sea x B x , , entonces
n
 
y = x x0 B 0, .
n

De aqu,
 
x = x0 + y, y B 0, ,
n

129
y por tanto, v
n u n
X uX
x = x0 + yi ei , t yi2 < . (5.14)
i=1 i=1
n
Podemos asumir que yi 0, para i = 1, . . . , n. Si yi0 < 0, para algun i0 , entonces
podemos elegir ei0 como ei0 en (5.14).
1
Como x0 = 2
+ ei ) + 21 (x0 ei ), para todo i = 1, . . . , n, tenemos que x0 X.
(x0
Pn Pn
Denamos ahora, = i=1 yi 0. As tenemos, = i=1 yi = ||y||1 n||y||2 <
 P  
n
n n = , de donde < 1. Ademas, de la denicion de , la i=1 yi = 1. Denamos,

P
yi = yi entonces, ni=1 yi = 1De aqu,
n
X
x = x0 + yi ei
i=1
X n
0 0 0
= x x + x + yi ei
i=1
  n n
0 X 0 X
= 1 x + yi x + yi ei ,
i=1 i=1
  n
!
0 X 0 i
= 1 x + (yi )(x + e ) .
i=1

Pn
Como x0 X y i=1 yi (x0 + ei ) X, por ser combinacion convexa de {x0 +ei }, se
garantiza que x X.
Por lo tanto, al ser x combinacion convexa de {x0 +ei }, se tiene
2n
X 2n
X
0 i
x= i (x +e ) : i = 1 , i 0,
i=1 i=1

aplicabdo f !
2n
X
f (x) = f i (x0 +ei ) ,
i=1
y por la convexidad de f ,
2n
X
f (x) i f (x0 +ei )
i=1
X2n
= i maxi=1,...,2n {f (x0 +ei )}
i=1
= maxi=1,...,2n {f (x0 +ei )},

130
es decir, existe M = maxi=1,...,2n {f (x0 +ei )} tal que f (x) M, para todo x

B x0 , n .

Observacion 5.6 Es facil ver que una funcion convexa puede ser discontinua en la
frontera de un conjunto cerrado. Por ejemplo, considere la funcion f : R+ R definida
por

x2 , si x > 0
f (x) =

1, si x = 0
Se puede verificar facilmente que f es una funcion convexa en R+ pero no es continua
en x = 0.

El siguiente teorema nos permitira armar luego que toda funcion convexa es continua
en el interior de su dominio.

Teorema 5.11 Sea f : C R una funcion convexa y x0 int(C), entonces f es


localmente Lipschitziana en x0 .

Demostracion. Por el Lema 5.3, existe > 0 tal que f (x) M , para todo x B(x0 , ).
Considere x B(x0 , 2 ) tal que x 6= x0 . Sean:
2
= ||x x0 || > 0 (As, 0 < < 1)

1
z = x0 + (x x0 ),

1
entonces, x = z + (1 )x0 . Es claro que ||z x0 || =
||x x0 || = 2
< luego,
z B(x0 , ).
Por la convexidad de f se tiene,

f (x) f (z) + (1 )f (x0 )

= f (x0 ) + (f (z) f (x0 ))

f (x0 ) + (M f (x0 ))
2
= f (x0 ) + (M f (x0 ))||x x0 ||

2
entonces, f (x) f (x0 ) 2 (M f (x0 ))||x x0 ||. Deniendo k(x0 , ) = (M f (x0 )).

tenemos

f (x) f (x0 ) K x x0 .

131
1 0
Por otro lado, sea u = x0 + 1 (x0 x). de donde, ||u x0 || = ||x x|| < , entonces

u B(x0 , ). Tambien , x = x0 + (x0 u), luego aplicando el Teorema 5.10 tenemos,

f (x) f (x0 ) + (f (x0 ) f (u))

f (x0 ) + (f (x0 ) M )

= f (x0 ) + 2 (f (x0 ) M )||x x0 ||

2
entonces, f (x) f (x0 )
(M f (x0 ))||x x0 ||. Por lo tanto se tiene, |f (x) f (x0 )|
k(, x0 )||x x0 ||, para todo x B(x0 , /2).

Corolario 5.9 Sea C un conjunto convexo y f : C R una funcion convexa, si x


int(C), entonces f es continua en x.

Prueba. Como f es localmente Lipschitziana en x int(C), entonces esta es continua


en x.

Corolario 5.10 Sea f : Rn R una funcion convexa, entonces f es continua en Rn .

Prueba. Como int(Rn ) = Rn , el resultado se obtiene inmediatamente.

5.6 Derivada Direccional de Funciones Convexas


Definicion 5.14 Sean C un conjunto convexo y f : C R una funcion. Dado x C y
d Rn , d 6= 0, si el siguiente limite

f (x + d) f (x)
f (x, d) = lim ,
0
>0

existe, entonces f (x, d) es llamada la derivada direccional de f en la direcion d en el


punto x.

Dado x int(C), entonces existe > 0 tal que B(x, ) C. Para d Rn , d 6= 0,


 

denimos la funcion : 0, ||d|| R tal que

f (x + d) f (x)
() = .

132
Teorema 5.12 Sean C un conjunto convexo con int(C) 6= y f : C R una funcion
convexa, entonces para x int(C) y d Rn , d 6= 0, se tiene que:

i. La funcion es no decresciente

ii. La derivada direccional f (x, d) existe.

Demostracion.
i. Sea dom tal que , luego existe t (0, 1] tal que = (1 t)0 + t = t.
h 

Denamos la funcion : 0, ||d|| R tal que () = f (x + d). Por la convexidad de
f se tiene que es convexa, as

() = ((1 t)0 + t)

(1 t)(0) + t().

Restando a ambos lados (0) y dividiendo por obtenemos


() (0) (1 t)(0) + t() (0)


t(() (0))
=

t(() (0))
= ,
t
entonces
() (0) () (0)
,

es decir,
f (x + d) f (x) f (x + d) f (x)


Por lo tanto () ().
ii. Probemos ahora la existencia de f (x, d).
La funcion f es convexa en x int(C), entonces por Teorema 5.11, para sucientemente
pequeno, existe K > 0 tal que

|f (x + d) f (x)| K||d||

f (x+d)f (x)
entonces, K||d|| <
, y as K||d|| < () Luego, es acotada inferiormente
para sucientemente pequeno.

133
Como es acotada inferiormente para sucientemente pequeno y no decreciente, ver
tem i., entonces existe,
f (x + d) f (x)
lim = f (x, d)
0
>0

Observacion 5.7 Como decresce a medida que toma valores pequenos tenemos que:
f (x + d) f (x)
f (x, d) = inf .
>0
Lema 5.4 Sean C un conjunto convexo, f : C R una funcion convexa y x int(C),
entonces f (x, .) : Rn R es homogenea de grado 1, convexa y para todo y C:
f (y) f (x) + f (x, y x).

Prueba. La homogeneidad y convexidad son inmediatos, por eso solo probaremos la


desigualdad f (y) f (x) + f (x, y x).
Por la convexidad de f , y todo [0, 1]

f (y + (1 )x) f (y) + (1 )f (x).

En particular para (0, 1]:

f (x + (y x)) f (x) + (f (y) f (x)),

luego
f (x + (y x)) f (x)
f (y) f (x),

tomando lmite cuando 0 y de la existencia de la derivada direccional tenemos que

f (x, y x) f (y) f (x).

5.7 Subdiferenciabilidad
Definicion 5.15 Sea C un conjunto convexo de Rn y f : C R una funcion. El punto
s Rn es un subgradiente de f en x C si

f (y) f (x) + hs, y xi, para todo y C.

134
El conjunto de todos los subgradientes de f en x es llamado el subdiferencial de f en x
y es denotado por f (x), esto es,

f (x) = {s Rn : f (y) f (x) + hs, y xi para todo y C}.

Ejemplo 5.7 Sea f : R R definida por f (x) = |x| (ver Figura 5.11) entonces

Figura 5.11: La funcion valor absoluto.





[1, 1] , si x = 0,


f (x) = +1, si x > 0,





1, si x < 0.

En efecto, consideremos los siguientes casos:

i) Sea x = 0, entonces

f (0) = {s R : |y| |0| + s.y, y R} .

Observamos que:
Si y = 0 entonces s puede tomar cualquier valor real, esto es s R.

135
Si y > 0 entonces y s.y, implicando que s (, 1]
Si y < 0 entonces y .y, se tiene s [1, +).
Interceptando los conjuntos tenemos que s [1, 1] y por lo tanto

f (0) = [1, 1] .

ii) Sea x > 1, entonces

f (x) = {s R : |y| x + s. (y x) , y R} .

Si y = 0, entonces 0 x(1 s), lo que implica que s [1, +).


Si y > 0 y y 6= x, tenemos que y x > 0 y x < 0

Si y x < 0 tenemos que y x s (y x) lo que implica que s [1, +).

Si y x > 0 tenemos que y x s (y x) , y esto implica que s (, 1].

De ambos casos obtenemos que s = 1.


Si y > 0 y y = x observamos que la desigualdad y x s (y x) se cumple
trivialmente para cualquier s R. En particular para s = 1.
Finalmente si y < 0, se tiene que y x + s(y x), lo que es verdad para s = 1.
Por lo tanto,
f (x) = {1}, x > 0.

iii) Para x < 0, se procede de manera similar al caso del item anterior, obteniendo

f (x) = {1}, x < 0.

Ejemplo 5.8 Sea f : R R definida por



1 x; x<1
f (x) =
x2 1, x 1.

136
Figura 5.12: Graca de la funcion del Ejemplo 5.8.

La graca de la funcion es dada por la Figura 5.12. Hallaremos el subdiferencial en cada


punto de R.
Como la dimension es n = 1, entonces:

f (x) = {s R : f (y) f (x) + s(y x), y R}

i) Si x < 1, entonces f (x) = 1 x. Estudiaremos los siguientes casos:

i.1) Si y < 1, entonces f (y) = 1 y. Reemplazando en la desigualdad del subd-


iferencial tenemos 1 y 1 x + s(y x), lo que es equivalente a:

(y x) s(y x). (5.15)

Si (y x) > 0, simplicando (5.15) se tiene 1 s, entonces s


(, 1].
Si (y x) < 0, de (5.15) se tiene que 1 s, y de aqu s [1, +).
Si y = x simplicando (5.15), se tiene: 0 s.0, lo que es verdad para
todo s R.

Interceptando los conjuntos tenemos que la unica posibilidad para que se


cumpla la desigualdad en la denicion del subdiferencial es que s = 1

137
i.2) Si y = 1, entonces f (y) = 0, reemplazando en la desigualdad de la denicion
del subdiferencial se tiene que 0 1 x + s(1 x), lo que es equivalente a
s(1 x) (1 x). Observemos que esta ultima desigualdad se cumple para
s = 1.

i.3) Si y > 1, entonces f (y) = y 2 1, reemplazando en la desigualdad del sub-


diferencial se obtiene y 2 1 1 x + s(y x). Tomando s = 1 se tiene
(y1)(y+1) 1x+(1)(yx) lo que equivale a (y1)(y+1) 1xy+x
simplicando se tiene y 2 lo que es verdadero.

Luego de (i.1), (i.2) y (i.3) tenemos f (x) = {1}.

ii) Si x = 1, entonces f (x) = 0.

ii.1) Si y < 1 entonces f (y) = 1 y, luego la desigualdad del subdiferencial es


1 y 0 + s(y 1), lo que es equivalente a 1 s y de aqu s [1, +).

ii.2) Si y 1, entonces f (y) = y 2 1 y de aqu la desigualdad del subdiferencial es


(y 1)(y + 1) s(y 1) lo que equivale a (y + 1) s. Tomando limy1 f (y),
se tiene que s 2 luego s h, 2] .
De (ii.1) y (ii.2) se tiene que

f (x) = [1, 2].

iii) Si, x > 1, entonces f (x) = x2 1. Se tiene los siguientes casos:

iii.1 Si y 1 entonces f (y) = y 2 1, tenemos de la denicion:

c f (x) = {s R : f (y) f (x) + s(y x), y R}

Como: (y x)2 0, desarrollando el binomio tenemos


y 2 + x2 2xy, lo que es equivalente a:

y 2 1 x2 1 + 2xy 2x2 ,

lo que tambien es equivale a:

y 2 1 x2 1 + 2x(y x).

De la desigualdad anteriore se tiene que: s = 2x.

138
iii.2 Si y < 1 entonces f (y) = 1 y, veriquemos que la siguiente desigualdad

1 y (x2 1) + 2x(y x), x > 1.

es verdadera.
La desigualdad anterior es equivalente a:

1 y x2 1 + 2xy

lo que equivale a: 0 x2 2 + y(1 + 2x)...()


tenemos los siguientes casos:

Si y 0 la desigualdad () es verdadera.

Si 0 < y < 1 tenemos lo siguiente:


y(1+2x) < (1+2x)lo que equivale a: x2 2+y(1+2x) < x2 1+2x
lo que equivale a: x2 2 + y(1 + 2x) < (x2 2x + 1)
lo que equivale a: x2 2 + y(1 + 2x) < (x 1)2 < 0 es verdadero
de (a) y (b) tenemos: c f (x) = {2x}.

Lema 5.5 Sea C un conjunto convexo, f : C R una funcion y x C, entonces f (x)


es convexo y cerrado.

Prueba. Probemos inicialmente la convexidad.


Sean s1 , s2 f (x), entonces para todo [0, 1] se tiene f (y) f (x) + hs1 , y xi,
para todo y C, esto es (1 )f (y) (1 )f (x) + h(1 )s2 , y xi, para todo y C.
Sumando ambas desigualdades tenemos

f (y) f (x) + hs1 + (1 )s2 , y xi

para todo y C, esto implica que s1 + (1 )s2 f (x), para todo [0, 1].
Ahora probemos la cerradura.

Sea sk una sucesion en f (x) tal que limk sk = s. Como sk f (x) entonces
f (y) f (x) + hsk , y xi, para todo y C. Tomando lmite cuando k va para el innito
y usando la continuidad del producto interno h, i tenemos f (y) f (x) + hs, y xi, para
todo y C. As, s f (x). De ambos resultados obtenemos que f (x) es convexo y
cerrado.

139
Lema 5.6 Sea C un conjunto convexo y f : C R una funcion. Si para todo x C se
tiene f (x) 6= , entonces f es convexa.

Prueba. Sean x, y C y [0, 1]. Como C es convexo entonces x + (1 )y C,


luego por hipotesis f (x + (1 )y) 6= . Sea s f (x + (1 )y), entonces

f (y) f (x + (1 )y) + hs, y (x + (1 )y)i.

Multiplicando la desigualdad anterior por 1 y respectivamente tenemos

(1 )f (y) (1 )f (x + (1 )y) + hs, (1 )y (1 )(x + (1 )y)i,

f (x) f (x + (1 )y) + hs, x (x + (1 )y)i.

Sumando ambas desigualdades, tenemos que

f (x + (1 )y) f (x) + (1 )f (y).

Observacion 5.8 La funcion puede ser convexa y tener su subdiferencial igual al vacio
en algun punto. En efecto, considere por ejemplo la funcion f : [1, 1] R tal que

f (x) = 1 x2 .

i) Convexidad de f .

La funcion g(x) = 1 x2 es concava en R y h(z) = z es creciente y concava en

R+ , entonces la funcion (h o g)(x) = 1 x2 es concava en [1, 1]. As la funcion

f (x) = 1 x2 = (h o g)(x) es convexa en [1, 1].

ii) f (x) = en |x| = 1.


p
Considere el punto x = 1 y supongamos que existe s f (1), entonces 1 y 2
s(y 1), y [1, 1], lo que es equivalente a

(1 y)(1 + y) s2 (1 y)(1 y), y [1, 1].

Considerando y < 1, en la anterior desigualdad se tiene que (1 + y) s2 (1 y).


Finalmente, tomando y 1 obtenemos que 2 0, lo que es una contradiccion.

140
Ahora consideremos el punto x = 1 y supongamos que existe s f (1), entonces
p p
1 y 2 s(y + 1), esto es, 1 y 2 (s)(y + 1), lo que es equivalente a

(1 y) (s)2 (1 + y), y [1, 1].

Tomando y 1 se tiene 2 0, lo que es una contradiccion.

El siguiente resultado muestra que el subdiferencial de una funcion convexa en cualquier


punto interior es no vacio y acotado.

Teorema 5.13 Sea C un conjunto convexo y f : C R una funcion convexa y x


int(C) entonces f (x) es no vacio y acotado.

Demostracion.
Probemos inicialmente que f (x) es no vacio.
Note que (x, f (x)) Front(Epi(f )) entonces por el Teorema 5.6 (teorema del plano
soporte), existe p := (p, ) Rn R tal que h(p, ), (x, ) (x, f (x))i 0, para todo
(x, ) Epi(f ), esto es,

hp, x xi ( f (x)) 0, (x, ) Epi(f ), (5.16)

y de aqu,
hp, xi f (x) + hp, xi, (x, ) Epi(f ), (5.17)

Como p 6= 0, podemos asumir que ||p||Rn R = 1, esto es, ||p||2 + 2 = 1.


Desde que para todo f (x), el punto (x, ) Epi(f ) y en (5.17) obtenemos que
0 ( f (x)) y as 0.
Por otro lado, por el Teorema 5.11 existe > 0 y M > 0 tal que B(x, ) C y
f (x) f (x) M ||x x||, x B(x, ). Usando este resultado en (5.16) obtenemos que
para todo x B(x, ) se tiene

hp, x xi (f (x) f (x)) M ||x x|| (5.18)

Considere el punto x = x + 2 p y como ||x x|| = 2 ||p||


2
(usando ||p||2 + 2 = 1)
entonces x B(x, ). Reemplazando este valor en (5.18) se tiene que

||p||2 M ||p||.

141
De esta ultima desigualdad se concluye que > 0 y usando este hecho en (5.16) obten-
emos que
1
h p, x xi f (x), (x, ) Epi(f ).

Deniendo s := 1 p y tomando (x, f (x)) Epi(f ) se tiene que f (x) f (x) + hs, x xi,
para todo x C, esto es s f (x) y de aqu f (x) 6= .
Ahora probemos que f (x) es acotado.
s
Sea s 6= 0y considere x = x + 2 ||s||
y as x B(x, ). Aplicando este punto en (5.18)
tenemos que ||s|| M.

Corolario 5.11 . Sea C un conjunto convexo y f : C R una funcion convexa,


semicontinua inferior y x int(C) entonces f (x) es no vacio, convexo y compacto.

Prueba. Simple aplicacion del Teorema anterior y del Lema 5.5

Proposicion 5.4 Sea C un conjunto convexo, f : C R una funcion convexa y x


int(C). Si f es diferenciable en x, entonces f (x) = {f (x)}.

Prueba. Por el Corolario anterior, f (x) 6= . Sea s f (x) y supongamos que f (x)
s 6= 0, entonces por denicion de subgradiente se tiene que

f (y) f (x) + hs, y xi, y C. (5.19)

Por otro lado, como f es diferenciable en x entonces f (y) = f (x) + hf (x), y xi +


o(||y x||), donde
o(||y x||)
lim = 0.
yx ||y x||
Reemplazando el valor de f (y) en la desigualdad (5.19) obtenemos

hf (x) s, y xi + o(||y x||) 0

Tomando en particular y = x t(f (x) s), con t > 0 se tiene

o(t)
hf (x) s, f (x) si + 0.
t

Finalmente, tomando t 0 obtenemos que ||f (x)s|| = 0, lo que es una contradiccion.


Por lo tanto se tiene que s = f (x).

142
5.8 Funciones Convexas Diferenciables
Teorema 5.14 Sea C un conjunto convexo y abierto de Rn y f : C R una funcion
diferenciable en C. Entonces, f es convexa si, y solo si, f (x) f (y) + hf (y), x yi,
para todo x, y C.

Demostracion. Sea f convexa y x, y C, entonces en particular (0, 1] se tiene que


f (y + (x y)) f (y) + (f (x) f (y)), luego
f (y + (x y)) f (y)
f (x) f (y).

Tomando lmite cuando 0 y por la diferenciabilidad de f tenemos hf (y), x yi
f (x) f (y), esto es, f (x) f (y) + hf (y), x yi.
Recprocamente, supongamos que se satisface la desigualdad

f (x) f (y) + hf (y), x yi , para todo x, y C. (5.20)

Probaremos que f es convexa. Sean x, y C y [0, 1]. Tomando los puntos x y


x + (y x) en (5.20) y multiplicando por (1 ) se tiene

(1 )f (x) (1 )f (x + (y x)) (1 ) hf (x + (y x)), y xi . (5.21)

Analogamente, tomando ahora los puntos y y x + (y x) en (5.20) y multiplicando por


se tiene

f (y) f (x + (y x)) + (1 ) hf (x + (y x)), y xi . (5.22)

Sumando (5.21) y (5.22) obtenemos:

f (y) + (1 )f (x) f (x + (y x)) + (1 )f (x + (y x))

= f (x + (y x)),

por lo tanto f (y + (1 )x) f (y) + (1 )f (x).

Teorema 5.15 Sea C un conjunto convexo abierto de Rn y f : C R una funcion dos


veces continuamente diferenciable en C. Entonces, f es convexa si y solo si 2 f (x)  0,
para todo x C.

143
f

)
T (y
x
)
f (x
)+
f (x

Figura 5.13: Graca de una funcion convexa

Demostracion. Sea x C y d Rn un vector arbitrario no nulo. Como C es abierto


entonces existe > 0 tal que x + d C, para todo (0, ). Usando el Teorema 5.14
se tiene
f (x + d) f (x) hf (x), di 0.

Usando la aproximacion de segundo orden de Taylor, tenemos


2 2
h f (x)d, di + o(2 ) 0,
2
dividiendo por 2 y tomando lmite cuando va para cero tenemos hdf (x), di 0.
Recprocamente, supongamos que 2 f (x)  0, para todo x C. Probaremos que f es
convexa, en efecto, sea x, y C; por el teorema del valor medio, existe (0, 1) tal que
1
f (y) f (x) hf (x), y xi = h2 f (x + (y x))(y x), y xi 0.
2
Esto implica que f (y) f (x) hf (x), y xi 0. Por el Teorema 5.14 se tiene que f
es convexa.

5.9 Ejercicios Resueltos


Ejercicio 5.1 Las siguientes funciones son convexas?
Pn
i) f (x) = i=1 (2xi 1)(ln xi ln(1 xi )), xi (0, 1).

144
ii) f (x) = ln(ex1 + ex2 + ex3 ).

Solucion.
Pn
i) Como f (x) = i=1 fi (xi ), donde fi (xi ) = (2xi 1)(ln xi ln(1 xi )). As, la
funcion f es separable con terminos de la misma forma, por tanto, para hallar la
Hessiana de f es suciente analizar la funcion

g(t) = (2t 1)(ln t ln(1 t)), t (0, 1).

Hallando la primera y segunda derivada tenemos respectivamente

1 1
g (t) = 2(ln t ln(1 t)) + (2t 1)( + )
t 1t
1
g (t) = >0
t2 (1 t)2
luego
1
x21 (1x1 )2
0 0

1
0 x22 (1x2 )2
0
f (x) =
2
.. .. ... ..
,

. . .

1
0 0 x2n (1xn )2

de donde se deduce 2 f (x) 0, as tenemos que f es estrictamente convexa y en


particular convexa.

ii) f (x1 , x2 , x3 ) = ln(ex1 + ex2 + ex3 ). Las derivadas de primer y segundo orden de f
son
f exi
= x1 , para todo i = 1, 2, 3.
xi e + ex2 + ex3
2f exi (ex1 + ex2 + ex3 ) exi exi
= , para todo i = 1, 2, 3
x2i (ex1 + ex2 + ex3 )2
2f ex1 ex2
= x1 ,
x1 x2 (e + ex2 + ex3 )2
2f ex1 ex3
= x1 ,
x1 x3 (e + ex2 + ex3 )2
2f ex2 ex3
= x1 .
x2 x3 (e + ex2 + ex3 )2

145
Luego, la matriz hessiana es

1 ex1 ex2 ex1 ex3


2 f (x) = ex1 ex2 2 ex2 ex3 ,

ex1 ex3 ex2 ex3 3

1
donde = (ex1 +ex2 +ex3 )2
y i = exi (ex1 + ex2 + ex3 ) exi exi ,, i = 1, 2, 3. Sea z = (ex1 ,
ex2 , ex3 ) y e = (1, 1, 1), entonces

z 0 0
1 1
T T
2 f (x) = T 2 e z 0 z2 0 zz
(e z)
0 0 z3

Sea v R3 , veamos si se cumple v T 2 f (x)v 0.


" 3
#
1 X
v T 2 f (x)v = T 2 eT z vi2 zi (v T z)z T v
(e z) i=1
" 3 3 3
#
1 X X X
2 2
= T 2 ( zi )( vi zi ) ( zi vi ) .
(e z) i=1 i=1 i=1

Deniendo ai = zi , bi = vi zi , para todo i = 1, 2, 3, tenemos

X3 X3 X3
T 2 2 2
v f (x)v = ( ai )( bi ) ( ai bi )2 0,
i=1 i=1 i=1

donde la desigualdad anterior es obtenida por la desigualdad de Cauchy-Schwartz, por


lo tanto, la funcion f es convexa.

Ejercicio 5.2 Sea C un conjunto convexo y x C. Pruebe que si C = Rn+ , la desigual-


dad hf (x), y xi 0, para todo y C se reduce a


f (x) 0


xi fx(x) =0

i

x 0
i

Solucion.

i) De la hipotesis xi 0 pues x C.

146
ii) Probemos que f (x) 0. Para ello, denamos para cada i = 1, . . . , n

x 0 x1
1

x2 1 1 + x 2
y = x + ei =
.. + .. = ..


. . .

xn 0 xn

luego se tiene que


0

..
.


yx= 1 .

..
.

0

Reemplazando en la desigualdad tenemos para cada i = 1, . . . , n


f (x)
hf (x), y xi = 0,
xi
por lo tanto f (x) 0.

iii) Probemos que xi fx(x)


i
= 0.
n
X
T f (x)
f (x) (y x) = (yi xi ) 0, para todo y Rn+ .
i=1
xi

Ahora tomando y = 0 obtenemos


n
X f (x)
(xi ) 0.
i=1
xi

f (x)
Usando los items i) y ii) se obtiene que xi
xi = 0, para todo i = 1, 2, . . . , n.

Ejercicio 5.3 Si h es una funcion afn y diferenciable pruebe que

h(x) = h(x) + hh(x), x xi

Solucion. De la Decion 5.13 se tiene que h en particular es convexa, luego aplicando


el Teorema 5.14 se tiene que

h(x) h(x) + hh(x), x xi, x Rn (5.23)

147
Tambien, de la Denicion 5.13 se tiene que h es concava, esto es, h es convexa, aplicando
de nuevo el Teorema 5.14 para h obtenemos

h(x) h(x) + hh(x), x xi, x Rn (5.24)

Finalmente de (5.23) y (5.24) obtenemos el resultado.

5.10 Ejercicios Propuestos

148
Captulo 6

Optimizacion Convexa

El problema de optimizacion convexa consiste en resolver el siguiente problema de opti-


mizacion:
opt{f (x) : x X Rn }

donde f : X Rn R es una funcion convexa y X es un conjunto convexo.

Ejemplo 6.1 El problema de optimizacion lineal

min{hc, xi : Ax = b, x 0},

donde A Rmn , b Rm , c Rn , es un problema de optimizacion convexa.

Ejemplo 6.2 El problema de optimizacion


 
1
min hQx, xi hc, xi : Ax = b, x 0 ,
2

donde Q es una matriz simetrica y semidefinida positiva, es un problema de optimizacion


convexa.

Una de las propiedades importantes de la minimizacion convexa es que los mnimos


locales son globales como muestra el siguiente resultado.

Teorema 6.1 Si f : X Rn R es una funcion convexa definida en el subconjunto


convexo X, entonces todo mnimo local es global.

149
Demostracion. Sea x un punto de mnimo local de f , entonces existe (0, 1) tal que

f (x) f (x), para todo x B(x, ) X. (6.1)

Sea y X tal que y 6= x. Por la convexidad de X tenemos que para todo t (0, 1)

z(t) = ty + (1 t)x X.

Por la convexidad de f se tiene

f (z(t)) tf (y) + (1 t)f (x), para todo t (0, 1). (6.2)

Observe que
 

z(t) B(x, ) X, para todo t 0, ,
||y x||
luego usando (6.1) para x = z(t) y (6.2) tenemos que f (x) f (y). As x es un mnimo
global de f sobre X.

Teorema 6.2 Considere el problema de minimizacion convexa

min{f (x) : x X Rn }.

Entonces, el conjunto de las soluciones optimas del problema, X , es convexo.

Demostracion. Sean x , y X , entonces f (x ) = f (y ) = f f (x), para todo


x X. Sea (0, 1) entonces f (x + (1 )y ) f (x ) + (1 )f (y ) = f f (x),
para todo x X. As, x + (1 )y X .

Teorema 6.3 Si f : X Rn R es una funcion estrictamente convexa definida en el


subconjunto convexo X y x es un mnimo de f, entonces x es el unico mnimo global.

Demostracion. Supongamos que exista otro mnimo global y 6= x, as f (y) = f (x).


1
De la convexidad de X tenemos 2
x + 12 y X. Como y es mnimo global y usando la
estricta convexidad de f , se tiene
 
1 1 1 1
f (y) f x + y < f (x) + f (y) = f (x).
2 2 2 2
De esta desigualdad obtenemos f (y) < f (x), lo que es una contradiccion. Por lo tanto
el mnimo x es unico.

150
Teorema 6.4 Si f : X Rn R es una funcion convexa definida en el subconjunto
convexo X. Si x es un mnimo local estricto entonces x es el unico mnimo global.

Demostracion. Analogo a la demostracion anterior.

6.1 Minimizacion Convexa Irrestricta


Teorema 6.5 Considere el problema de minimizacion

min{f (x) : x X Rn }

donde f : X Rn R es convexa en el conjunto convexo X 6= y diferenciable en Rn


entonces, x es un mnimo global si y solo si hf (x), x xi 0, para todo x X.

Demostracion. Sea x un mnimo de f en X y x X un punto arbitrario, entonces por


la convexidad de X se tiene que x + (x x) X, para todo (0, 1). Luego
f (x + (x x)) f (x)
0.

Por la diferenciabilidad de f en x se tiene que
o()
hf (x), x xi + 0.

Tomando lmite cuando converge para cero se tiene que hf (x), x xi 0.
Recprocamente, supongamos que hf (x), x xi 0, para todo x X. Como f es
convexa en x se tiene para todo x X que f (x) f (x) + hf (x), x xi. Ahora, usando
la hipotesis obtenemos que x es un mnimo global de f en X.

Corolario 6.1 Considere el problema de minimizacion sin restricciones

min{f (x) : x Rn },

donde f es una funcion diferenciable en Rn . Entonces, x es un mnimo global si, y solo


si, f (x) = 0.

Prueba. Si x es un mnimo local, tenemos del Teorema 6.5 hf (x), x xi 0, para todo
x Rn . Tomando en particular x = xf (x), se tiene que f (x) = 0. Recprocamente,
si f (x) = 0, del Teorema 6.5 concluimos que x es un mnimo global.

151
6.2 Minimizacion Convexa Restricta
En esta seccion probaremos que si el problema de minimizacion es un problema convexo
donde las funciones que denen las restricciones de igualdad son anes entonces las
condiciones de KKT dadas en el captulo anterior son tambien sucientes. Consideremos
el problema de optimizacion no lineal




min f (x)


(P N L) s.a. : h(x) = 0





g(x) 0

donde h : Rn Rm y g : Rn Rp .

Teorema 6.6 Si en el problema (PNL), f, gj son convexas, hi son funciones afines


(hi (x + (1 )y) = hi (x) + (1 )hi (y), para todo R) y x satisface las condiciones
de KKT :
m q
X X
f (x) + i hi (x) + i gi (x) = 0
i=1 j=1

hi (x) = 0 i = 1, . . . , m (6.3)

gj (x) 0, j = 1, . . . , q (6.4)

j gj (x) = 0, i = 1, . . . , m (6.5)

j 0, i = 1, . . . , m

para algunos i y j , entonces x es un minimizador global del (PNL).

Demostracion. Sea X = {x Rn ; h(x) = 0, g(x) 0} y consideremos x X tal que


x 6= x. De la convexidad de f en x y de la viabilidad del punto x tenemos
m q
X X
T
f (x) f (x) + f (x) (x x) + i hi (x) + i gi (x).
i=1 j=1

Usando la denicion de funcion afn de hi y de la convexidad de gj en x se tiene


m q
X X
T T
f (x) f (x)+f (x) (xx)+ i hi (x)+i hi (x) (xx)+ i gi (x)+i gi (x)T (xx).
i=1 j=1

152
De (6.3) y (6.5), la desigualdad anterior se reduce a
m q
X X
T T
f (x) f (x) + f (x) (x x) + i hi (x) (x x) + i gi (x)T (x x).
i=1 j=1

Asociando los gradientes de f, hi y gj tenemos

m q
!T
X X
f (x) f (x) + f (x) + i hi (x) + i gi (x) (x x).
i=1 j=1

Usando (??), nalmente obtenemos f (x) f (x).

Ejemplo 6.3 Considere el problema

min{x1 2x2 : 2x1 + x2 6; x1 + x2 4; x1 0; x2 0}.

Vimos en el ejemplo 4.13 que el unico candidato a solucion del problema es (0, 4).
Ahora, como el problema es un problema de optimizacion convexa entonces aplicando el
Teorema 6.6 tenemos que (0, 4) es un unico mnimo global.

Ejemplo 6.4 Considere el problema lineal

(P L) min{cT x : Ax = b, x 0},

donde A Rmn , b Rm , c Rn . La funcion objetivo f (x) = cT x es lineal y as


convexa, la funcion h(x) = Ax b es una funcion afn y g(x) = x es lineal y de aqu
convexa, entonces del Teorema 6.6 tenemos que la condicion necesaria y suficiente para
obtener los puntos de mnimo global son


AT + s = c




Ax = b

xi s i = 0




x ,s 0
i i

Ejemplo 6.5 Considere el problema


 
1 T T
(P C) min x Qx c x : Ax = b, x 0 ,
2

153
donde Q Rnn es una matriz simetrica y semidefinida positiva, A Rmn , b Rm ,
c Rn . La funcion objetivo f (x) = 12 xT Qx cT x es convexa, la funcion h(x) = Ax b
es una funcion afn y g(x) = x es lineal y de aqu convexa, entonces del Teorema 6.6
tenemos que la condicion necesaria y suficiente para obtener los puntos de mnimo global
son:

AT + s = Qx + c




Ax = b

xi s i = 0




x ,s 0
i i

6.3 Convexidad Generalizada


Definicion 6.1 Una funcion diferenciable f : Rn R es pseudoconvexa si, para todo
par de puntos distintos x, y tenemos

hf (x), y xi 0, entonces f (y) f (x).

Definicion 6.2 Sea C un conjunto convexo y f : C R una funcion real. f es llamada


cuasi-convexa en C si para todo x, y C, t [0, 1], se cumple que

f (tx + (1 t)y) max{f (x), f (y)}.

Teorema 6.7 Sea C un conjunto convexo y f : C R una funcion. f es cuasi-convexa


en C si, y solamente si, para cada c R el conjunto de nivel de f , Lf (c) = {x C :
f (x) c}, es convexo.

Demostracion. Supongamos que f es cuasi-convexa y sea c R. Probaremos que Lf (c)


es convexo. Sea x, y Lf (c), entonces f (x) c, f (y) c y por tanto max{f (x), f (y)}
c. Usando la cuasi-convexidad de f y el resultado previo, tenemos que

f (tx + (1 t)y) max{f (x), f (y)} c.

De la desigualdad anterior obtenemos que Lf (c) es convexo. Recprocamente, supong-


amos que Lf (c) es convexo para cada c R. Probaremos que f es cuasi-convexa en C.

154
Sea x, y C y denamos c := max{f (x), f (y)}. De lo anterior tenemos inmediatamente
que x, y Lf (c) y como Lf (c) es convexo, esto implica que

tx + (1 t)y Lf (c).

Finalmente, por la denicion de Lf (c) obtenemos nuestro resultado.

Teorema 6.8 Sea f : Rn R una funcion diferenciable y cuasi-convexa y sea x, y Rn .


Si f (x) f (y) entonces hf (y), x yi 0.

Demostracion. Sea t (0, 1). Por la aproximacion de primer orden de una funcion
o(t)
diferenciable tenemos f (y+t(xy)) = f (y)+thf (y), xyi+o(t), donde limt0 t
= 0.
Como f es cuasi-convexa, esto implica que

f (y) + thf (y), x yi + o(t) max{f (x), f (y)}.

Usando la hipotesis de que f (x) f (y) y dividiendo por t obtenemos

o(t)
hf (y), x yi + 0.
t

Tomando lmite cuando t converge a cero, obtenemos el resultado.

6.4 Ejercicios Resueltos


Ejercicio 6.1 Sea A Rnn una matriz simetrica y semidefinida positiva. Pruebe que
los siguientes problemas son equivalentes

1. Resolver el sistema lineal Ax = b.

2. min{f (x) = 12 xT Ax bT x : x Rn }.

Solucion. Observemos inicialmente que f es convexa ya que 2 f (x) = A es semidenida


positiva y por el uso del Teorema 5.15.
Sea x solucion del sistema lineal Ax = b. Probaremos que x es mnimo global de f.
Como f (x) = Ax b = 0, f es convexa y por el Corolario 6.1 se tiene que x es un
mnimo global de f.

155
Reciprocamente, supongamos que x es un mnimo global de f. Probaremos que x resuelve
el sistema lineal Ax = b.
Como f es convexa y x es un mnimo global de f se tiene por Corolario 6.1 (tambien
puede usar el Teorema 4.1 del Captulo 4) que f (x) = Ax b = 0, esto es, x resuelve
el sistema lineal.

Ejercicio 6.2 Sean las funciones

f (x1 , x2 ) = x21 + x22 ; g(x1 , x2 ) = (x1 1)2 x22 .

(a) Determinar todos los puntos que puedan ser extremos locales de la funcion f sujeta
a la restriccion g(x1 , x2 ) = 0.

(b) Determinar si los puntos obtenidos en item anterior dan maximos o mnimos de f
con la restriccion establecida.

Solucion.

(a) Hallemos los candidatos a solucion. La funcion lagrangiana es L(x1 x2 , ) = x21 +


x22 + ((x1 1)2 x22 ) donde

x L(x1 x2 , ) = (2x1 + 2(x1 1), 2x2 2x2 ).

Luego, las condiciones de optimalidad son

x1 + (x1 1) = 0 (6.6)

(1 )x2 = 0

(x1 1)2 x22 = 0. (6.7)

Si x2 = 0 entonces de (6.7) x1 = 1, esto contradice (6.6). As x2 6= 0 y = 1, esto


implica que x1 = 1/2 y x2 = 1/2. Luego, los puntos candidatos a solucion con
= 1 son
(1/2, 1/2) y (1/2, 1/2).

156
Como la unica restriccion es g(x1 , x2 ) = (x1 1)2 x22 y su gradiente es g (x1 , x2 ) =
(2(x1 1), 2x2 ) entonces

Ker(g (x1 , x2 )) = {(v1 , v2 ) | v1 (x1 1) x2 v2 = 0}

y la matriz hessiana del lagrangiano esta dado por



2 + 2 0
2xx L(x1 , x2 , ) = .
0 2 2

Analicemos en cada punto candidato a solucion. Para x = ( 12 , 21 ), = 1 se tiene

Ker(g (1/2, 1/2)) = {(v1 , v2 ) | v1 = v2 }

y
 
1 1 4 0
2xx L , ,1 = .
2 2 0 0

As, para todo v = (v1 , v2 ) Ker(g ( 12 , 21 )), v 6= (0, 0) tenemos que

v T 2xx L(1/2, 1/2, 1)v = 4v12 > 0,

luego aplicando la condicion suciente de segundo orden concluimos que el punto


(1/2, 1/2) es un mnimo local estricto. Para el punto x = (1/2, 1/2), = 1,
tenemos
Ker(g (1/2, 1/2)) = {(v1 , v2 ) | v1 = v2 }

y
 
1 1 4 0
2xx L , ,1 = .
2 2 0 0

As, para todo v = (v1 , v2 ) Ker(g ( 12 , 12 )), v 6= (0, 0) tenemos que

v T 2xx L(1/2, 1/2, 1)v = 4v12 > 0.

Luego aplicando la condicion suciente de segundo orden concluimos que el punto


(1/2, 1/2) es un mnimo local estricto.

157
(b) Para ver que (1/2, 1/2), (1/2, 1/2) son mnimos globales tenemos que ver si f es
convexa y g es afn. En efecto, es claro que f es convexa, pues la matriz hessiana
de f ,
2 0
2 f (x) = ,
0 2
es denida positiva. Pero la funcion g no es convexa, entonces no podemos armar
que (1/2, 1/2), (1/2, 1/2) sean mnimos globales.

Ejercicio 6.3 Sea A Rnn una matriz simetrica. Pruebe que resolver el problema
min{f (x) = 12 xT Ax : ||x|| = 1} implica hallar el menor autovalor de A.

Solucion. Observemos inicialmente que el problema tiene solucion ya que la funcion


objetivo es continua y el conjunto de las restricciones es compacto (ver Corolario 3.1).
p
Sea h(x) = ||x|| 1 = hx, xi 1, entonces se tiene que
1 2x x
h(x) = p =
2 hx, xi ||x||
 T
x
||x||I x ||x||  
2 1 xxT
h(x) = = I .
||x||2 ||x|| ||x||2
Ademas, como para todo x tal que ||x|| = 1 se tiene que h(x) = x 6= 0, entonces todo
punto viable es regular.
Sea x una solucion del problema de optimizacion entonces usando el Teorema 4.9 se tiene
que existe un unico R tal que x L(x, ) = 0, donde L(x, ) = 21 xT Ax h(x). Como
x L(x, ) = Ax x se tiene que
Ax = x,

esto es, es un autovalor de A y x es su respectivo autovector.


Probaremos nalmente que es el menor autovalor de A. Como x es un punto de mnimo
de f (x) = 21 xT Ax sobre ||x|| = 1, entonces

xT Ax xT Ax. (6.8)

Sea un autovalor de A, luego existe x Rn tal que ||x|| = 1 satisfaciendo Ax = x.


Reemplazando esta ultima expresion en (6.8) se tiene que , as es el menor
autovalor de A.

158
Ejercicio 6.4 Sea f : Rn R una funcion semicontinua inferior y limitada inferior-
mente. Dados p Rn+ y > 0, pruebe que el problema:
n
X xi
min{f (x) + (xi ln( ) + pi xi ) : x Rn+ }
i=1
pi

tiene solucion.
Pn
Solucion. Es suciente probar que la funcion h(x) = i=1 (xi ln( xpii ) + pi xi ), es
coerciva en Rn+ . Sea {xk } una sucesion crtica entonces kxk k +, es decir, (xk1 )2 +
(xk2 )2 + ... + (xkn )2 +, as existe un ndice i0 {1, 2, ..., n} tal que |xki0 | +, por
la restriccion del problema se tiene xki0 +.Recordemos que la funcion g(z) = z ln z
es convexa en R+ , entonces

g(z) g(t) + g (t)(z t).

xi xi
Tomemos z = pi
y t = 1, entonces pi
ln xpii xi
pi
1 lo cual equivale a xi ln xpii xi + pi 0.
As, h(x) xi ln xpii xi + pi , para todo i = 1, 2, ..., n.
En particular se tiene
xi0 k
h(xk ) xi0 k ln xi0 k + pi0
pi0
de aqu se deduce que h(xk ) xi0 k (ln xi0 k ln pi0 1) + pi0 como xi0 k +, entonces
xi0 k (ln xi0 k ln pi0 1) +. Por lo tanto, h es coerciva.

Ejercicio 6.5 Sea S abierto y convexo en Rn .


h : S Rn R es llamada frontera coerciva si para todo {y k } Rn tal que y k y
F ront(S), se tiene que limk+ hh(y k ), x y k i = , x S. Pruebe que h(x) =
Pn n
i=1 xi ln xi es frontera coerciva en R+ .

Solucion. Sea hi (xi ) = xi ln xi , entonces hi (xi ) = ln xi + 1. As



k
ln y1 + 1

k
ln y2 + 1
h(y ) =
k
..


.

k
ln yn + 1

159
Luego, se tiene que
n
X
hh(y k ), x y k i = (ln yik + 1)(xi yik ).
i=1

Sea {y k } una sucesion tal que y k y F ront(Rn+ ), entonces existira un i0 {1, ..., n}
tal que yik0 0. Tomando limite cuando k + al producto interno
n
X
hh(y k ), x y k i = yik ln yik + xi ln yik yik + xi , k +.
i=1

Ejercicio 6.6 (Mnimos cuadrados). Dados los puntos: P1 = (x1 , y1 ), P2 = (x2 , y2 ),


. . . , Pn = (xn , yn ) del plano, queremos encontrar la recta y = mx + b tal que la suma de
los cuadrados de las distancias verticales de estos puntos a la recta sea mnima, esto es,
queremos minimizar
n
X
f (m, b) = (yi mxi b)2 .
i=1

Demuestre que el mnimo ocurre cuando:


X n n
X n
X

2

x i m + x i b = xi yi
i=1 i=1 i=1
n
X Xn



xi m + nb = yi .
i=1 i=1

n
X
Solucion. Tenemos que f (m, b) = (yi mxi b)2 es convexa, (por ser suma de
i=1
funciones convexas), y esta denida en R2 que es un conjunto convexo. Entonces el
problema de minimizar f es un problema convexo, as bastara resolver f (m, b) = (0, 0)
para encontrar los puntos de mnimo global.
En efecto:
n
X n
X
f (m, b) = (2 (yi mxi b)(xi ), 2 (yi mxi b)(1)) = (0, 0)
i=1 i=1

entonces: X
n



(yi mxi b)xi = 0
i=1
n
X



(yi mxi b) = 0
i=1

160
luego se tiene X
n n
X n
X

2

x y
i i m x i b xi = 0
i=1 i=1 i=1
n
X X n



yi m xi nb = 0
i=1 i=1

o equivalentemente, X
n n
X n
X

2

xi m + xi b = xi yi
i=1 i=1 i=1
n
X Xn



xi m + nb = yi .
i=1 i=1

Ejercicio 6.7 Encuentre la recta de mnimos cuadrados (ver el problema anterior) para
los datos: (3, 2), (4, 3), (5, 4), (6, 4) y (7, 5).

7
6
p5
5
p3 p4
4
p2
3
p1
2

1 2 3 4 5 6 7

Figura 6.1:

Solucion. Del ejercicio anterior tenemos:


X n Xn n
X

2

xi m + xi b = xi yi
i=1 i=1 i=1
n
X Xn



xi m + nb = yi .
i=1 i=1

161
Por dato tenemos los puntos P1 = (3, 2), P2 = (4, 3), P3 = (5, 4), P4 = (6, 4) y P5 = (7, 5).
Reemplazando los puntos en las ecuaciones anterior se tiene

(9 + 16 + 25 + 36 + 49)m + (3 + 4 + 5 + 6 + 7)b = (3)(2) + (4)(3) + (5)(4) + (6)(4) + (7)(5)


(3 + 4 + 5 + 6 + 7)m + 5b = 2 + 3 + 4 + 4 + 5

As debemos resolver el sistema



135m + 25b = 97
25m + 5b = 18

7 1
Resolviendo tenemos que m = 10
yb= 10
. Por lo tanto la recta es 10y = 7x + 1.

1
x+
7
0y =7
1
6 l:
5

1 2 3 4 5 6 7

Figura 6.2: La recta 10y = 7x + 1 es la recta mas proxima a los puntos dados

6.5 Ejercicios Propuestos

162
Bibliografa

[1] Alizadeh, F. Interior Point Methods in Semidenite Programming with Applica-


tions to Combinatorial Optimization, SIAM J. Optimization, v. 5, n. 1, pp.13-51,
1995.

[2] Barbolla R., Cerda E., Sanz P., Optimizacion: cuestiones, ejercicios y aplicaciones
a la economa, Prentice Hall, Espana, 2001.

[3] Bartle R.G., Introduccion al Analisis Matematico, LIMUSA, Mexico, 1980.

[4] R.S. Burachik, L.M. Grana Drummond, A.N. Iusem and B.F. Svaiter, Full conver-
gence of the steepest descent method with inexact line searches, Optimization, v.
32, pp. 137-146, 1995.

[5] Cauchy, A., Methode Generale pour la Resolution des Systemes dEquations Si-
multanees, Comptes Rendus de Academie des Sciences, Paris, v. 25, pp. 536-538,
1847.

[6] Donath, W.E., Homan, A.J., Lower Bounds for the Partitioning of Graphs, IBM
J. Res. and Devel., v. 17, pp. 420-425, 1973.

[7] O. Guler, On the convergence of the proximal point algorithm for convex mini-
mization, SIAM J. Crontrol and Optimization, v. 29(2), pp. 403-419, 1991.

[8] A.N. Iusem and B.F. Svaiter, A proximal regularization of the steepest descent
method, RAIRO Recherche Operationelle/Operation Reseach, v. 29 (2), pp. 123-
130, 1995.

163
[9] Izmailov A. and Solodov M., Otimizacao Volume 1, IMPA, Rio de Janeiro, Brazil,
2005.

[10] K. Kiwiel and K. Murty, Convergence of the steepest descent method for mini-
mization quasiconvex functions, JOTA Journal of Optimization Theory and Appli-
cations, v. 89 (1), pp. 221-226, 1996.

[11] Lages Lima, E., Curso de Analise, v. 2, IMPA, Rio de Janeiro, 2005.

[12] B. Martinet, Regularization, dinequations variationelles par approximations suc-


cessives, Revue Francaise qInformatique et de Recherche Operationelle, pp. 154-
159, 1970.

[13] Martinez J.M. and Santos S. A., Metodos Computacionais de Otimizacao, 20 Colo-
quio Brasileiro de Matematica, IMPA, 1995.

[14] Overton, M.L., Womersley, R.S., Optimality Conditions and Duality Theory for
Minimizing Sums of the Largest Eigenvalues of Symmetric Matrices, Math. Pro-
gramming, v. 62, pp. 321-357, 1993.

[15] Papa Quiroz E.A, Oliveira P.R., A New Barrier for a Class of Semidenite Opti-
mization, RAIRO Operations Research, v. 40, p. 303-323, 2006.

[16] E.A. Papa Quiroz E.M. Quispe, and P. Roberto Oliveira, Steepest descent method
for quasiconvex minimization on Riemannian manifolds, Submmited, 2006.

[17] Papa Quiroz, E. A. and Oliveira, P. R. (2006), Classic and logarithmic-


quadratic proximal point method for quasiconvex minimization. Optimization On-
line, Preprint.

[18] Souza S., Oliveira P.R., Cruz Neto J.X. and Soubeyran A., A proximal Point Al-
gorithm with Bregman Distances for Quasiconvex Optimization over the Positive
Orthant. Por aparecer en la revista European Journal of Operational Research.
Preprint, 2006.

164
[19] R.T. Rockafellar, Monotone operators and the proximal point algorithm, SIAM J.
Control and Optimization, 14 (5) 1976, 877-898.

[20] R.T. Rockafellar and R. Wets, Variational Analysis. Grundlehren der Mathematis-
chen, Wissenschaften, 317, Springer, 1998.

165

También podría gustarte